Jan-Feb 2024 Questions _ ابطال الديجتال

Download as pdf or txt
Download as pdf or txt
You are on page 1of 314

🟡 Jan-Feb 2024 Questions | ‫ابطال الديجيتال‬

✅🟢
= The answered question with reference
= Answer without a reference

🔁 = Not sure about the answer


= The correct answer

‫اي شخص عنده حل اخر لالسئلة سوا في الملف هذا او الملفات السابقة او حاب يضيف معلومة على االسئلة اللي في الملفات‬
[email protected] ‫يراسلني على االيميل‬
‫االجوبة قد تحتمل الخطا فنصيحة اي سؤال ماعليه رفرنس تاكدوا منه بنفسكم‬

A. 7 days
B. 1month

1. Hepatitis virus active in room temperature

C. 8month
D. 12moth


2. young patient presents to the clinic appearance moon face pink cheeks ,swelling in back and neck,fat and large
abdomen, but the legs are thin
a. cushing's syndrome


3. How to avoid adrenal crisis


4. definition of pseudo class iii malocclusion

a. Must the consent from parents


6. Shank of the instrument during scaling?

5. 14years old came without her parents and ask for dental diamond

a. Perpendicular to long axis


b. Parallel to long axis
c. 45 degree

d. 75 degree

7. pt came for scaling and superficial filling he did transplant 2 month ago
a. Defer treatment 6 month


8. Comparing Three Unit FPD replacing upper canine with three unit FPD replacing lower canine on the same side,
which statement is true?
a. Upper canine is subjected to more lateral forces
b. Lower canine is subjected to more lateral forces

c. Upper fpd doesn’t follow, ante’s low
d. Lower fpd doesn’t follow ante’s low


9. pedo pt with ankylosed primary molar and missing permanent
a. Refer to specialist

a. Refer to endodontist✅
10. pt after giving anesthesia refuse to do endo treatment with rubber dam

11. hypertension or hypotension patient i can't remember taking perindopril 20ml "While talking to the doctor, he began

a. Overdose medication
b. Misuse of medication

to feel tired and dizzy, became nervous

c. Hyper or hypothyroidism
d. Hypotension


12. pic of angular cheilitis mouth ulcer and he mentioned something on ocular

Note :Mostly Plummer syndrome but if ocular involvement the answer could be pemphigus
13. pt with asymptomatic lesion on checks can’t rubes off but disappear if you stretch the cheeks
a. Leukodema


14. pic of missing 6 and tilted 7 and 5 , ask about the treatment to replace the 6
a. Uprighting the 7 and replace it with implant ( Ideal option )

a. Do Implant over denture


b. Adjust

15. You did complete denture for middle aged pt now he complains of difficulty in eating

16. a patient came to the clinic to continue his orthodontic treatment, upon examination he has multiple caries and heavy

a. Permanent crowns
b. Root canal therapy

calculus and severely destructed teeth. Which of the following is the treatment after he finishes ortho.

c. remove caries and restorations.


d. Scaling root planing


17. Deep carious lesion and indicated for extraction. Which of the following criteria help in choosing type of extraction?

b. 3 site caries
c. Pulpal pain

a. Gingival swelling in buccal

d. Percussion


18. patient came to do implant, he has missing #17, #15,#14 and #46( pic provided with supraerupted #16 and limited
space on #15,#14). What is the best treatment?
a. place implant (after ortho )


b. ortho treatment ( most Ideal option )
c. Removable (best option in this case )
d. fixed (can not be used )


19. a patient came with upper Kennedy class 2 no modification and he has high buccal frenum. What is the clasp to be
done on tooth #24?
b. RPA ✅
a. RPI ( contaidicatio)

c. Circlet
d. Ring


20. Pt has CD she has redness under the denture she told the dentist that she dose not remove the denture during night
a. Explain to her she need to remove the denture


21. young pt had car accident and he lost 12,11,21,22 all the other teeth is good what is the kennedy Classification
a. Class 4


22. pic of upper lateral broken it was build up with post ask about the treatment (depend on the picture but mostly
extrusion due to the aesthetic demand )
a. Extraction and implant


b. Redo the post and crown
c. Ortho extrusion


23. Xray of lateral radiolucent on 45?
a. lateral canals


24. Ulcers on tongue and gingiva ?
a. pemphigoid
b. mucous pemphigoid
c. Pemphigus vulgaris ✅

25. Old pt present with ridge resorption atrophy of tongue and loss of all the teeth she complain from burning sensation
of tongue
a. Improve nutrition ✅


26. Burs can be protected during autoclave by keeping them submerged in
a. 2% sodium nitrite solution


27. porcelain etchant
a. hydrofluoric acid 9.5 to 10 %


28. dental assistant had injury while the surgery by some instrument she get fever after and they give her sick leave
a. take a sick leave


29. pt after extraction he suffer from continuous bleeding
a. Blood test ( good history before the treatment )✅

30. sickle cell disease pt present with several pain he need rct or exo i can’t remember, he ask what should do before
treatment
a. ensure hydration ✅


31. after extraction of upper 6 you determine 2mm oroantral communication what to do
a. Gel foam applied then figure 8 suture

a. lipoxins ✅
32. which of the following is an indication of gingival inflammation resolution?

b. Prostaglandin lysozyme

33. RPD cause mobility on abutment tooth because what
a. reciprocal arm fit after the retentive arm ✅

a. Candidal infection
b. lichenoid reaction

34. Patient has asthma pic of mouth (picture of oral thrush )


35. Patient need rpd and he has 35 (34 and 36 37 missing ) what to do ?
a. Rest mesial and distal to 35


36. Patient bruxer has fracture amlgam what to do ?
a. Composite
b. ceramic inlay
c. Cast restoration ✅


37. Short trunk of 27 you need to do crown lengthening what are you afraid of ?
a. Biologic invasion
b. Furcation invasion ✅
a. High level disinfection
b. Heat sterlization

38. Cheek retractor which how to sterlize it ?

c. Medium level


39. How to refine the access for C shape canal ?
a. Ultrasonic


40. Tooth # 16 has furcation grade lV how to manage it ?
a. Odontoplasty
b. Hemisection
c. Resection of root ✅

41. Patient has white lesion in buccal mucosa with redness of gingiva always take antiviral and it recur what to give this
time ?
a. Antibiotic
b. Fluconoide ( if it is Fluconazole will be the right choose)
c. Nystatin mouth wash

d. Preidonsolon


42. Patient allergic to avocado , during examination he developed itching and rash. What to do ?
a. Remove latex gloves and give him antihistamine and call er


43. Implant what should you do ?
a. High cusp
b. narrow occlusal table ✅


44. PA ot tooth 32 with very large resorption of bone and RL and tooth is vital ask is it due to( Tooth #31 is implant and

a. Perio
b. Endo

has no enough space so it cause resorption to the bone adjacent to 32)

c. Perio Endo
d. Endo Perio


45. Patient is not satisfied with her smile and the teeth is short


a. CL and crown ( if there’s discoloration will be the right choice)
b. CL alone

a. CL and crown
b. CL

46. another question the same but there is very dark staining of all teeth

47. Patient has problem with eye , cant wrinkle face ?


a. Needle went to parotid gland

a. Incisional biopsy
b. Excisional biopsy

48. Lesion in bone 8X6 CM cause teeth displacement and root resorption management ?
( b.c it is more than 1 cm )

c. Surgical resection
d. Follow up


49. Which type of dentine deposit with very deep caries ?
a. Reactionary
b. Reparative ✅

50. Patient has severe pain when you touch his face and after the attack episode is relieved if you touch the face again
there will be no pain. Which condition ?
a. Multiple sclerosis (This is called remission period and its golden sign for trigeminal neuralgia )

51. very limited interarch space and you want to do RPD what to do ?
a. Acrylic base
b. Meshwork
c. Metal base ✅


52. Lower anterior compromised and posterior is missing which of the following acts as indirect retainer ?
a. Lingual plate


53. There is no undercut in the abutment and the tooth is sound with no caries , restoration , endo nothing its fully sound
what to do ?


a. Surveyed crown
b. Dimple


54. While you do post prep there was bleeding from the canal and it didn't stop management ?
a. immediate MTA
b. PA radiograph ✅

55. most fracture occur in:
a. Coronoid
b. Body
c. Angle ✅
d. Symphysis


56. PA of immature central that has trauma before long time and its necrotic :
a. RCT
b. RCT with MTA
c. Pulpotomy


57. you did condensation for amalgam for 8 minutes what will happen?
a. Easy to carve
b. Amalgam will be set ✅
c. Residual mercury increase

a. 0.02
b. 0.2

58. ISO Diameter of K file :

c. 2


59. you have three canal , MB ML D at the end of instrumentation you couldn't negotiate the ML (‫قبلها كنت توصل لها عادي‬
‫ )ووضعك سليم‬And in the radiograph there was nothing

b. Ledge ✅
a. Perforation

c. Separated file

a. Ameloblastoma
b. Osteosarcoma

60. CBCT pic of expansion adjacent to bone what is the diagnosis ?


61. patient said i have pain with hot drink and food and it reliefs with cold , endo diagnosis is
a. Necrotic
b. Normal
c. Symp irr
d. Asym irr

62. Picture of overhang on two teeth with localized bone lose on the crestal bone adjacent to overhang , main etiologic

a. Plaque ✅
factor for this case?

b. Occusal trauma

a. V shape palate
b. U shape palate

63. Resist horzontal movment but poor vertical ?

a. Supra scaling and antibiotic


b. Deep scaling

64. Patient came to you and he has NUG he is very feverish and his high temp :

c. OHI and mouth wash

65. patient has long dental treatment while opening his mouth , no sublaxation , no lock , no crepitus just pain and
clicking :

b. 2
c. 3

a. Wilkes 1

d. 4

a. Autoclave
b. Chemiclave

66. What cause corrosion of carbon steel instrument ?

c. Dry heat
67. Pic of very very small point of pulp exposure before three days in central of 7 years child ?
a. Pulpotomy
b. Cervical pulpotomy
c. Pulpectomy

d. DPC with MTA

a. Infiltartive mask ✅
68. Patient has TB emergency case and he is active TB :

b. First patient in the morning


c. Regular setting

69. Taurdontitism is ?
a. Amelogenesis imp
b. Dentinogenesis imp

70. sever periodontits case after finish active treatment in all the recall visit there was residual multipile pocket of >5
mm what it indicate ?
a. Low chance of recurrence of periodontits
b. High chance of recurrence of periodontitis
c. Case is going for advanced periodontitis

71. Patient is hypertensive controlled , His MI before 3 years , Prosthetic heart valve ,Taking anticoagulant (No dose
provided ) need extraction :
a. Prophy antibiotic
b. Regular patient
✅️
c. Hospital setting


72. Sensitivity during prep on enamel ?
a. Tuft
b. Lamella
c. Spindle
d. Rods
✅️


73. Chart with result of Platelet,Aptt ,PT and said that patient has problem since childhood ?(‫)ناقص نتايج الفحوصات‬
a. VWB
b. Hemophilia a
c. Thrombocytopenia

74. Case about adenoid cystic carcinoma (Mention perinural invasion )


75. 7 years has Ant open bite , Always tongue out , occasionally thumb sucking and rarely tongue thrust while

a. Thumb ✅️
swallowing what is the cause of open bite ?

b. Always tongue forward position


c. Swallowing while tongue thrust
76. Patient has perfect denture but there is rednees in all area , other wise the fit is perfect , what is the cause ?
a. Poor fit
b. Wear it all the time ✅️
c. Poor relief for some area
d. increase VD

a. Edema of soft tissue from extraction


b. Wearing skills of patient is poor
✅️
77. Patient has immediate denture and it was perfect in the first day , second day he couldn't wear it why ?


78. Whats the most destructive force on abutment tooth ?
a. vertical
b. horizontal ✅️
c. Vertical close to the ridge
d. vertical away from the ridge

a. Conflict of interest
b. Autonomy
✅️
79. Dentist cooperate to work with implant company to use there implant , what the doctor has violate ?

c. non maleficence
d. denteology

a. Floucanazol
b. valcyclovair
✅️
80. HIV Pt came with white spongy on the palate shows erythmous after scrapping what should you prescribe for the pt

c. paracetamol
d. prednisolon


81. Dental assistant cleaned the burs and instrument in the clinic then sent to CSSD , the technician did manual cleaning
then put utility gloves and start sterilization …. What should be corrected on the previous scenario ?
a. should put gloves before cleaning

✅️
b. technician should not clean the instrument in CSSD
c. assistant should not clean in the clinic


82. How should the technician set up posterior teeth for patient with skeletal class II ?
a. Lower slightly buccal
b. upper slightly buccal
c. upper slightly palatal ✅️
d. upper on the crest of the ridge


83. There was a case vital tooth with periapical radiolucency asking if it is
a.
b.
c.
perio origin
endo origin
✅️
true combined lesion


84. Pt complaining about her appearance she is 11 years i think with Ceph xray , xray shows deep bite with incisors
maxillae shows excess vertical growth ( it looks like ) Asking about the reason of her appearance
a. skeletal class III
b. Skeletal class II ✅️
c. Vertical excess maxilla

d.
85. Down syndrome pt showed up and he needs immediate extraction but the parent are not answering the phone, who
should sign the consent ?
a. The nurse
b. the doctor
c. the patient
✅️
d. the proxy decision maker


86. Pt has white spots on the molar and incisor region and looks hypo plastic
a. Fluorosis
b. Dentinal dysplasia
c. Dentinogenesis imperfecta
d. Molar incisor hypomineralization ✅️
a. Turner dysplasia ✅️
87. pt has discoloration on all of her teeth while the premolar has brown discoloration what's that ?

b. amelogenesis imperfecta
c. fluorosis
d. syphilis

88. Pt has crohn disease will controlled with his medication and you will perform RCT what consideration should be
done ?
a. treat normally ✅️
b. stop corticosteroid a day before the procedure
c. Double the dose

a. Neglect ✅️
89. Child with multiple abscesses , caries , medical care looks neglected

b. sexual abuse
c. physical abuse


90. Dentist using latex gloves and Polysulfide impression, after the setting time of the manufacturer dentist remove it and
impression was not setting ?
a. sulfur contamination
b. wrong paste proportion
✅️


91. What would happen if excess monomer is there during packing ?
a. granules will appear on the palate surface of the denture
b. the pink color will not show
🔁


92. Image of a case where #24,26,27 is missing and #25 is pier abutment, tooth has MOD amalgam and recession with
grade I mobility, what's your management ?
a. Extract #24
b. Survey crown
c. M & D rest seat
🔁
d. Mesial rest seat
93. Image for tooth #46 has grade II buccal furcation asking about the Tx
a. Furcation plasty
b.
c. ✅
Tunnel preparation
GTR

a. 1-2
b. 3-4

94. How many days it takes for traumatic ulcer under the CD to appear ?

c. 5-6
d. 7-8
95. Pt came complin of broken tube and irritating wire and his appointment after 2 weeks?
a. remove the wire
b. cut the excess
c. tell him to wait

a. ECG ✅
96. Pt having ascites , neck pain, can’t tolerate exercise and he’s having fatigue?

b. CHEST Xray
c. blood pressure’


97. #36 is missing you want to fabricate a bridge from 35 – 37 but #26 is supra erupted interference which side?
a. Protrusive

a. pterygomandibular raphe
b. Max tuberosity.

98. What you can see in maxilla and mandible during impression taking?

c. Retromolar pad.
d. Humular notch.

a. CT
b. OPG

99. Pt having periorbial trauma or bleeding + echemyosis which radiograph?

c. Submentovertex
d. occibtovertial

100.

101.
a. Disclusre to the pt
External hex?

U broke a file and bybass it what is the best management?

a. 5
b. 7
c. 10

102. Pt 14 years have crowding upper and lower + missing upper canine?
a. Extraction
b. Expansion ✅
c. nonextraction
103.

Case in Stillman cleft and nail baiting habits recession reaching to near MJG what is the treatment?
a. Connective tissue graft
b. RMGIC

104. Pic after removal of badly broken bridge and carious teeth with inflamed gingiva ?
a. Take impression
b. disease control ✅
c. crown lengthening
105. Cleaning of instrument in sink?
a. Flat
b. fully immersed
c. partly

106. Asthma management?


a. Nitrous oxide ✅


107. Pregenat pt 28weeks starting to feel an enlargement from her right side from 2 months, a large gingival

a. pregnancy tumour ✅
enlargement on #15 buccal & palatal they give clinical pic+ xray ?

b. lateral periodontal cyst


108. Pt have multiple fillers + cosmetic surgery want to do her chin also , she’s underweight this known as ?
a. Aneroxua nervousa
b. body dysmorphic syndrome ✅

109. Diabetic pt vital tooth?


a. Incision & drainage ✅

110.

Diabetic require extraction + acute abscess + swelling ?
a. postop antibiotic
b. 1 hour before as prophylaxis

111. interdental inflammation #11,12,13 this is ?


a. localized diffuse
b. localized marginal
c. localized papillary gingivitis✅

112.

Bone loss to appear in radiographs ?
a. 30-50%

113.

Periodinse pt have lower teeth mobile grade 1 , calclus pd – 5mm what to do ?
a. scaling and rooth planning then evaultation after 4-6 weeks
b. extraction
114.

Pt have PD 3-5mm , bone loss 20% all around diagnosis ?
a. Generalized stage II grade b
b. localized stage II grade b

115. Pt have all upper teeth , lower only the anterior lingually tilted what to do?
a. Extraction
b. swing lock rpd
c. immediate


116.

Pedo pt multiple surface caries 4 y?
a. SSC

117. Pedo pt multiple coloured restoration fluoride ?


a. 1100 ppm fluoridated toothpaste , 0.05 fluoride mouthwash ✅

118. Pedo 11 need expansion?


a. Rapid 0.5 daily
b. rapid 0.5 weekly

119.

Implant on 34# pt feels pain which nerve?
● Mental nerve
● Middle superior alveolar nerve

120.

Implant on 37# pt feels pain which nerve?
● Inferior alveolar nerve
● Incisive nerve
121.

Pt missing 36 , 8mm MD interocclusal 7mm implant mearsument?
a. 5mm implant width

122. Restoration accumulates food + with loss floss ?


a. open margin
123.

Pt during class I u did perforation , bleeding stops what to do ?
a. DPC + GIC+ COMPOSITE
b. CaoH + RMGIC, Zoe + Amalgam

124. Child before 3 days pic of pulp exposure after trauma what to do?
125.

Pt after endo feels pain what is the analgesic to relieve the pain?
a. NSIAD
b. Acetaminophen

126.

Pt feels pain after you already give him Ibuprofen. What to do?
a. Ibuprofen combined with acetaminophen

127.

Ground glass appearance with?
a. Fibrous dysplasia

128. Pontic ridge lap complication?


a. Tissue irritation (due to difficulty of cleaning)
b. less esthetic

129. Doctor after giving infiltration, handed the syringe to the nurse and she capped it, during this action the nurse
injured what should have been done to prevent the injury?


a. Passing the syringe unsheasthled
b. one hand capping technique

130.

Sharp instrument container discard after?
a. ¾

131.

Pt have #37 tilited ?
a. Ring clasp

132. Pt have #37 tilted what to do?


a. Extended the occlusal rest more toward the centre
133. Fixed PFM porcelain overlapped the cervical margin this is called?
a. Metalless
b. collarless ✅

134. Intern long nails , etc etc ?


a. honor of the profession ✅

135. Senior consultant let her relative be treated instead of the waiting list she violated ?
a. disrespect the profession
b. taking advantage of her position to violate the regulations ✅
136.

After access to the molar you notice pulp stone what to do ?
a. referral to endodontist

137.

Third molar the root inside the inferior alveolar canal?
a. Referral OMFS
138.

#36 with 4 roots on PA, what should you do , patient in pain?
a. Pulpotomy and referral
b. continue the treatment

139.

End renal failure had simple occlusal caries, low salivary flow & complaints of bad breath. What is the cause?
a. Ammonia
b. caries
c. xerostomia

140. Pt long scenario bald tongue?


a. Serology
b. exfoliative cytology✅


141.

Impregnated Retraction cord for hypertentsice?
a. Aluminium chloride
b. Zinc chloride
c. Epinephrine
d. Nor epinephrine
142. Impregnated Retraction cord for hyperthyroidism?
a. Phenylephrine

143. Pt after chemotherapy 3 weeks , 250k platelets , 5500 WBC , 2.5 RBC require extraction?
a. Do it without precautions
b. give prophylaxis
Normal platelets count, normal WBCs , Low RBCs “ ANSWER DEPEND ON THE VALUES”
144. Most common attachment for mandibular overdenture with 2 implants?
a. Clasp
b. Bar
c. Ball
d. Locator

e.
145.
a. Assure ✅
Endo 2 post on molar without RCT , 25years , percusision , papaltion normal , PD = 2mm what to do?
Usually while there are no signs and symptoms of failure, no need for the interventition just
inform and assure the pt
146.

Water line bacteria maximum number?
a. 500
b. 1000
c. 5000
d. 7000

147. Pt with Prosthetic heart valve if we donnt give him prophylactic antibiotic .. how much in % he will develop to
bacteremia or infective endocarditis ?
a. 15-25 “ 10 and less than 10 also right”
b. 35-45

c. 55-65
d. 75-85

148.

Waterline bacteria in the dental unit wash between pt ?
a. 20-30 sec
149. Soap and water?
a. 40-60 sec ✅

150.
✅✅
Pt with trauma 20 years ago on #22, there's calcification on the coronal and apical part, prognosis ?
a. Poor ( RCT )
b. Fair ( Bleaching )
151. Pt came to the hospital looking for the emergency section , you’re a doctor in the facility and you ignore the
patient without directing him where to go, you violate what?
a. Autonomy
b. Beneficence ✅
c. nonmalfeneince
(Definitions below from Omar Notes)

152. Pt having desquamative gingivitis and buccal white striate ? (Mentioned before)
a. Erythema multiforme (EM)
b. Pemphigus vulgaris (PV)
c. Mucous membrane pemphigoid (MMP) ✅
According to multiple references; MMP can have white striae since it can cause scarring, another very similar
disease with the same symptoms (Desquamative gingivitis + buccal white striae) can be Lichen planus, but in this
case it’s not mentioned in the choices. If the 2 were together another differential criteria should be present to help
identify the correct diagnosis.

153. Child with ulcer on the palate appeared last year and resolved within a week, it came now with measles 2 days
ago diagnosis?
a. Recurrent oral herpes 🔁
b. Recurrent aphthous stomatitis
It could be A but there is no direct correlation between recurrent oral herpes and measles, but recurrent
herpes comes as herpes labialis so it can be wrong. The question must have had more details ( maybe
something like; ulcers only on soft palate and throat with no other ulcers noticed on other body parts) The
correct answer may not be written here as it also could be Herpangina. As stated more details needed.


154. Mother complaining that her child have missing teeth, that she also complains from the same issue (the mother
also had 2 missing teeth) confirmed by radiograph (no x ray provided)
a. maxillary lateral
b. mandibular 2nd premolar
c. maxillary 1st premolar

155. Polyacrylate better than zinc phosphate?


a. biocompatibility


156. Clindymicin and diarrhea which bacetica ?
a. C. difficile ✅
157. Pt u want to locate the posterior palatal seal area in his mouth what to use?
a. Indelible pen
b. T-burnisher ✅
158. Patient had TB 4 months ago and did the test three times and all results were negative He came for dental
treatment, how do you treat him?
a. Treat in isolated room
b. Last patient of the day
c. Use infiltration mask
d. Request blood count


159. 19 yr old patient with class II malocclusion, what’s the treatment?
a. Palatal expansion
b. Lefort (not mention which one only lefort) ✅


160. Definition of class I malocclusion?
a. Perfect aligned teeth
b. Malaligned teeth ✅


161. Well controlled diabetic patient took her insulin does before coming to the clinic, in clinic she had sudden

a. Insulin shock
b. Hyperglycemia

dizziness and talking aggressively, (no other diseases mentioned, only diabetes) what is the cause?

c. Hypertension
d. Hypotension


162.

Patient with INR 2.5 and came for extraction of a single tooth, do you treat or defer?
a) Treat


163. pedo patient 30kg and you want to prescribe ibuprofen, What’s the dose?
a. 15-20
b. 4-10 ✅

164. Doctor in Saudi did research on a drug and had bad results, he found out that there was research on the same
drug in Europe many years ago that had good results, he refused to publish his results because he was afraid of a bad
reputation.What did the doctor violate?


a. Ethics of confidentiality
b. Ethics of researcher


165.

How to check the effectiveness of IANB?
a. Canine
b. First molar
c. Second molar
d. Third Molar


166.

Which tooth has a consistent number of canals?
a. Maxillary Canine
If there is max incisor will be more correct


167. Patient takes cortisone,dose is 25, comes to the clinic for a single tooth extraction, How to manage?

b. Same dose on the day of procedure ✅


a. Double the dose on the day of procedure

c. Take drug a day before and a day after surgery (wasn’t sure about options)


168. Questions about stages and grades of periodontitis and what stage and grade is patient who is non smoker but has
diabetes 8%?


169. Syneresis and imbibition of alginate?


170. which impression can be poured after 1 day?
a. PVS


171. patient had pain third molar (pericoronitis) how to manage?
a. Third molar extraction

c. Give AB and followup


d. Operculectomy

b. Give AB and defer treatment


172. PerioEndo questions: if a patient had a very deep pocket, abscess or cyst, what do you do first?
a. Do curettage first then endo
b. Do endo then wait
c. Surgical intervention
Note: if there is option to do endo first then curettage will be the right choice
173. Questions about paget's disease:
a. Denture not fit or loose with time and cotton wool appearance in xray


174.

Child his family hurt his feelings, which type of abuse?
a. Emotional abuse
175. doctor cut her finger In Thursday, In Sunday she went for a dental procedure, she wear

a. Remove diamond ring


b. remove the wedding ring

wedding ring in the right hand and diamond ring in the left hand, what should she do before hand hygiene?

c. wipe the cut with alcohol


d. put a waterproof band on the cut


176.

177.
a. Consent from parents ✅
Under 18 yrs patient want diamond in her tooth, what to do

Patient stopped smoking for 3 weeks and started having big and multiple ulcers in his mouth?
a. Herpes
b. Erythema multiforme
c. Lichen planus
d. aphthous ulcers (I Add it ) ✅

178.

Patient with decreased VD and had angular cheilitis, How to manage?
a. Topical antifungal
b. Topical corticosteroids


179. HIV patient had white lesion with no pain, how to manage?
a. Systemic corticosteroids
b. Topical corticosteroids

d. Reassure patient ✅
c. Refer to his doctor because its a diseases progression

Note: insufficient information if he said on the lateral side of the tongue D will be the right choice
180.

181.
a. HBV ✅
Impression sent to the lap with blood on it, what virus could spread?

Sterilisation bag with Blue colour indicator, what do you do?


a. Use the instruments
b. Report and Send it back ✅


182.

Complication of aker clasp ?
a. Mobility
b. Recession


183. Patient cares about esthetics and came for anterior crowns, what's the most healthiest and esthetic finish line?
a. Supragingival
b. Subgingival ✅


184. Broken file in the middle of the canal, what is the prognosis?
a. Good
b. Poor
c. Fair ✅
● Question is missing some information but if they can remove it a good prognosis
185.

Lesion 4x3 cm, How to manage?
a. Marsupialization


186. Patient had a crown in the anterior tooth and the crown fell three times after cementation, the dentist informed

a. Do endo and post & core


b. Use different cement

the patient that it will fall again if he cements it. What's the best management?


187. Patient came and needed crown lengthening but has a short root trunk, what should you worry about?
a. Damage the bifurcation area
b. Make perforation in furcation area
c. Cause sensitivity


188.

procedure on tooth #16 patient had sudden pain. You did perforation, how to manage?
a. MTA

189.

Extracted wrong tooth, what should you do?
a. Inform the patient


190.

Best type of floss for Bridge?
A Superfloss
B Interdental brush


191.

Negative rake angle?‫مكرر‬
a. Kfile
b. Hfile


192.

What's the effect of steroid on diabetic pt :
a. increase blood sugar
b. Decrease blood sugar
c. no effect
d. Stabilize


193.

Human papillomavirus is associated with?
a. Oropharyngeal cancer
194.

patient come to clinic to restore #23 what is your choice for occlusion ?
a. Unilateral occlusion
b. Bilateral occlusion
c. Mutually occlusion
d. linear occlusion
195. .Patient with INR 2.9, ready for surgery and had prophylactic antibiotics with paracetamol, adverse effect is ?
a. Reduced INR
b. Incr

c. eased INR Angina
d. Heart failure


196. obliterated canal and short root?
a. Dentinogenesis imperfecta
b. Amelogenesis imperfecta

c. Dentin dysplasia type 1


197. Survival rate of hepatitis stay at room temperature for how long?
a. 4 hours.
b. one month.
c. 8 months

d. Few minute


198. patient came to the clinic concerned about broken instruments in her tooth that was previously treated 2 years
ago. Upon examination of tooth #26 there was no pain on percussion or palpation, Also PA shows no sign of
inflammation or any apical lesion (PA shows #26 with two broken instruments in the apical area in two roots with
good obturation and crown). What is management? (No follow up in options)
a. Apical surgery
b. root amputation

199.
c. extraction
d. try to bypass it reRCT again.✅
patient came with pain in the left upper area. Upon examination, the patient has generalized horizontal bone loss
and tooth #26 has pain upon percussion but did not respond to cold or EPT and it was sound. What is the diagnosis?
a. true combined
b. primary endo secondary perio
c. primary perio secondary endo
d. primary endo


200. Risk assessment, what would you do?
a. recognize the hazard, record the findings
b. recognize the hazard, evaluate the risk, inform the supervisor
c. recognize the hazard, evaluate the risk, record the findings, review the assessment periodically
d. recognize the hazard, evaluate the risk, record the findings, inform the supervisor


201.

Pt with unilateral crossbite due to functional shift of mandible
a. Bilateral maxillary expansion
b. Treatment of unilateral cross bite in affected side
c. Treatment of unilateral cross bite in non affected side


202.

203.
a. pause frequently ✅
Which of the following is the most appropriate when breaking bad news to a patient ?

you extract the upper molar tooth and the remaining bone to the sinus was 7 mm, what is the Treatment?
a. Immediate implant
b. External sinus lift
c. Internal sinus lift ✅


204. .Patient comes with missing maxillary anteriors, premolars and molars. Treatment is a combination of fixed and
removable.
a. Plan the rpd design first
b. Start fixed first

c. Start removable first
d. Start both at the same time
205.

Aluwax is softened in how many seconds?
a. 30 sec


206.

Needle stick injury during removal of waste bags to the worker which failure?
a. Active failure
b. Latent


207.

long case bite on tube make irritation and have appointment on orthodontist after 2 weeks
a. Cut wire
b. Remove wire
c. Bond the Buccal tube
d. Wait her appointment
208. Def of Autonomy?
a. Autonomy : right of competent adults to make informed decisions about their own
b. medical care
209.

What lesions appear after blunt trauma ?
a. Hematoma
b. hemangioma
c. Ulceration


210. Headache around eye when you give oxygen it goes away in 10 min?
a. Migraine
b. Cluster ✅


211.

Which of the following has a high recurrence rate?
a. Keratocyst


212. patient with increased lower facial height , abnormal increased ramus growth (I think they said backward

a. mouth breather ✅
downward direction) open bite with proclined incisors and supraerupted posteriors, that is the cause?

b. tongue thrust habit


c. corozon syndrome


213. How to diagnose the severity of asthma
a. Cheat x ray
b. clinical symptoms
c. O2 saturation

214. Only teeth show 8 configurations?
a. Upper 4
b. Upper 5 ( Max 2nd premolar )
c. Upper 6

d. Lower 6


215. Crown fracture involving enamel, dentin and without the pulp, classify?
a. Crown infarction
b. complicated crown fracture
c. uncomplicated crown fracture ✅


216. location of second canal in lower central ?(mentioned before)
a. Mesial
b. Distal
c. Labial
d. Lingual ✅
217.

Which tooth constant number of canal ?(Mentioned before)
a. Maxillary central incisor
b. Mandibular central incisor
c. Maxillary canine
d. Mandibular canine
218.

Negative rake angle in ?
a. Kfiles
b. H files


219. Probe with 1235789 ?(mentioned before)
a. Marquis
b. William
c. Who

220. Female patient present in the clinic complain from swelling on the upper lip, the patient never use filler, what is
the most likely the patient has?
a. Mucocele
b. Angioedema ✅
c. facial granulomatosis
d. allergic contact


221. Which of the following is the most diagnostically dental erosion location in helpful patients with bulimia
nervosa ?
a. Incisal edges
b. Occlusal surfaces
c. Labial surfaces of maxillary teeth
d. Palatal surfaces of maxillary teeth ✅

222. Pt class 2 Kennedy classification with deep mesiobuccal undercut(0.02) which clasp you will use ?
a. RPI
b. combination clasp ✅


223.

a 4 years old patient came to the clinic and while closing his mouth his mandible shift. What is management?
a. remove from canines
b. functional appliance


224. 5 child old have prognathic mandible, open bite, cross bite, maxillary deficiency what to treat first always start
with expansion
a. mandible prognathic
b. Maxilla deficiency
c. Cross bite

d. Open bite


225.

lesion with bluish colour/transparent what could be the content
a. mucin
b. blood
c. Fat


226.

patient with neck bruising and ulcers on the palate(mentioned before)
a. sexual abuse
b. neglect
c. physical abuse


227. While working on calcified canal the canal suddenly disappeared, what caused the disappearance in the canal?
a. Calcification
b. bifurcation ✅

228. case about patient her parents complain about firm swelling in her lips 44 stands for 6 month with history of
multiple trauma ,tx:
a. marsupialization
b. Excisional biopsy

229.
c. FNA
● Note:I think it’s Incisional biopsy because the swelling diameter ✅
a patient referred to you to extract third molar while examination you found out that there is resorption in lower 7

b. Tell the patient only


c. Go with extraction

a. Refer patient until you tell his doctor

230. a case about patient complains abouts crack sound “no other details “ :
a. Mandible condylar fracture
b. Disc Dislocation with reduction
● Missing info

231.

long scenario about heart failure, the first thing you do :
a. INR
b. Pt
c. Ptt


232. case about TB ,patinet has history 4 years ago , with 3 negative tests now , you consult his physician & he gave
you the green light :
a. Make sure it is the final patient

c. Mask ✅
b. isolated room

d. Ask for blood test.


● Note: Treat him as normal pt
233. Minimum age for dental implant?(Mentioned before)
a. 21
b. 18 ✅
234.

What instrument is used to check enough flaring is done:
a. spreader
b. File
c. MAC ( Master apical cone )


235. Case about pt with emphysema during treatment he started having difficulty breathing and coughing blood
pressure is 178/121, o2 is 93% And the doctor stopped the treatment. What is the reason for postponing the
treatment?
a. o2 is 93
b. Blood pressure
c. Coughing

d. Shortness of breath at rest

236. Diabetic pt with gangrene in the palate the causative agent?
a. viral
b. Fungus
c. Bacteria ✅

237. Immunocompromised pt has a lesion in the ventral surface of the tongue that is very painful and pt can’t eat
because of the pain pt said that before the transplant he had recurrent aphthous ulcer and herpese libialis (Lab result
show normal neutrophils and low WBC)
a. atypical herpes
b. Neutropenic ulcer
c. Recurrent aphthous stomatitis
d. Traumatic ulcer


238. Female patient came complaining from parotid (i forgot if its symp or asymp) swelling that increases during
mealtime, x rays didn’t reveal any calculi, what is the possible dx?
a. Mumps


b. sarcoidosis
c. sialadenosis
d. Mucus Plug obstruction


239. 8 years old came complaining from vesicular ulcers in the palate with fever and malaise 2 days ago, he had the


same lesions 2 years ago and was healed within a week. What’s the dx?
a. HSV
b. recurrent oral herpes
c. Erythema multiforme


240. According to the American society of anthropologists (ASA) , a patient with a history of MI 1 year ago
considered?
a. I


b. ll
c. lll
d. IV

241. Which muscle is responsible for moving the mandible downward?
a. Digastric
b. Buccinator


c. Temporalis
d. Lateral pterygoid


242. a 3 years old patient presented with asymptomatic, multiple white nodules on the palatal raphe. What's the dx?
a. Epstein pearls
b. bohn's pearls


243. Primary roots compared to permanent roots?
a. long and divergent
b. short and divergent
c. long and convergent
d. short and convergent


244. Internal resorption scenario in mid root ( it was discovered accidentally by the dentist, the tooth is asymptomatic)


what is the management?
a. RCT
b. Follow up after 6 months
c. Occlusal reduction


245. Mid root Circular radiolucency(xray), the tooth is ve to cold test and is asymptomatic. What's the next step?
a. perform RCT
b. Take shifted xray or CBCT (in the same option)
c. Investigate pulp status more using EPT.

246. Poorest prognosis ?


a. Avulsion >60 min closed apex
b. Avulsion >60 min open apex
c. Avulsion <60 min closed apex
d. Avulsion <60 min open apex


247. A doctor cut her finger on Thursday, on Sunday she went to the clinic to see her patients and was wearing a


diamond ring and wedding ring. What should she do before washing her hands?‫مكرر‬
a. remove diamond ring
b. remove wedding ring
c. put waterproof plaster
d. wipe her finger with alcohol


248. In try in and before instrumentation, there was high occlusal contact, what instrument used to measure crown


thickness?
a. Iwanson calliper


249. Patients came with increased palatal vault, periodontitis, Xerostomia and crossbite. What does he have?
a. Asthma
b. Leukaemia
c. Crohn's disease


250. What is considered a parafunctional habit?
a. Erosion


b. Chewing
c. Clenching
d. Abfraction


251. Branch of ethics that is related to identification, analysis and resolution of moral issues that arise in healthcare


for individual patients?
a. Clinical ethics


252. What are the 3 essentials in dental ethics?
a. Compassion, competence, autonomy
b. autonomy, profility, competence


253. What instrument is used to check root surface smoothness after scaling?
a. hoe
b. chisel


c. mirror
d. Explorer


254. Oral ulcer, genital lesions and +ve VDRL what to give?
a. Penicillin

255. Scenario about a lady who came in with edematous swollen gingiva and she started OH measures 3 days ago,
there was a histopath description suggesting plasma cell gingivitis (please read about it) what tx?
a. gingivectomy


b. plaque control and removal of the causative factor.
c. Allergen identification and removal.


256. Immediate allergic response scenario: what cells are founded?
a. IgE
b. IgM
c. Lymphocytes


257. Healthy gingival tissue what is found?
a. Mild Lymphocytes
b. Moderate lymphocytes (chronic infection)
c. Sever lymphocyte (acute infection)


258. Patient came with a deep carious lesion and Apical RD (no xray) and mild Hemophilia A. What to do? (They


didn’t mention anything about tooth restorability)
a. Perform RCT
b. Factor IX replacement
c. Extraction with acrylic splint

259. Female patient came complaining from rough teeth surfaces, upon IO, she has staple occlusions with mild
attrition and linea alba, pt is bruxiser, what is the immediate management?
a. Reassure the patient


b. Educate the patient and construct night guard
c. smooth the teeth roughness


260. Patient came to construct a CD, the patient has a severe cough, history of blood in sputum, night sweats and
weight loss, upon examination the patient has ulcer with everted margins in the posterior of the tongue. What's your


management?
a. Take Chest xray and consult (or refer) to a physician.
b. Take biopsy and proceed with taking impressions
c. Take the impressions anyway (without biopsy)


261. Child that needs antibiotics prophylaxis, what is the regimen as for (mg/kg)
a. 50 mg amoxicillin


262. Which condition could cause prolonged bleeding and the patient might come back again complaining from
unstoppable bleeding?


a. hereditary thrombocytopenia
b. Von willebrand type 3
c. hereditary hemorrhagic telangiectasia


263. 3.5 to 4.5 implants considered as what?


a. wide
b. regular
c. narrow


264. Patient had a provisional crown for 2 weeks and you wanted to make a final impression but you couldn’t control
the bleeding, what to do?


a. RC+ferric sulphate with polyether impression
b. Adjust (or remake) the provisional and take it after 2 weeks
c. RC + Epinephrine and PVS impression


265. Where to put the keyway in class l RPD ?
a. abutment


266. Why remove the smear layer before obturation ?
a. To allow the sealer to go into open dentinal tubules
b. To remove the sealer easily
c. To increase the antimicrobial activity of the sealer


267. a patient who is healthy and has a healthy periodontium. He had anterior crowns placed a while ago and now


presents with spacing and other issues indicating trauma from occlusion. What type of occlusal trauma does he have
a. Primary
b. Secondary
c. Combined
d. Persistent


268. patient who is taking phenytoin and has gingival growth. What needs to be done
a. changing the drug


b. gingivectomy
c. consult the physician and do plaque control or scaling


269. NUG and he has a high fever. What should be done on the first visit?


a. oral hygiene and mouthwash
b. Supracrestal scaling and antibiotics


270. 1.5 radiolucency semi circular cross and above the roots of upper premolars and they were healthy?
a. Apical scar
b. Apical cyst


c. Apical infection
d. Normal anatomical landmark
271. hiv and well controlled with medication and the viral load is mostly undetectable (as per the physician) and you
got a needle stick what should be done?
a. Follow up


b. Send patient for testing viral load
c. Go to the doctor immediately and start PEP or antiviral


272. patient taking zoledronic acid (zometa) and is for extraction which of the following is the appropriate
management?


a. Antibiotic prophylaxis
b. give antibiotics during and postoperative
c. Give antibiotics if only there’s signs of infection


273.

about actinic cheilitis management:
a. laser ablation
b. Excision
c. Corticosteroids ointment


274. Principle of tooth preparation for full ceramic margin supragingival 0.5mm interocclusal space 1.5mm Length
of axial wall 2mm What does the dentist violate?
a. marginal integrity
b. retention and support ✅
c. Preservation of tooth structure
d. Preservation of periodontium


275.

about the most difficult tooth access during the treatment of furcation involvement?
a. 1st max molar
b. 2nd max molar
c. 1st mand molar
d. 2 mand molar


276. Patient on a regular cortisone need extraction and major surgery at the same session.


a. You should double the dose.
b. Keep the same dose.
c. Refer to specialist
d. ask for adrenaline tests.


277. Rabid expansion and Quad helix Appliance should be activated
a. once daily
b. once weekly
c. twice weekly
d. once every month


278. asthmatic pt and has an allergy to benzodiazepines What is the management
a. ibuprofen
b. adrenaline analysis to find alternative


c. Adrenaline
● The choices provided are all wrong. This is common question and the answer is: Nitrous Oxide


279. a case need RBD .The tooth has cervical undercut. Which clasp you will use?
a. Ring,


b. Aker
c. RPI
280. Liver cirrhosis Patient Which is unsafe for him?


a. Paracetamol
b. Lidocaine
c. adrenaline
d. Cortisone


281. Picture of a red line at the marginal gingiva management
a. scaling
b. gingivoplasty
c. Gingivectomy


282. Patients feel dizzy after multiple injections. The management
a. oxygen
b. maintain the airway.


c. Epinephrine,
d. Supine position


283. Pedo patient swallowed the clamp .How to prevent
a. use dental floss


284. scarlet fever symptoms.


285. Hemophilic patient with lab tests ,normal readings of CBC need extraction. What is the management?


286. Flabby ridge management?
a. Mucostatic impression


287. The definition of confidentiality


288. Why is the composite better than a GIC?


a. Less micro leakage
b. less marginal leakage.


289. Xray with a fractured mandible.
a. Simple,
b. compound


290. The material used in pouring the cast?


291. doctor should have
a. Professionalism, laws, Ethics
b. morals,professionalism,ethics
c. Morals, ethics and professionalism.


292. Pedo pt needs pulptomy for upper primary central incisor. Which nerve will you anaesthetise?
a. Anterior Superior Alveolar nerve.


293. A picture of a short clinical crown with gingivitis The patient doesn't like his appearance. The Management?
a. Crown Lengthening
b. Gingivectomy
c. Gingivoplasty+Lengthening,
d. Scaling and Follow up.


294. Pic of instrument and pointing on the shank ?


295. Pic of girl with Treacher Collins Syndrome


296. Pic of recession #43 by 1 mm an #44 by 2 mm and asking of the class of the recession
a. Class 1


● The first picture on the left is for class I.


297. modified ridge pontic
a. Esthetic and no surgery required


298. bacteria most associated with gingivitis:


299. which lubricant remove smear Leyer
a. EDTA


300. R flx cross section
a. Triangle


301. Resorbable sealer
a. ZOE(if they ask about canal filling )
● Caoh this type of sealer resop as it has greatest solubility


302. Sri lak rapid
a. 8% ✅


303. Avulsion for 30 minand the tooth kept in mik what’s the type of splint
a. Functional 2 weeks splint


304. Dry heat sterilizations 320 C
a. 120 min ( if it Fahrenheit)


305. Mylohyoid ridge reduction which nerve will be injured
a. Lingual nerve.


306. Tooth formation in embryo
a. 6 weeks


307. Muscle responsible for protrusive movement of the jaw


308. scenario … petechia in hard palate . Child abuse
a. sexual abuse

309. leeway space in mandible per side
a. 1.7mm
● Reference (Contemporary orthodontist)


● Reference (mecdonala for the child and adolescent)

310. pic of pt with fixed orthodontic appliance with diastema not fully closed and low frenum . asking of the nexst
step of the treatment.


311. Class III molar relationship and asking about camouflage extraction .


312. The most fractured root while extraction
a. Upper 4


313. Scenario of sudden allergy after rubber dam
a. type 1 allergy


314. What is the grade of fremitus palpable movement not visible?
a. Grade 2 ✅


315. Main advantage of screw retained implant over cemented one.
a. Retrievability


316. Minimum interocclusal space for screwed implant
a. 4 mm


317. Enamel. Dentin fracture whats the name of this fracture.
a. Uncomplicated fracture.


318. Most common mandibular fracture
a. Condyle then angle .
319. Pt taking prednisone and need simple extraction whats the proper management
a. 100mg of corticosteroid 1 hr pre operative


b. 200 mg of corticosteroid 1 hr pre operative
c. same dose 2 hr pre operative


320. Scenario of asthmatic pt need to extract 3rd molar what is the Safest analgesics


a. Ibuprofen
b. Paracetamol
c. Naproxen


321. caseating granuloma in the lung
a. Tuberculosis
b. Sarcoidosis

322. Pedo pt did SSC and came after 2 days complaining of pain and bleeding . 2 mm ssc high from occlusion . PA
xray was taken and reviled overhangs SSC .Whats the cause of overhangs:


a. Iinadicuate occlusal reduction
b. Improper SSC size


323. X ray for facial asymmetry(mentioned before)
a. Cephalo
b. Ct
c. Anterior posterior


324. assistant spoke with her friend about the patient Violate
a. patient privacy


325. Biopsy when send to lab
a. put bihazred libelled bag


326. If remove orth and have white spot what to do
a. varnish gel


b. florid
c. Reinforce oral hygineic


327. Frequently extracted tooth


a. primary first molar
b. First molar permenent
328. If there is blood on gloves and hand what should the doctor do
a. alcohol


b. desinfect
c. wash hand with soap 60 second


329. Modified pen grasp pic


330. Pedo with multiple caries what type of abuse
a. Neglected


331. a doctor speaks to a patient and tells them it's good that they didn't go to the other doctor what he did violate


332. The access of upper 4 and 5
a. oval


333. Aesthetic pontiac
a. ovate


334. Bacteria of gingivitis


a. gram+
b. gram -
c. A.A


335. Pt has removable denture for teeth 31, 35, 41, 44 and then he are lost 32, how to replace it

a. implant
b. RPD


336. Avulsion of 4 hours to a patient of 14 years ?
a. R.c t immediately
b. Rct after one week
c. No need for R.C.T


337. From where to do incision for an abscess


338. Width of lateral 4.4 what is the width of canine according to golden proportions


339. bur cut porcelain
a. diamond


340. intrusion splint


341. tracing by getta pecha in size


a. 20
b. 25


342. position asthmatic attack in clinic


343. position for cardiac pt


344. Increase retention in fissure sealant
a. acid etch


345. fraction in midline called
a. symphysis


346. Extruded gp with no signs or symptoms,no pain on percussion
a. Follow up
b. Rct
c. Surgical RCT


347.

patient with asthma history 3 months ago & have burning sensation ,tx:
a. Rhomboid glossitis & antifungal
b. Geographic tongue & reassure


348. what is the most component of dentin in weight:
a. Water
b. In organic ✅


349. 75 years old came to restore broken MO restoration, probing depth was normal, cold test no response, EPT

a. normal pulp and apical tissue


b. necrotic pulp

positive response, precaution and palpitation was normal what is the diagnosis

c. asymptomatic irreversible pulpitis


d. symptomatic irreversible pulpitis


350. etching of porcelain in case of porcelain chipping :
a. Hydrofluoric acid 3-5%
b. Hydrofluoric acid 5-10% ✅

351. the patient complained about his denture ,why ? “ picture “ ‫حسب الصورة‬
a. Midline shift
b. increase vertical dimension
352.

deferential radiopacity between fiber post and flowable composite in xray?
a. Less
b. More
c. Same
d. Not different


353. Which is the most common tumour with diabetic ?
a. Lipoma
354. What is the most common tumour with denture ?
a. Liomyma


355. Patient came with implan failed with bone fragment what is the treatment?
a. Implant with GBR


356. Ethical dilemma: who determined it ?
a. Research


357. What is the problem face dentist when he take impression PVS on inflame gingiva?
a. Retraction cord engagement
b. accelerate setting

358. RPI fracture cusp how can manage?


a. Re casting
359. Undercut midbuccal 0.02 sever under cut ?
a. Combination W.W


360. Frenum interfere with denture ?
a. Vestuplasty


361. Pic for ridge mapping ? ( Ridge mapping = determine the hight and width of ridge)

🔁
362. Bridge to replace #24,#25 ?
a. FPD #26 ,#23 four unite

363. Combination syndrome complications ?
a. Bone resorption to the upper


364. Patient want anterior crown but you suggested for him treatment less aggressive. What do you have to do ?
a. Refer to take Second opinion


365. White lesion present from birth but now change on colour from white to brown ?
a. Enamel hypoplasia
366. 2 implants in anterior lower and ask about the adjacent natural tooth,It was mobile (grade3) and very short root

🔁
with no pain with endo tests What is the treatment?
a. Endo immediately and adjunctive Perio
b. Endo and follow up
c. Perio and follow up
d. Perio then endo
367. During a surgical extraction on a patient with hepatitis B the suture needle is covered by a bloody swab and both
are discarded into a waste sack. The needle punctures the plastic and the cleaner receives a sharps injury when she


empties the pedal bin: (mentioned before)‫مكرر‬
a. Active failure
368. Scenario about dentist did very good explanation to the patient before and after the treatment, what he did ?
Answers were definitions of his act


369. Brown pigmentation in the buccal increases in size in short period and they provide what they found in the
histology


a. Melanoma
b. Melanoacanthoma


370. Molar tooth with deep restoration and with one canal obliterated, what is it?
a. Calcification

371. What is the consequence of direct pulp capping in primary tooth ?


a. Pulp obliteration
b. Internal resorption
c. External resorption


372. Ortho patient with pain and takes type of medication I can’t remember it, what type of medication I can give it to


relieve the pain?
a. Paracetamol
b. Ibuprofen

Note: Paracetamol=Acetaminophen.
373. Ortho patient came with elongated wire with multiple irritations to buccal, tongue, lips What you will do?


a. Wax
b. Cut the wire


374. Patient came with ulcer it was tingling before , what you should do to prevent
a. Wear gloves
375. Where to stop the obturation ?


a. Major apical construction
b. Minor apical construction

376. Patient came complaining of pain with changing temperature of beverage and he has recent gold onlay in
premolar, and something affect the tooth with the adjacent canine, clinical photo and xray were provided, the
restoration is good no overhang or excess cement and there’s nothing happened to canine and premolar, what is the
reason?
a. Not polished restoration
b. Allergy from restoration
c. Irritation from cement used
377. Short anterior with brown fluorosis pigmentation, the treatment is veneers what type of preparation?
a. Window
b. feather


c. Buttjoint
d. Lapped


378. Deep class II restoration, xray provided good margin and adaptation but with bone loss in the area of restoration


what is affected?
a. Periodontium
b. Pulp
c. Enamel
d. Dentin


379. Pedo fall in his chin, what probable fracture?
a. Condyle
b. Angel
c. Symphysis


380. Where to deposit in gow gate nerve block technique?
a. Condyle
b. Condylar process


381. Pic of patient with large swelling in left area under the jaw the mandible is not palpable, what is the area affected


or in which area
a. Digastric tendon
b. Platysma


382. Perio swelling, clinical pic was provided, name?
a. Gingival abscess
b. Periodontal abscess
c. Periapical abscess


383. What type of classification in primary ends with class III


a. Distal step
b. Mesial step
c. Flush

384. What is most common malocclusion pictures were provided


a. Class one allignd teeth
b. Class one crowding
c. Class 2
d. Class 3


385. What type of restoration for posterior have esthetic, strength, durable:
a. High filler composite
b. Low filler composite
c. Amalgam
d. RMGI


386. 132 c autoclave what is the minimum minutes ?
a. 2


b. 3
c. 4


387. Heavy smoker


a. At least 10 cigarettes per day
b. At least 20 cigarettes per day
c. At least 30 cigarettes per day


388. Implant size 4 and the dimensions available were provided, Vertical 10 Mesiodistal 7 Crest of bone to the upper
7 Width 4 what is compromised ?
a. Width
389. Referral from prosthodontist for crown lengthening , when he did the flap the margin was 1mm from the bone
What should he do?


a. Close it and suture
b. Remove 2mm more


390. Most difficult tooth during perio
a. 1st maxillary premolar
b. Maxillary distobuccal root of 1st molar
c. Lower molar


391. Most difficult to access during Perio surgery I think
a. Maxillary 1st premolar
b. Mandibular molar
c. Maxillary 1st molar
392. For implant placement, where is the easiest area free from vessels or nerve or less anatomical something


a. Upper posterior
b. Lower anterior
c. Upper anterior
d. Lower posterior
393. What is the most common complication of herpes zoster:


a. Ear hearing loss
b. Neuralgia


394. Necrotic tooth without swelling:
a. start endo
b. Follow up
395. Trauma with pulp exposure 1 day


a. Direct pulp caping
b. Pulpotomy
c. Partial pulpotomy
d. RCT
396. Patient with history of breast cancer and takes bisphosphonate need multiple extraction 2 in the upper left and
one in lower left and her oncology said that we can give her prophylaxis each visit
a. Extract all in one visit


b. Extract one each visit with 2 weeks intervals
c. Extract one each visit with 2 months intervals
d. Don’t extract till became symptomatic
397. Intrusion of primary centrals what is the effect on the permanent successor?
a. Devitalization

b. hypoplasia
c. Palatal orientation of crown


398. Drugs cause xerostomia?
a. Antihistamine

399. Patient has and there’s a horizontal root fracture between middle and apical thirds and fracture line has
radiopacity, there’s no response to the cold test, what’s the treatment?


a. rct for all root
b. rct for coronal part only
c. extraction
d. Pulpotomy
400. Bilateral firm slowly growing swelling of parotid gland, no lymphnodes involvement
a. Oncocytoma
b. Oncocytosis


c. Warthin tumor
d. Pleomprphic adenoma


401. How to check crown & root fracture?
a. Tooth slooth
b. Seating jig

402. Best practices for caries risk assessment is during active periodontal therapy and prior to maintenance. What is
the reason to do caries risk assessment?
a. Loss of coronal tooth


b. Increase pocket depth.
c. Exposure of root surface
403. 25 yrs old patient has retained 53 and the 31 is impacted with poor prognosis. The 53 has favorable crown size
and full root length? What is the appropriate management?
a. Extraction of both canines & implant


b. extraction and space closure
c. No treatment NOTE since the primary has good prognosis so we should keep it


404. ANB in class II
a. Increase
405. Chipped porcelain at incisal angle. Type of fracture?

a. Static
b. Cohesive
c. Adhesive


406. what happen if retentive arm above the height of contour?
a. Tipping


407. The denture wearer came to you after delivering dentures complaining of numbness in the mental area and


asking about the material used in the Intaglio surface to see the contact?
a. Chloroform and rouge
b. Titanium oxide
c. Methylene blue
d. aerosol starch spray


408. Pic of implant analogue


409. Responsible for the emergence profile?
a. Healing abutment


410. Space infection
a. infratemporal
b. Submandibular
c. Retropharyngeal
d. Pterygomandibular

411. Pregnant women need antibiotics
a. Fluconazle


b. Tetracycline
c. Clindamycin
d. Clarithromycin


412. Why do we use water spray during cavity preparation ?
a. To decrease dentine heating
b. To prevent inspiration of particles
c. To decrease dentinal fluids movement
d. To clean the field


413. A 74 year old female patient comes to your surgery with a complaint of unable to wear her dentures. You have
seen the patient less than 24 hours ago, when you inserted an immediate denture. After insertion of the immediate
complete denture, the patient reported no significant issues during the day. The patient removed the denture at night,
cleaned and placed it in an appropriate solution. The next day, she hasn't been able to wear it and came to you. Why
did this happen?
a. Lack of frenum relief
b. Lack of insertion skills


c. Something in occlusion
d. Swelling and inflammation after alveoplasty


414. Sign of lidocaine toxicity
a. drowsiness
b. Cardiac arrest
c. Allergic reaction
d. Respiratory depression


415. Best temporary filling which provides sealing ability after RCT ?
a. Fermit
b. IRM


c. Cavit
d. glass ionomer


416. 14 yrs old class I malocclusion with Single tooth crossbite
a. Removable appliance with finger spring ✅
b. Fixed orthodontic appliance


417. How to check root smoothness after periodontal debridement ?
a. Explorer
b. Probe


418. 85 y uncontrolled diabetes came for annual routine visit exam revealed she has 12 probing depth and localized


bleeding on 31 32 41 radiographs revealed she has 50% bone loss what to do?
a. SRP and OHI every 2 month
b. SRP and OHI every 3 weeks
c. SRP and OHI every 6 months
d. SRP and prescrip ab to prevent bone destruction.


419. One of those is most likely to be abused
a. ‫‏‬child with low socioeconomic status
b. ‫‏‬child with high socioeconomic status
420. Pt medically fit or mentally stable what is the thing that may determine the type of floss to be used

✅️
a. ‫‏‬manual dexterity
b. ‫‏‬pt preference
421. Pa of #25 that has a relatively short root and Periapical radiolucency with around 2 mm above the bone” they
mentioned that the tooth is equigingival clinically and asked what is the best way to isolate the tooth
a. ‫‏‬no need
b. ‫‏‬cotton roll


c. ‫‏‬extract and implant
d. ‫‏‬Clamp the #26 and extend the dam to #24 (‫)هي اإلجابة األقرب حسب االختيارات‬

422. Picture showing 3 teeth with exposed bone with bleeding and bony defect they mentioned that the pt has heavy
plaque and Bop and necrosis of soft tissue
a. leukemic pt or something (this the most likely answer)


423. Pedo with deep carious lesion with intermittent pain provoked by sweets and lasts for seconds what is the
diagnosis
a. ‫‏‬pulp polyp


b. ‫‏‬pulp necrosis
c. ‫‏‬reversible pulpitis
d. ‫‏‬irreversible pulpitis


424. How much is the width of a shoulder margin finish line


a. 0.5-0.7
b. 1-1.5
c. 1.6-2


425. Active TB came to the ER with sever pain, he is coughing blood with sputum what is the management
a. ‫‏‬use filtration masks
b. ‫‏‬defer the tx for 1 month
c. ‫‏‬treat in an outpatient setting


426. Tooth with deep caries normal pulp and “sensitive to percussion” not tender, upon caries removal pulp exposure


occured what is the diagnosis
a. ‫‏‬AIP/NAT
b. ‫‏‬AIP/SAP

427. (LONG CASE) Pt with Grade I furcation related to lower molar, what is the TX, all probing depth were normal
rage?

✅️
a. GTR
b. Odentoplasty
c. Root coverage
d. forgot but something related to the root


428. 25 yo male has diabetes type 1 he is well educated and well controlled came for an extraction of an
asymptomatic tooth
a. ‫‏‬Hba1c must be taken


b. ‫‏‬prophy Ab before and continue after
c. ‫‏‬extraction in a morning appointment after taking breakfast and insulin shot


429. Pt wants to extract a lateral insicor what is the temporary FPD pontic design
a. ‫‏‬ovate
b. ‫‏‬conical/bullet
c. ‫‏‬Modefied ridge lap


430. Old pt came for extract with thickning around the apex of his teeth what is the cause
a. ‫‏‬deposition of cementum throughout the life
b. ‫‏‬physiologic remodeling of the cementum
431. Mesial resorption of bone and distal deposits what is the cause


a. ‫‏‬ortho extrusion
b. ‫‏‬physiologic mesial migration of a tooth
c. ‫‏‬winding of the distema
432. Which pf the following is an disadvantage of zirconia core :


a. Fracture
b. ‫‏‬marginal discrepancy
c. ‫‏‬weak bonding with porcelain

433. Patient said i feel discomfort in my lower bridge 44-46 , 44 is previously treated with Tear drop RL, what is the
dx of #44 ( No xray , no PB , No other information, did not mention if it has post or not )


a. failed endo
b. ‫‏‬VRF

434. Hemodent contains from :
a. alimunim chloride
435. pic of apex locator and asking about previous treated by another doctor once you put inside the mouth locator has


the sound what is happening?
a. Coronal perforation


436. HPV which cancer ?
a. Oropharngral


437. Saliva ejector inside the mouth ?
a. Mouth is less pressure than ejector


438. Pain in right side of tonsil ?
a. Glossopharngeal neuralgia


439. Prickle cell found in ?
a. Pemphigus


440. Doctor wants to exo 14 by mistake he extracts the 25 what he did ?
a. near miss medical error


441. hand hygiene time: 4060sex( Repeated )
a. What doctor should do first thing in PPE before seeing patient: Hand hygiene


442. bullous pemphigus test :
a. immunofluorescence


443. lateral peg shaped and ask for stage cause it
a. bell stage
b. cap stag
c. bus stage


444. trunk rash and fever and tongue and white tonsil, red and bumpy tongue and ask for diagnosis
a. scarlet fever ( strawberry tongue )


445. calculate D9 of file 30 taper 0.04
a) 0.66 ✅
● File is 30 so ( 0.30)
● Taper is 0.04 9 = 0.36
● 0.36+0.30 = 0.66


446. cervical tooth fracture splint
a. 4 month


447. sound tooth have pain on percussion and no response in cold test ask for diagnosis


a. endo primary with perio 2nd
b. perio primary with endo 2nd
c. true combined
d. periodontal combined


448. case endo perio with deep pocket and ask what management
a. start endo then perio
b. start endo and followup

● start perio
449. amalgam restoration which opposite to gold and cause pain, ask for management
a. varnish in amalgam


b. Extraction
c. change restoration


450. tooth with 2 file broken and beyond apex ask for prognosis
a. poor
451. tooth had RCT and fail and reRCT and picture of apical abscess and ask for prognosis

452. patient had leukaemia and give you lab results ( he have platelets below normal but above 70,000 and wbc 4.3
below normal normal 4.5 ) and ask you what to do before start treatment
a. defer appointment

b. ask for platelets lab investigation in day of treatment,
c. give antibiotics prophylaxis
d. start treatment without anything


453. diabetic patient had his insulin in morning and came and when the doctor start she feel dizziness‫مكرر‬
a. insulin shock
b. Hyperglycemia
c. hyperventilation


454. tooth did survey crown and after that you see that it didn’t required management
a. full coverage crown


b. Restoration
c. contour
455. medical compromised patient came for extraction of 21 under GA and Doctors extract 11 tooth but he realised
and bring it back and extract 21


a. write incident report
b. tell the patient
c. nothing to do


456. patient came for extraction of lower 3rd molar and the tooth proximal to IAN, what you should do
a. Refer to OMF
b. explain to the patient and give him choice of refer to OMF
c. extract the tooth
457. patient have pseudo class III with mandible shift to anterior


a. palate expansion
b. smoothing with mod of cusp with bu
c. face mask


458. you take wray and not clear so you take with mesial shift which is mesial
a. mesiolingual ( SLOB )
b. .mesiobuccal
c. distal
459. you want to work on buccal of quadrant 1 and palate of qudrant 2 , position
a. 11-12 clock
b. 8 clock,


c. 6 clock
d. 9 clocK


460. Distal deep pocket to #47 Which incision you will make?


a. intrasulcular
b. distal wedge


461. Tooth with hx of trauma 20 years ago, remaining root with 1/3 of crown is left, prognosis? ‫مكرر‬
a. Fair
b. Good
c. Poor
462. Motion of ultrasonic tip ( depending on the type)
a. Circle
463. Patient cares about the esthetics and came for anterior crowns, what's the healthiest and most esthetic finish
line?’‫مكرر‬
a. Supragingival
b. Subgingival


c. Supra or suprabony
d. equgingival
464. Patient stopped smoking for 3 weeks and started having big and multiple ulcers in his mouth?
a. Herpes
b. Erythema multiforme
c. Lichen planus
Note : if there is aphthous ulcer will be the right choice

465. PerioEndo questions: if a patient had a very deep pocket, abscess or cyst what do you do first?
a. Do curettage first then endo.


b. Do endo then wait
c. Surgical intervention
466. Patient came and needed crown lengthening, but he has short roots, what should you worry about?
a. Damage the bifurcation area.
b. Make perforation in the furcation area.
c. Cause sensitivity
Note: if there is exposure of furcation will be the right choice
467. Pt having desquamative gingivitis and buccal white striate?
a. Erythema multiforme


b. pemphigus vulgaris
c. mucous membrane pemphigoid
Note: if there is a lichen planus will be more correct depending on other details


468. long scenario about a patient diagnosed with heart failure, the first thing you do?
a. INR
b. PT
c. PTT


469. Patient low respiratory rate and heart rate is 50, what should be the seating position?
a. Upright
b. Supine
c. Semisupine


470. Bacteria associated with periodontitis?
a. T. forsythia
b. A.A


471. Use of prophy jets?
a. For implant


472. Patient present to clinic with pain on biting on #45 and #44 Upon examination you found large amalgam
restoration in relation to two teeth, what is the best diagnostic to do?
a. Transillumination


b. PA
c. Percussion
d. palpation
473. Patient came to you while measuring probing depth, 6 mm from CEJ to the base. What did 6 mm express?
a. 6 mm attachment loss


b. 6 mm pseudo
c. 2 pocket depth , 4 mm attachment loss
d. 4 pocket depth, 2 mm attachment loss


474. Which of the following GIC types is used with cement?
a. Type 1


475. Nurse wiped vomit with a towel, the towel is considered as?
a. infectious
b. Hazardous


476. Doctor after giving infiltration handled the syringe to the nurse and she capped it. During this action the nurse
injured what should have been done to prevent the injury?


a. Passing the syringe unsheathed
b. one hand capping technique


477. Image of a case where #24,26,27 is missing and #25 is pier abutment, tooth has MOD amalgam and recession
with grade I mobility, what's your management?


a. Extract #25
b. Survey crown
c. M & D rest seat
d. Mesial rest seat

478. What is PCA composition


479. doctor passing by patient without helping him, violation of
a. justice
b. beneficence


480. a Dentist cooperates to work with implant companies to use their implant, what the doctor has violated ?
a. Conflict of interest
b. Autonomy
c. non maleficence
d. Deontology


481. Pic of rest seat in cingulum area and asks what should be done


a. Increase it buccolingual
b. Increase mesiodistal
c. Prepare the area of the rest seat


482. Pt came with conservative MO access throw amalgam restoration , what to do after endo.
a. ‫‏‬Amalgam core and crown
b. composite core and crown
c. ‫‏‬fiber post and ceramic crown
d. ‫‏‬cast post and PfM
483. CD patient he is also a smoker compline of mouth under the denture is red, you can see traces of denture flanges
on the patient’s mouth what is the management?


a. remove the denture temporary
b. tissue conditioner and antifungal


484. Patient with rheumatoid arthritis and DM takes metformin, methotrexate and corticosteroids have oral lesions.


Which of these causes the lesion? ‫حسب السؤال‬
a. Corticosteroids thrush
b. Methotrexate ulcer
c. Fungal infection


485. When you do preparation for onlay, and you want to do beveling, what instrument do you use?
a. chisel


b. gingival margin trimmer
c. diamond instrument


486. thick cortical bone?
a. buccal surface of max molar tooth


b. labial surface of max anterior tooth
c. buccal surface of man molar tooth
d. labial surface of man anterior tooth


487. Long case scenario about Fibrous gingival inflammation and bone loss (it is referred to chronic periodontitis)
what bacteria causes?
a. Streptococcus mitis
b. Aggregatibacter actinomycetemcomitans


c. Treponema denticola
d. Bacteroides fragilis


488. Preparation of root canal up to?
a. O.5 to 1 mm before the radiographic apex
b. 1 to 2 mm before the radiographic apex
489. Scenario Pt want to replace 14 space available 5 mm:What is tx
a. Fpd 13,15

b. Cemented crown implant
c. Screw crown retained implant
● I think he means 5mm occlusally .

✅️
490. Pt complain of Complete denture come out when he pull the lip what is the cause :
a. Labial frenum
b. Buccal frenum
c. Posterior palatal seal

✅️
491. 11 missing due trauma 1 day ago pt want replacement :
a. Transitional
b. Rpd


492. Pt came complaint of bleeding and pain around crown done recently pt mentioned that doctor did something

✅️
laser What is the management :
a. Crown lengthening
b. Assure follow up
c. Refabricate another crown

✅️
493. lower anterior recession and high frenum and no keratinized gingiva?
a. frenectomy with free gingival graft
b. frenectomy with ct graft
494. Stock tray retention with irreversible material

✅️
a. Micro mechanical
b. Mechanical
c. chemical
495. Pt with right side face pain and viscle on the same side:
a. Trigeminal Neuralgia
b. Cluster

✅️
c. Migraine
d. Postherpetic neuralgia


496. Senario Pt with C ii with excessive vertical incompetent lips :

✅️
a. Tooth jaw discrepancy
b. Skeletal
497. Hbv when to say vaccinated:


498. Tb when to say infected:

499. Dentist do diagnosis treatment plan:‫مكرر‬

✅️
a. Advocat
b. Medical expert
c. Health advisor


500. Fixed crown 16 with lateral interference were to reduce where to reduce ‫يحذف‬
a. Incline palatal cusp
b. Incline lingual cusp
501. scenario Perio stage pt came with a complaint of food impaction in lower 6 with x ray? ‫السؤال ناقص‬
a. Grade1 stage b
b. Grade2 stage A
c. Grade3 stage3

✅️
502. Vaccination is successful if the (antiHBs)
a. 100 IU\L
b. 1215

✅️
503. Scenario about Immediate Allergy from rubber dam ?
a. Ige
b. Igg

✅️
504. Pt came for dental check up you refer to another doctor you break what ?
A. Veracity


✅️
505. Wikes classification Pt only complain of clicking due to long dental treatment the previous day
a. I
b. II if he has pain
c. III
d. IV

✅️
506. Diabetic pt with acute abscess and the tx planned is extraction Hbc1a = 10:
a. postoperative prophylactic
b. insulin before dental treatment
c. hypoglycemic after procedure
d. prophylactic before procedure


507. Pt female has viral ulcer

✅️
a. Disclose the diagnosis to the husband
b. Discuss with only the pt


508. TB how to indicate its infected in mm
a. >5

✅️
b. <5
c. >15


509. activation during scaling and rootplaning:

✅️
a. Less than 45
b. 45-90
c. Less than insertion angle


510. muscle activated after opening or (closing mouth)
a. Medial pterygoid and masseter
b. Lateral pterygoid and masseter
c. Mylohyoid and lateral pterygoid
d. Mylohyoid and medial pterygoid


511. I bar got broken for pt has metal base rpd what is the conservative way to fix it:
a. WW
b. Recasting bar clasp
c. Refabricating


512. What is the absolute contraindication for i bar clasp:


a. High esthetic demand
b. Severe undercut
c. Lingual or labial tilted


513. Periodontist started perio surgery with horizontal bone and multiple vertical defect one wall (hemisepal) defect
what is the interproximal correction is called:


a. Radical
b. Interproximal flattening


514. What could prevent interproximal papilla from healing after surgery
a. Releasing flap


b. Intrasulcular incision
c. Persistent inflammation
515. Intracanal medicament accident What is the clinical immediate management


a. Surgical
b. Follow up
c. Antibiotics


516. to Locate of orifice endo


a. File
b. Explorer


517. Restoration of caries extending to dej
a. Composite
b. Amalgam


c. Gic
d. RMGIC


518. Orbit fracture ecchymosis what type of x ray :


a. Submentovertical
b. Ct scan

519. Resorption of ridge of upper


a. Inferior medially
b. Superior medially


520. long senario about pt complain of 47 pain and after examination and panorama there is dentigerous cyst in 38
region he say what is the immediate management :


a. Rct only for 47
b. rct ad tell the pt about lesion in 38
c. Start treating lesion


521. Pt complain of pain posterior to 17and she has fixed orthodontic appliance what could be the possible cause
a. Extended wire


522. Pt with CII and parent was asking about the possibility of mandible deficiency what could aaid in dx:


a. SNA
b. SNB
c. SNOP
d. SNmg


523. Normal platelet count to do procedure:
a. 50000


524. Pt came for rct after access you could not access any canals due to calcification as shown on xray what to test is
the best :


a. Methylene blue
b. Bubble champ


525. Pvs impression and immediate pouring Caused :
a. Hydrogen gas


526. Upper Cd Lower rpd ci What can happen after years:
a. Resorption of posterior lower


527. Black indicator


a. Repeat the cycle
b. The sterilisation killed all spors on paper
c. correct time and temp
d. It show that its Correct time


528. Which rotary file had reciprocation movement :
a. WaveOne
b. Protaper


529. Why remove smear layer during endo :
a. To improve properties of sealer
b. To allow bacterstatic properties
c. To reduce setting time of sealer


530. What is the best financial or economical preventive:
a. Fluoride gel
b. Fluoridated water ✅
c. Fluoride varnish


531. Avulsed tooth in milk for 3 hours what is the management
a. Emerge in 2.%sodium fluoride for 20 min and splint
b. immediately with rigid splint


532. Pt asking for Hollywood smile guarantee for life
a. Explain it's not true
b. Tell her it's only 5 year guarantee


533. oral melanotic macule pic and said what is the management
a. Observation
b. Surgical removal
c. Biopsy


534. pt with herpetic ulcer with fever and malaise:


a. Refer to pedodontist
b. Reassure and antiviral


535. How to locate pterygomandibular
a. T burnisher
b. William probe
c. Mirror
536. after space analysis 26 ml and the space required 25
a. Crowding
b. Normal
c. Spacing


537. 15 years with class III and reverse overget with 10ml to start managing
a. Immediately
b. In 3 years
c. In 6 years


538. Pt with multiple with discolouration white patch covering all the teeth poor oral hygiene and diet with
carbohydrate mother said she has the same:
a. Amelogénesis imperfecta
b. Dentinogénesis imperfecta
c. Dental caries
539. Senario Pt came complaining of how he looks he has poor oral hygiene multiple caries and cii or iii i don’t
remember clearly :
a. Orth


b. Surgery
c. OH,restoration ,ortho surgery


540. What is the cause of the broken file?
a. Inadequate access cavity
b. Curve

c.


541. A patient presents with an OPG where the right side is larger than the left.:
a. The patient is looking to the left.
b. The patient is looking to the right.
c. The patient is looking upwards.
d. the patient have a genetic problem


542. A 9 year old child presents with a double horizontal root fracture. What is management?
● Question given without options.. It depends whether the tooth is primary or permanent.


543. A 5 year old child has lost all anterior primary teeth. What type of dental appliance would be most appropriate
for this situation?


544. A patient presents with a minor tongue injury due to using a palatal crib. What would be your course of action?
a. Reassure the child.
b. Reassure the parent.
c. Remove the palatal crib
545. A patient returns two weeks after an anterior tooth with a metal post threaded crown has fallen out. What would


be your approach?
a. Reattach the crown.
b. Remove the post and fit a new post on the same crown.
c. Remake everything.
546. A patient reports discomfort while biting and a gap between the gutta percha and the metal post. The patient
desires relief from the pain. What would be your course of action?


a. Perform endo surgery.
b. Redo the RCT.
c. Reassure the patient.
d. Prescribe a analgesic
547. To have a high posterior cusp?


a. Shallow condylar guidance
b. Increased vertical overlap of anterior teeth
c. Increased horizontal overlap of anterior teeth


548. What type of incision to raise the flap coronally?
a. Internal bevel
b. External bevel
c. Sulcular ✅

If releasing incision in options it is more correct


549. lesion in the tongue near foramen cecum
a. Choristoma


550. patient has unilateral crossbite caused by
a. unilateral maxillary constriction


551. Patient came complaining of sensitivity in the lower right area, clinical examination showed caries on #47 With
generalized calculus deposits, radiographic examination showed an impacted molar, what is the best treatment
sequence?
a. Restoration, scaling, extraction


b. Extract, scaling, restoration
c. Scaling, restoration, extraction


552. case of anterior upper central recession with procedure done, what is the procedure? (The picture has semilunar
flap and connective tissue graft and both options were in the answers)
a. semilunar flap
b. connective tissue graft


553. case of a patient gagging with a hairy yellow tongue, what is your management?


a. biopsy
b. brush tongue
c. no tx



554. Periodontist wants to evaluate his patient after 6 years of perio treatment. What should he expect?


a. Compliant patients have better improved attachment loss
b. Noncompliant patients have worst attachment loss


555. The patient does not compliant because her denture came with pinpoint "hematoma" (red dots, I forget the exact


term ) diabetic and controlled. What should the dentist do ?
a. Antifungal
b. Wear the denture only during daytime.
c. Take vitamins.
d. Remove denture until healing of the lesion.

556. A patient with Down syndrome, deemed capable of making treatment decisions by a psychologist, requires a


consent form. Who should provide the consent?
a. The father
b. The mother
c. The doctor
d. The proxy maker


557. Transmission of disease from mother to baby :


a. Horizontal transmission
b. Vertical transmission
c. Bottle feeding


558. “Xray of a lateral incisor no caries or resto but with vertical bone defect to mid root“ they mentioned that there is
deep probing around the tooth and the tooth is necrotic


a. perio disease
b. Dens invaginatus

559. Implant crown with 5 mm probing with bleeding and a cropped radiograph of only the fixture (u can’t tell if it’s
screw or cement retained but no exccess cement present) clinical picture of slight recession but not extending to
show the threads


a. excess cement
b. retighten the screw
c. adjust the contour
d. adjust the occlusion


560. “Sagittal view of a molar with radiolucency below the furcation and roots only, the mesial and distal bones are
fine” they mentioned in the Qs there is deep probing depth in the buccal and the tooth is necrotic with no plaque or


calculus?
a. Endo only
b. primary Endo secondary perio


561. Same pic



a. Implant analog
b. Transfer pin


562. What type of gingivitis extends from margin to mucobuccal fold?


a. localized marginal
b. Localized diffused
c. localized papillary


563. Case about pedo pt with fever 37 C and ulcer in the soft palate no other ulcer in the body


Diagnosis?
a. Herpangina


564. what is the name of projection in palatal of maxillary central incisor make difficult in occlusion
a. Talon cusp


565. Pt diagnosed with jaundice 2 years ago, hepatitis profile shows antihbs and antihbe?


a. Carrier and low infections
b. immune

566. Pt with easily bleeding gingiva with haemorrhage under in her skin, multiple epistaxis , all haematological tests


were normal except platelets less than 25,000, what is the probable diagnosis?
a. Thrombocytopenic purpura


567. Picture Case of peripheral giant cell granuloma histo and clinic


568. Patient presented with nonscrapable White corrugated patches on the buccal mucosa close to the corner of the


mouth that does not change when stretched. What could be the diagnosis?
a. White sponge Nevus
b. Biting sign


569. Ask about the cyst between molar teeth with vital teeth ?
a. Stafne bone cyst


570. OPG of patient having irreversible pulpitis in 37 and the 38 is impacted under 37 and having large cyst. I think
(dentigerous cyst) what you will do first?


a. tilling the pt about the cyst
b. Treat #37 and tell the pt
571. A patient presents with a condition where measuring the Probing Depth is challenging. What type of gingiva the
patient have?
A. Thick scalloped


B. Thin scalloped
C. Thick flat
D. Thin flat


572. ⁠gloves torn during procedure and covered with blood…management
a. wash hand with water and soup
b. wash hand with water and soup and alcohol


573. undercut gauge ( pic)


574. ⁠complete unilateral cleft palate and lip ( pic)


575. ⁠ 9 of rotary file 10 Taper 0.02 ?
D


576. A Patient need mandibular advancement?
a. BSSO

577. Pic lefort 2


578. calculate canine proportion if lateral 5 mm ?
a. 3.1


579. Most accurate method for determining biological width ?
a. Vertical bitewings
b. Probe


580. patient come with Lesion 8X5cm It was discovered by accident yeahWhat’s your management?
a. Incisional biopsy
b. Excisional biopsy
c. Mandibular resection


581. better medication for reduce pocket
a. Monocycle gel


582. the patient has 6 anterior PFM crowns unsatisfied with her appearance. Why ?
A. Over contour and opaque
B. Under contour and opaque
C. over contour and translucent
D. under contour andtranslucent

583. content of titanium alloy


a. Silver


b. Gold
c. aluminium
d. Palladium


584. pic of short roots with bone replacement
a. external replacement
b. surface resorption
c. Cervical resorption

585. upper 1st premolar, how many canals?


a. 1
b. 2
c. 3
d. 4
586. pic of semilunar flap


587. picture of Ridge reservation


588. what type of trauma is the central incisor longer than adjacent teeth?
a. Extrusion


589. dr Post the pr photo Violated?
a. Patient information
b. privacy


590. cavernous thrombosis
a. Canine
b. Deep temporal


591. patient during treatment, change, mood and He started to sweat, headache?
a. hypoglycemia
b. Hypothyroidism


592. scenario about cemented implant and ask about minimum distance
a. 7-8


593. time for hand washing?
a. 40-60

594. definition of collaboration


595. patient pointed to the lateral incisor, and she complained of the shape?
a. Morphodifferentiation


596. picture of pyogenic granuloma? And ask about treatment, it covered the whole occlusion.?
a. excisional


597. The most difficult tooth in anesthesia when a patient has symptomatic irreversible?
a. Lower molar
598. ask about patient complaint of retention of recently completed denture ( if everything it was okay, except flat


ridge)
a. Give him a Adhesive
b. Remake
c. Reline
d. rebase
599. ask about hyperplasia in incisors tip and occlusal surface of molar?


a. Tetracycline
b. Fluorosis
c. Metallic
600. scenario about a patient having jaundice?


a. Infection
b. Bleeding
As the Scenario is missing, there was a similar question that has the same answer (Bleeding) the question was
about (jaundice, what is the most feared complication after extraction?)
General knowledge about Jaundice:


601. Tooth diagnosed with condensing osteitis Sudden disappearance of canal?
a. canal, bifurcation
b. calcified

602. In a scenario involving a newly fitted denture, the dentist suspects that the freeway space is insufficient. How
should the dentist determine if the freeway space is indeed deficient?


a. Ask the patient if there is any clicking sound.
b. Measure the occlusal and rest vertical dimensions and subtract them.

603. What are the potential impacts of an impacted mesioangular class C wisdom tooth (tooth number 8)?

Due to lack of choices, there are 2 ways to interrupt the question: 1Impaction complications or 2 Impaction surgery
complication (removal):

1Impaction complications (Mentioned before and the answer was [Affect the second molar ✅]):


604. What is a combination clasp in dental prosthetics?


605. What is the best intraoral site for graft harvesting?


a. maxillary tuberosity
b. the retromolar pad area


606. For a patient with a prosthetic valve and no drug allergies, what prophylactic antibiotic should be administered?


Options are:
a. 2 gm Amoxicillin
b. 1 gm Penicillin
c. 600 mg Clindamycin


607. Safely tolerated dose of fluoride mg/kg for adults?
a. 0.5
b. 5 (Toxic dose)
c. 10
d. 15
608. Bluish black discoloration and gingival inflammation?


a. Lead intoxication
b. Bismuth
c. Arsenic
d.
609. the x ray for infra orbital ecchymosis ? (Mentioned before)


a. mri
b. ct
c. submentovertex view
d. occipito mentovertex ( water veiw )

610. first step when a child abuse mark was noted?


a. Reassure and review again
b. Documentation and report

611. pedo pt came with his mother for treatment of cavities. He was very apprehensive and the mother wanted dental
chair treatment. The primary molar has multiple surface caries. What restoration is used ?


a. composite
b. SSC
c. amalgam
d. Gic


612. a diabetic pt come to hospital bec of ketoacidosis and gangrene in foot he is in coma and Dr decided after he
wake the will amputation his foot but his sons and daughters disagree even after the dr explain the risk of septicemia
if the didn’t cut his foot and the son told they rather let their father die with complete body than to cut his foot which
will make the decision?


a. court
b. pt family
c. doctor
613. pt came to hospital and after the doctor saw the document that his case is hopeless and no need for rapid
responses the pt family disagreed and while they discussed something happening to the pt what the nurse will do ?


a. Call the team
b. not call them and let it die
614. question about what will prevent or destroy organisms from inanimate surfaces?


a. antiseptic
b. disinfection

615. what is the minimum preparation, not thickness of metal retainer of resin bonded bridge?
a. ✅
1-0.5
b. 2-0.7
c. 3-1.1
d. 4-1.5

616. pt need complete denture and he has the lower ridge buccal to upper ridge how you will put the teeth ?
a. Class 1
b. Class 2


c. Unilateral crossbite
d. Bilateral crossbite
617. a 4 year pt have multiple tumour masses in the jaw which causes early missing of primary teeth and have
malignant characteristics. What organism causes it ?
a. Hiv
b. Hhv8


c. Cmv
d. EBV


618. a pt hypertension and has aortic stenosis and he needs extraction. What will you do ?


a. give him 2g amox 1 h before
b. no need
c. give him 1g amox 1 h before

619. dr wash his hand for 60 sec and after that he puts one pump of alcohol gel and rub for 30 sec. What can improve


his technique?
a. Dry the hand after washing then rub
b. Add another pump of gel
c. Check the hand if contaminated
d. something about the time !

620. pt have submandibular space infection and 38 c fever ask about the order of treatment?
a. I&d then antibiotics till reduce infection then extraction
b. I&d then extraction then antibiotics
621. pt came to the dental clinic because of sensitivity in lower #47 which have deep caries and have calculus and dr
saw impacted 3rd molar the order of treatment?


a. Restoration then scaling then extraction
b. Scaling then resto then surgery


622. a test that shows the glucose level for the last 2 months?
a. fasting glucose test


b. random glucose
c. HbA1c

623. a drug that cause gingival hyperplasia?


a. beta bloker
b. Calcium channel blocker

624. A patient came want to replace his missing teeth. See the OPG (missing lower anterior teeth and periodontal
compromised posterior teeth) which of the following is the suitable major connector?
a. Lingual bar


b. Sublingual bar
c. Lingual plate
d. Labial bar

625. question about pt have implant in upper lateral and pt complain of black line asking what is the cause (there was
a pic I didn’t see black line just blue shadow at cervical area with good and healthy gingiva)


a. labial position of the implant
b. shadow of titanium abutment
626. pt have overjet and inverted lower lip and deep labiomental and in ceph report they wrote the pt have normal
SNA and decrease in SNB the pt have ?


a. Lower deficiency
b. Lower deficiency and upper prognathic
c. microdontia
627. question about slowly growing swelling in the hard palate with non ulcerated mucosa and bone erosion what is
the lesion?
a. Pleomorphic adenoma


b. Mucoepidermoid carcinoma
c. adenoid cystic carcinoma

628. question about recurrent EM and herpes and ask what prevents the recurrent?
a. systemic steroids


b. systemic antifungal
c. Antiviral


629. Pt aortic stenosis and hypertension need prophylaxis antibiotics or not ?
A. No need
630. psychiatric patients have swelling buccal near the parotid gland?
a. cheek biting


b. drug interaction
c. Eating disorder


631. glass ionomer cements in comparison with composite have??
a. excellent coefficient of linear thermal expansion
b. modulus of elasticity


632. Patient came to the orthodontist complaining of his appearance, Cephalometric analysis revealed: ANB: 6


Upper incisors angle to SN: 108 Which angle classification does he has?
a. C II de 1
b. C II de 2
633. The patient has leukaemia, and you did extraction, and the bleeding did not stop ; what is the cause?


a. Increase calcium level in blood.
b. Platelet deficiency
c. Platelet disorder
d. Issue in internal clotting complex

634. Women has Breast cancer, and she is now under chemotherapy and came to your clinic for a routine procedure.
What is the recommended approach?


a. Do the routine treatment only
b. Defer the routine treatment after 1 months of chemo
c. Defer the routine treatment after 3 months of chemo
d. Defer the routine treatment after 6 months of chemo
635. 52yo man came for routine check up and you found a deep stained groove in 36 after checking it with an


explorer . There is no softness. What is the appropriate treatment?
a. No treatment required
b. Fissure sealant
c. PRR
d. Class I composite
636. 32 yo Patient with prognathic mandible has 6mm show (written exactly like this) and he is only concerned about


esthetic only. Which treatment is required for him?
a. Orthognathic surgery
b. Maxillary expansion
c. Mandibular expansion
d. Camouflage treatment
637. with finger projection of the palate and fungal infection and generally ill fitting denture What is the early


treatment to start with?
a. Prosthesis adjustment and oral hygiene
b. Antifungal


638. What is the maximum length that the curette can achieve in a nonsurgical department?
a. 2.75


b. 3.75
c. 4.75
d. 5.75


639. 56 yo patient has all upper left teeth mobile and indicated for extraction. Which sequence is required?
a. Posterior then anterior
b. Canine then all other teeth
c. Anterior then posterior


640. To have a good antimicrobial effect from using calcium hydroxide it should be placed in the canal for ?
a. 24 Hours
b. 3 days
c. 2 weeks ✅
d. 4 weeks


641. What is the maximum size of endo voxel?
a. 0.2mm
b. 0.02mm
c. 0.4mm
d. 0.04mm

642. What are the minimum implants in mandibular overdenture?


a. 4
b. 2
c. 6
d. 1

643. 48 yo diabetic patient came for follow up complaining form upper RPD he feels discomfort while wearing it
what is the management? same this pic and the choices are exactly like this?
a. Prednisone


b. Prescribe insulin
c. Reduction from the fitting surface
d. Prescribe acyclovir
644. Patient healthy and discuss with the dentist about his advice on how to choose dental floss, so choosing the


dental floss is based on?
a. using the most comfortable for him
b. according to the proximal contact tightness
c. according to the roughness of contact
d. based on manual dexterity.

645. 38yo diabetic patient you will do final impression for him, What is the type of retraction cord is contraindicated
to be used with diabetic pt?


a. Epinephrine
b. Zinc Chloride
c. Aluminium chloride
d. Aluminium oxide
646. Pedo patient and you just finished PRR and pit and fissure sealant for him. When should we see him regarding


follow up?
a. 3 months
b. 6 months (not sure)
c. 12 months
d. 2 years

647. Patient has sarcoidosis and you take a biopsy from swelling of a minor salivary gland. What will you see?
a. Fibrosis
b. Acanthosis


c. Acinar hypertrophy
d. Granulomatous inflammation


648. Which instrument has an active cutting tip?
a. K file
b. H file
c. Reamer

649. Patient has Kennedy class I and you found a usable undercut on the last abutment. What is the most flexible
clasp and its cross section ?
a. cast Clasp with round cross section


b. Cast Clasp with half round cross section
c. WW with round cross section
d. WW with half round cross section


650. What is the type of antibiotic prophylaxis for patients with ventral arrhythmia and pacemaker?
a. No prophylaxis is indicated.
b. Amoxicillin
c. Clindamycin
d. Metronidazole

651. Which of the following factors doesn't increase GCF fluid?


a. Smoking


b. Hormones changes
c. Trauma from occlusion


652. Scenario of patient has asymptomatic buccal fat swelling see the pic, What is your diagnosis?

a. Lipoma ✅
b. Mucocele
c. Pyogenic granuloma


653. rinsing before crown lengthening
a. to reduce aerosol


654. MI ASA classification


a. Before 3 months ASA 4
b. more than 3 months ASA3
655. histopathological granules with vascular and ask about the disease ?
If means sulfur granules “ Actinomycosis ”


656. gutta percha length 22 and ask about the length of the post .
a. 19-17 mm (normal range of GP apical seal from 3-5mm)
657. pic of periodontal abscess with pus discharge :


a. RCT
b. incision and drainage
c. extraction ,
d. Observe
658. needle stick injury with HIV pt
a. leave it


b. wash direct and ask pt to do test
c. go direct to doctor and take prophylaxis and test


659. angular cheilitis supplements
a. iron
b. Zinc
c. calcium


660. Increase of VD
a. gag reflex
b. tmj dysfunction
c. angular chelitis

661. What to do if there is a bony undercut in the anterior maxilla and tuberosity region
a. no need intervention it’s good for retention
b. relieve the CD


c. remove the under cut in both sides
d. Relief anterior , surgical correction of bilateral posterior

662. pt with hx of renal transplantation, hx of recurrent aphthous ulcer , herpic labialis . And there is pic of two
ulceration in floor of the mouth and ask about the diagnoses and blood investigations WBC low and neutrophil
normal
a. HPV


b. traumatic ulcer
c. recurrent stomatitis
663. metallic sound found in
a. lateral luxation
b. Subluxation


c. Intrusion
Both intrusion and lateral luxation


664. pain increases while eating hot food and reduces with cold ?
Hot tooth “ symptomatic irreversible pulpitis”
665. why D4 in implant is the worst
a. high occlusion


b. high vascularity
c. lack of primary stability


666. 5 yrs with mandibular mesial shift, which one is the proper intervention?
a. wait until mix dentition “ 6-9 years mixed dentition”
b. Habitus breaking appliance
667. 13 yrs avulsed of 11 and already keep it in milk (

a. extra oral RCT
b. keep it in NF
c. splint it directly


668. blood during pulptomy (mentioned before)
a. irreversible pulpitis
b. reversible pulpitis
c. necrosis


669. severe class 3 concave profile(need details )
a. mandibular set back with maxillary investment
b. surgical with maxillary expansion
670. Picture of upper and lower arch for patient 14 years old his cheif complain is unerupted permanent maxillary
canines ( upper arch there is no space for canine eruption but no crowding present) lower arch only anterior crowding

🔁
Over jet is normal, no shift in the midlineMolar relation ship is class II subdivition left ( not very sure)
a. extraction ( I think camouflage)
b. Expansion
c. Distalization
d. non extraction


671. pic of swelling in the gingiva with histopathological discription of fingerlike projection….. etc
a. Squamous papilloma
b. Fibroma
c. Pyogenic granuloma
d. Peripheral giant cell granuloma


672. pedo patient with multiple cavitates ,bad OH cut in labial frenum , neck and arm bruises
a. dental caries
b. Gingivitis


c. Periodontitis
d. Abuse and neglect


673. posterior cross bite and open bite and thumb sucking appliance?
a. Quad helix


674. pt have overjet and inverted lower lip and deep labiomental and in ceph report they wrote the pt have normal


SNA and decrease in SNB the pt have ?(mentioned before)
a. Lower deficiency
b. Lower deficiency and upper prognathic
c. microdontia
675. Pic of odontoma bt the 2 centrals what is Tx?


a. Marsupilization
b. Excisional biposy
c. Resection
d. Follow up


676. disinfection removed?
a. Blood
b. Fluid


c. Blood and fluid
d. Virus


677. the process of preventing the growth of microbial and killing it in inanimate surfaced?
a. Disinfection
b. Sterilization
c. Antisepsis


678. patient want to construct ceramic anterior crowns she told the doctor that she wants the whitest color, doctor
advise that it will not be natural (mentioned before)
a. Show her the shades that is suitable for her case
b. Refuse treatment because she didnt respect doctor judgment
679. Diamond bur with wrong angulation during prep
a. Undercut

🔁
b. Vertical deep finish line
c. Over contour preparation
680. pt is missing from 14-17, 34-37,44,47
a. face bow maxillary and occlusal rim in MIP
b. Arbitrarily and occlusal rim in MIP


c. Arbitrarily and occlusal rim in CR
d. face bow maxillary and occlusal rim in CR


681. patient with stable MIC with difference between it and CO , wants to construct crown you put it in:
a. MIC
b. CO
c. Between them


682. patient has hep B active , dentist injured but she is vaccinated what to do?
a. Test titer of anti hbs
b. Take immediately HBIG
c. Nothing


683. TB active patient with urgent ttx
a. Respirator mask


684. you told the patient all the possible complication and instructions:
a. Veracity
b. Autonomy ✅

685. frankfort horizontal plane from?
a. porion to orbitale


686. A patient presents with a gunshot wound affecting the bone and teeth in quadrant four, with the exception of one
area. A picture of a denture with an implant for quadrant four is also provided..‫حسب الصورة‬


a. Overdenture
b. Fixed hybrid denture
c. Cemented FPD
d. Partial Removable retained implant


687. patient has swelling in upper lip he takes( ACE inhibitor methotrexate plavix) What causes the swelling?
a. ACE
b. Plavix
c. Methotrexate


688. case of pedo patient have lateral incisor with caries incisally and proximally they want esthetic ttx:
a. Resin faces ssc
b. Ssc
c. Composite


689. class III kennedy mandibular have mid buccal undercut
a. Rpi
b. Circumferential
c. Double
d. Reverse


690. special tray impression for final impression?
a. Rigidity
b. Strength
c. Flexibility


691. most ductile and malleable
a. gold
b. Titanium
c. Platinum
d. Amalgam


692. whats metal used in denture
a. Alloy
b. Titanum
c. Palladium


693. 75 year pt came to you with pain in 46 the doctor said to him i will extract and put FPD without telling him all
the possible treatments such as endodontic , implant ,,, and the doctor think that is better for him to prevent future
disease what the patient violate?
a. Informed consent
b. Treatment quality


c. Nonmaleficence
d. Autonomy


694. pic of round RL in the middle of the canal and asking what is this
a. Internal resorption


695. Non vital bleaching can cause what type of resorption?
a. External cervial
b. Internal
c. Surface


696. submandibular space infection in one side ask about the treatment
a. 1. I&D then remove the cause after that AB
b. Antibiotic and observe

697. melanotic macule and ask about treatment:
a. Observation


b. No ttx is required
c. Excision
d. Resection


698. all normal except one tooth you want to retract it:
a. finger spring
b. Hawley retainer with active labial bow

699. ceph picture pt with class II div1 he wants to do surgical treatment for retrognathic mandible:


a. Trans oral vertical osteotomy
b. BSSO
c. Lefort

700. A picture shows a flap extending approximately from the canine to the first molar, followed by a vertical incision
what will happem??


a. Limited access
b. Flap necrosis


701. mother telling the dentist she is worry about her child because she has missing teeth and afraid that her child


have the same problem( he is 10 years) ,, he probably miss which tooth?
a. upper lateral
b. Lower lateral
c. Uper first molar
d. Lower first molar


702. ro lesion in x ray under vital tooth and its asymptomatic?
a. Idiopathic osterosclerosis
b. Condensing ostitis
c. Cementoblastoma


703. patient with upper and lower complete denture have ulcer in lower frenum what is the cause?
a. Overextended lower labial flange


704. patient have immediate denture after extraction and then he didnt come to change it, after 4 weeks he came with
complain in the lower denture ‫كان في صورة وفيها تيشو زايدة‬


a. papillary hyperplasia
b. Epluis fissuratim


705. patient with nug what is the imp factor (Necrotizing ulcerative gingivitis (NUG))
a. Emotional stress
b. Periodontitis


706. patient with nug what you will do?


a. Antibiotics
b. Non surgical treatment
c. RCT
d. extraction


707. patient with multible sclerosis?
a. Have bilateral trigemninal neuralagia

708. the patient came after extraction of an impacted third molar with 4 weeks of pain and trismus and swelling.
What may he have?
a. Dry socket
b. fractire of buccal bone


c. pericoronitia
d. Subperiosteal abscess


709. fruit laden pranchless tree and shcrimer test 3 mm in 5 min?
a. Sjorgen
b. Behcet


710. ocular lesion( conjunctiva) and oral and ( epithelium rupture under minor touch)
a. Pemphigus Vulgaris
b. MMP
c. SLE


711. Epstein pearl
a. midline

712. pt with hx of Myocardial infarction and came to for PFM


a. defer the tx
b. do it and avoid retraction cord with Epinphrine
c. RPD

713. pt with chipped half of crown (pFM )


a. Remake it in lab
b. Fix it in clinic


714. Which orthodontic instruments are can be sterlized in an autoclave


715. 21 year old Patient, what will happen when she’s menstruating?


a. bone loss
b. increase exudate and bacteria
c. tooth mobility

716. gingival col occurs in?

717. Main advantage of quad helix


a. skeltal affect


b. rapid
c. doesn’t depend on the patient
718. Feature of osteomyelitis ?


a. Bone loss
b. Sequestra

719. Patient with generalized bleeding and eryhtema loss of stippling probing 13mm very good hygiene what is the
cause


a. over contoured ceramic
b. allergy to toothpaste
720. Patient with jaundice and has HBsAg postive HBsAe
a. highly infective and
b. carrier low infectivity
721. Patient heavy smoker has a lesion on the tounge with indurated margin what type of biopsy?
a. Incisional biopsy
b. Excisional biopsy
c. FNA
Note : if there is waiting for 14 days before taking biopsy and after removal of stimuli will be the right choice
if not depending on the size


722. pic of implant retained crown with recession asking about the reason?
a. Remaining cement


723. the same pic asking about the treatment ?
a. Remove the the remaining cement


724. ibuprofen dose for a 30kg child ?
a. 6-15


725. During try in of crown there was slide anteriorsuperior slide. How to adjust?
a. mesial incline of max
b. distal incline of mand
c. buccal of max
d. lingual of mand
726. the doctor is checking occlusal disharmony of a young patient, and there was interference in an eccentric


relationship. Centric relationship was normal
a. reduce upper incisal edge
b. reduce lower incisal edge
c. buccal of upper
d. buccal of lower
Note: if there is reduce from lingual inclines of maxillary will be the right choice

727. ceramic advantage


a. opaque
b. tensile

c. transparent
d. translucent

728. diabetic patient with a controlled condition wants to replace the missing areas?
a. removable


b. fixed
c. implant
d. Can’t replace it
729. diabetic parents taking medication for diabetes (sulfonylureas), recently he was diagnosed with arthritis and it
was given corticosteroids, he came to you and gave him ibuprofen. At night he called you saying he is tired and
feeling numbness all over his body what happened?


a. hyperglycemia
b. Drug interaction
c. Not related to
d. Hypotension

730. during treatment patients feel pain that radiates to nick and lower jaw and shoulder the doctor used (spry) in the


area under the patient tongue, what is the side effect of this medication?
a. Xerostomia
b. Burning
Note: in previous files they mentioned if it is spry burning if it is tablet Xerostomia but what I found in the
reference is a Xerostomia


731. Patient after ortho treatment not satisfied with the results and has gingival overgrowth (He has multiple


discoloured teeth and short crown) what tx you provide to enhance the esthetic 2 mm from crestal bone
a. Gingivectomy
b. crowns lengthening
732. Patient has 35 recesion on her upper anterior teeth and thin gingival biotype , what is the tx ? (Same photo)


a. Crowns
b. Connective tissue graft
c. widman flap
d. apically positioned flap
733. Initial way to diagnose VRF


a. Occlusal xray
b. CBCT
c. PA


734. Ortho cephalo findings asking about diagnosis ( SNB was dec and incisal NS was increased )
a. Class I
b. Class II


c. Class II DIV I
d. Class II DIV II


735. which plane is used to describe wits appraisal
a. Functional occlusal
b. frankfort plane


736. Patient with Desquamative gingivitis + ocular infection what test you will take


a. Bleeding test
b. Immunofluorescence & biopsy


737. 23y old Pat has history in family with severe periodontitis ,he is diagnosed with periodontitis Stage III , grade C
and asking about what is the most initial treatment to do for him
a. scaling and follow after 46 week
b. scaling and follow every 2 months


c. clearance and prosthetic treatment
d. scaling and applying AB amoxicillin + metronidazole


738. Pocket depth equals Clinical attachment when ?
a. Pocket depth 1 mm above CEJ
b. Pocket depth 2 mm above CEJ


c. Pocket depth 3 mm above CEJ
d. Pocket depth at the level of CEJ

739. Patient complained of severe pain , upon examination everything was normal , but asked for analgesics and he
was demanding , in this situation what you will do ?
a. Prescribe for him
b. Bring the security


c. Apologies and refuse politely
d. Refer for second opinion


740. what can you have from the patient to achieve a diagnosis before clinical examination
a. Pt interview
b. Radiographic examination
c. Impression for diagnostic cast


741. 14 y patient seek orthodontic tx his diastema IOE shows low frenal attachment, what should be examined before


initiating the ortho tx
a. blanching the incisive papilla with manipulation

742. Patient extracted a third molar a week ago and came with yellowish granules , what could be the diagnosis ?


a. Tuberculosis
b. Actinomycosis
c. Syphilis
743. Photo of lichen planus and asking about the type


744. A OMFS Resident he want to do horizontal third molar extraction , and he was afraid that the patient will not
sign the consent if he knows that paresthesia is one of the complications , and the risk of not extracting the tooth will
cause a mandibular resection in the future , what to do ?

a. Make the family responsible
b. Let the patient aware of everything


745. Pt with nephropathy Diabetic , what is the suitable analgesic?
a. Acetaminophen
b. Ibuprofen
c. Diclofenac
d. Aspirin

746. After anaesthesia of #16 pt start feeling itch and Swelling of the left side of the upper lip ,What type of LA cause


this
a. procaine
b. lidocaine
c. Prilocaine
d. xylocaine


747. 19 y old with maxillary constriction , what is the tx ?
a. SARPE


748. A 45 y old patient came with throbbing persistent pain and skin rash , asking about the causative factor ?


a. Herpes simplex
b. Zoster


749. what is the maximum days u must storage the instrument Type B autoclave
a. 21


b. 15
c. 30


750. patient has trauma 1 year ago , and she came with throbbing pain what can to prevent further complications
a. Chx irrigation

b. Double side vented needle
c. Single ended side needle
d. Low concentration Naocl

751. Trauma case photo asking about the results (depending on pic)
a. Extruded tooth with internal resorption
752. Q asking about what type of resorption (xray) (depending on pic )
a. Surface resorption
b. Replacement resorption
753. Tooth with displaced 1 mm buccally , mobility miller classification


a. Class1
b. Class 2
c. Class 3


754. Recession in the margin gingiva and not extended to mucogingival junction , what type ?
a. Class 1
b. Class 2
c. Class 3
d. Class 4

755. instrument used to finish the buccal an lingual proximal wall ?


a. Gingival margin
b. Enamel hatchet
c. Chisel


756. similar photo taking about the condition ?

a. taurodontism
b. Dentin dysplasia
757. Patient has swelling in mandibular and she wast complaint from it , but she came and she feels it’s getting bigger
and she want to remove it , intra orally everything was normal , what could it be ?


a. Basal cell adenoma
b. Pleomorphic adenoma


758. Pat has welldefined and unilocular radiolucency surrounding the crown with ameloblastic lining … what is the


management ?
a. Local surgery …
b. Mandibular resection
Note: if there is enculturation on the choice will be the right choice


759. Dentist do diagnosis treatment plan


a. Health Advocate
b. Medical expert
c. Health advisor


760. What could prevent interproximal papilla from healing after surgery
a. Releasing flap


b. Intrasulcular incision
c. Persistent inflammation


761. Which of the following articulators captures the record of mandibular border movement?
a. Fully adjustable
b. Arcon
c. Non acron
d. Semi adjustable


762. to Locate of orifice endo?


a. File
b. Explorer


763. Patient came complaining from food impaction. Did an OD amalgam restoration with tight contact smooth and
flat distal wall which of the following is cause of the patient CC?


a. Lack of embrasure
b. Open contact
c. Occlusion trauma


764. Normal platelets count to do procedure? ‘
a. 50000


765. Which rotary file had reciprocation movement?
a. WaveOne
b. Protaper


766. Why remove smear layer during endo?
a. To improve properties of sealer
b. To allow bacteriostatic properties
c. To reduce setting time of seale


767. Which pf the following is an disadvantage of zirconia core:


a. Fracture
b. marginal discrepancy
c. weak bonding with porcelain


768. a patient complaining about RPD Kennedy class I, has direct and indirect retainer, upon examination, the rest
seats are not stable when force is applied to artificial teeth, what is the Tx?


a. change denture.
b. change position of rest seats

769. medical compromised patient came for extraction of 21 under GA and Doctors extract 11 tooth but he realized
and bring it back and extract 21


a. write incident report.
b. tell the patient.
c. nothing to do


770. Complete denture delivered to the patient with wellbalance occlusion. He removes it at night then in the
morning he can’t insert it. What is the cause?
a. Lack of frenum relief
b. Lack of insertion skills


c. Something in occlusion
d. Left in dry filed at night


771. What is the role of flux in soldering?
a. improve flow of solder
b. reduce melting temp of solder.
c. act as separating medium


772. the dentist washed his hands then applied one pump of alcohol for 30 sec. What did he do wrong?


a. he should apply more alcohol.
b. dry hand before alcohol
c. more time to rub alcohol.
d. Hand should be clean before apllying alcohol
773. Tooth with deep caries normal pulp and “sensitive to percussion” not tender, upon caries removal pulp exposure


occurred what is the diagnosis
a. Asymptomatic irreversible pulpitis with normal apical tissue
b. Asymptomatic irreversible pulpitis with symptomatic apical tissue


774. White lesion present from birth but now change on color from white to brown?
a. Enamel hypoplasia


775. what is the disadvantages of midazolam sedation?
a. can cause anaphylactic reaction
b. long duration


c. Hyperventilation
d. Irritability


776. 13 years old pt have mouth breathing tx? (no more info in the Q) ( denpend on the rest of the info but since


there’s crossbite )
a. expansion
b. headgear
c. extraction of teeth
d. distalization


777. What is the treatment



a. bypass or retrieve
b. extraction
c. obturation until the file


778. reactive sclerosis which condition?
a. osteoid osteoma


779. pruritic papule which diagnosis?
a. Dermatitis Herpetiformis


780. rheumatoid arthritis associated with which disease?


a. Paget disease
b. sjogren's syndrome


781. mechanism of metronidazole:
a. DNA inhibitors
b. Cell wall inhibitor


782. picture of tooth that is prepped and the bur is in wrong angulation on tooth and asking what will happen?
a. Undercut
b. overcontour preparation


783. pic and asked about diagnosis, they mention the pt is healthy? (Same pic)



a. idiopathic gingival enlargement
b. inflammatory gingival enlargement


784. Pt wearing CD for a long time and has a red lesion in the palate, what is the diagnosis?
a. Denture stomatitis
b. Papillary hyperplasia


785. Antibiotic prophylaxis for patients with ventral arrhythmia and have a pacemaker?
a. No prophylaxis is indicated.
b. amoxicillin 2


786. Fracture of dentin, cementum and pulp What is the name of fracture?
a. Complicated


b. Uncomplicated
c. Root fracture
d. Enamel fracture


787. fracture of retentive arm with good reciprocal arm how to treat conservative?


a. Solder
b. Wrought wire
c. Do new one


788. lesion that she is concerned about in the lips and after diagnosis you found out that it’s a minor viral infection


what to do?
a. Tell her the diagnosis privately
b. write it in the patient board
c. Tell the husband
d. Tell all the staff in the clinics


789. cleft lip?
a. Failure of fusion of medial nasal process and maxillary process


790. a patient has been diagnosed with Myocardial infarction before 3 weeks and taking aspirin and he has a severe


pain in a tooth tx?
a. Treat now RCT and give nitroglycerin and Monitor
b. Extraction with AB
c. Treat after 3 months of MI

d.


791. pt feel pain behind one eye that radiate to the temporal area and upper anteriors, and he have it at night
a. cluster headache
b. migraine
c. trigeminal neuralgia


792. xray of pt have odontoma prevent eruption of tooth #21 and asking what is the management? Similar pic

a. Excisional biopsy✅
b. Marsupializationz
c. Resection
d. Follow up


793. Patient complain from food accumulation and bleeding, upon clinical examination tooth was previously initiated,
slight tender to percussion, probing depth was 4 or 5mm only on the distal site, with bleeding on probing. What is
your management? (Pic of PFM crown with defective margins especially distally)
a. Deep scaling and root planing
b. Combined endodontic treatment with periodontal surgery


c. Proceeds with endodontic treatment through PFM crown
d. Suction the crown and assess restorability
794. The factor that determines the rate of forces transmitted to soft tissues and teeth in RPD


a. The type of metal in the base
b. Base shape
c. Direction of forces
d. Retainer


795. How to manage bone loss around this implant exact same pic


a. Remove Implant
b. Bone graft
c. Remove excess cement
d. Change the overcounted crown


796. Patient with missing 4 upper anterior teeth and left upper second premolar, how many rest seats?
a. 4 rest seats, 4 retentive arms
b. 5 rest seats, 5 retentive arms


c. 6 rest seats, 6 retentive arms
d. 6 rest seats, 5 retentive arms


797. change in length per unit length of the material as 1°C change in temp:
a. Thermal coefficient of expansion or contraction


798. witch virus even if you take all of vaccine still you will be at high risk:


a. Hepatitis C
b. Hepatitis B
c. Hepatitis A


799. pt complain from pain in all of his teeth and upon clinical exam there is no abnormality:
a. fluoride varnish


b. observe pt
c. request radiograph
d. reassure

800. During surgery of mylohyoid parallel you feel afraid to injure which nerve ?
a. Mental


b. Mylohyoid
c. Lingual


801. a patient wearing a denture with extremely resorbed mandibular ridge complains of pain and paresthesia of the


lower lip. Which of the following nerves is the most probable cause
a. Mental
b. IAN


802. In dental practice using autoclave to sterilize instruments , what do we use in dental clinics


a. dry heat
b. steam pressure
c. ethylene oxide


803. 3.5 or 4.1 implant size what the category:
a. wide


b. small or narrow
c. regular


804. 11 replace it with implant
a. Endosteal root form
b. Endosteal blade form



805. Implant bone loss when you see it will be alarming in mm
a. 2 mm


806. Normal bone loss around implant first year
a. 1-1.5mm


807. Case about Multiple radiolucent radiopaque in all quadrant
a. Florid Cemento Osseous dysplasia


808. Erythematous , genital ulcer Mouth ulcer , lymphadenopathy Ttt:
a. nystatin


b. Penicillin
c. Prednisone
d. Antiviral vir
809. A patient is in a critical condition and the family requests to give him a drug to end the patient's life. The doctor


refuses this request but asks the nurse to stop providing nutrition.
a. passive euthanasia


810. Pedo patient with limited mouth opening and shift mandible to right (OBG mixed dentioned) Management?
a. TMJ exercise
b. Ant post splint
c. Arthroplasty
811. Best obturation technique ?
a. carries


b. Cold Lateral condensation
c. Warm lateral condensation


812. Which of the following Acid base reaction ?


a. Composite
b. Glass ionomer
c. Componer
d. RMGI
813. Most Flexible endo instrument
a. K file


b. H file
c. Reamer
d. Barbid aproach


814. Case with intraoral pic Right was class ii left was class i
a. class ii subdivision right
b. class ii subdividion left


815. Pedo patient give him midazolam 2.9mg/kg orally Sudden he fell dizzy and difficulty in speaking
a. overdose
b. minor sedation
c. moderate sedation
d. Anaphylactic reaction


816. Long Case scenario pt have thrombing + dull pain
a. myelinated nerve fibers A
b. myelinated nerve fibers b


c. Unmyelinated nerve fibers A
d. Unmyelinated nerve fibers C


817. Blood test for haemophilia A ?
a. pTT
b. PT
c. TT
d. Platlet count


818. Patient anxious has thyroid problem present for extraction Vital sign Bp low Hear rate 50 What problem might


happen
a. Myxodema coma
b. Thyriod storm

🔁✅
819. Gold only #25 After 2 weeks patient complain thermal hypersensitivity You found Facet on #23 #25
a. Hyperocclusion
b. sensitivity to cement


820. Patient you give him local anaesthesia , then he told you did local anaesthesia break my fasting?
a. Refer him to religious person in the department


821. CBCT and OPG picture show buccal impacted canine, ask about the position of canine impaction?‫حسب الصورة‬
a. Buccally implanted
b. Palatal impacted


822. Impinging clamp on tissue what can cause
a. Recession


823. Patient present to you complaining about the appearance of his soft tissue related to 13 canine , history of


(buccal impacted canine )what's the problem?
a. Recession
b. Gingival enlargement
c. Similar to 23


824. Wits appraisal


a. Frankfort line
b. occlusion plane ( Functional )


825. Frankel iii
a. Enhance position of maxilla


826. While trying on the final crown ( lithium disilicate) she said I want to place the final crown with temporary
cement to evaluate the sensitivity before final cement. Which cement to use?
a. Glass ionomer


b. Resin cement
c. Non eugenol zinc oxide


827. Maxilla resorption


a. Superior laterally
b. Superior, medially


828. What happens if you start with rest seat preparation before guiding a plan?
a. Inadequate rest seat preparation


829. The patient wanted to place all his sound amalgam restoration because of the amalgam effect of his body. What


is your management ?
a. Explain risk and benefit of removing amalgam
830. Upper 25 post and core crown have pain and you suspect VRF, what is the initial radiograph modality?


a. Cbct
b. PA
c. Bt


831. Patient has symptoms of seizure and then lost consciousness:
a. Supine
b. Semisupine
c. Retroclined
d. Semi Retroclinee


832. Brown participation is due to what :


a. edta with sodium hypochlorite
b. Chx with sodium hypochlorite


833. Patient has stent and valve replacement took prophylaxis and came with symptoms of allergic reaction what to


give him
a. something diphenhydramine


834. Patient complains about current mandibular denture, examination shows severely resorbed ride with firmly


adapted tissues, which of the following is the best impression technique to make a new denture?
a. Admix
b. Neutral zone
c. Selective pressure
d. Mucostatic


835. Female patient came after days of complete denture insertion complaining about dysphasia and nausea while


eating. What is the cause?
a. Under extended of post palatal borders.
b. Increase thickness of buccal flange
c. Overextending PPS

836. Generalized ulceration, body temperatures: 37 Management:
a. CHX mouthwash for two weeks then re evaluation
b. CHX + SRP
c. SRP + two types of Ab


837. Buccal shelf area?
a. primary stress bearing area


838. How many roots in 27?


839. case about a patient with gingival overgrowth asking for the surgical treatment of the overgrowth?


a. Crown lengthening
b. External bevel gingivectomy
c. Internal bevel gingivectomy


840. What will happen when you cement using zinc phosphate?
a. necrosis
b. Sensitivity


841. Pt had trauma to the right side of his face, feels diplopia and redness in the buccal mucosa of the right side
a. Le fort I


b. Le fort II
c. ZMC fracture


842. pt compline of mobility of implant crown pus and bleeding related to implant
a. screw losing
b. failed implant


843. function of periodontal probe?


a. check caries
b. check BOP
c. excavate caries


844. Assistant went to take her 5 minutes lunch break, she was wearing the gown, what should she do?
a. Remove PPE


845. At which stage we pack the acrylic?
a. Dough stage
b. Rubbery stage
c. Sticky stage
d. Stiff stage


846. Management of ulcerations of the lower lip following IANB?
a. Reassurance


847. A picture of PMF upper crown lower natural and asking about what type of interference?
a. working
b. Non working
c. Centric


848. Difference between crown down and step back technique
a. coronal flare and low torsional flexure
b. more tissue removal
c. fewer instrumentation


849. A pic with a scenario where a Pedo pt with high fever, fatigue and lesions on tongue and labial mucosa (I don’t
remember the whole scenario), what’s the diagnosis ?


a. Herpes Labialis
b. Primary herpes
c. Secondary herpes

850. 5 yo with permanent 6, what best describes the molar (bw was provided)
a. Internal resorption
b. Ankylosis
c. External resorption
d. Ectopic eruption
851. 20 years old patient came to dental clinic with multiple radiolucency in mandible. The mother of patient reported
that the patient father did surgical procedure and removed radiolucent lesions. What does the patient have?


a. Cherubism
b. Basal nevus carcinoma

c.


852. Penicillin mechanism of action?
a. Inhibit cell wall peptidoglycan
b. Inhibit beta lactamese

c.
853. patient use betel nut came with severe pain and limited mouth opening what to do?
a. surgery


b. intrlesional steroids
c. split fibrous tissue

d.
854. HBV sign
a. Pale skin
b. Weight loss


c. Cyanosis
d. Yellow skin tone white eyes


855. Upon intraoral examination, the dentist noticed the lower incisor is larger in crown size compared to adjacent
teeth. Radiograph interpretation: two separate pulp horns and one canal


a. Fusion
b. Gemination


856. Patient came to you in the ER complaining from labial fracture of recently placed porcelain fused to metal
crowns replacing the lower anterior, patient has missing lower posteriors and known case of deep bite, what is the


cause of the fracture?
a. occlusal overload
b. Incorrect anterior guide


857. Pedo pt multiple colored restorations, what type of fluoride you will give?(mentioned before)
a. 1100ppm 0.05 fluoride mouth wash


858. Extracted asymptomatic tooth:


a. hazards
b. Infectious
c. Contaminated.


859. A case about the reason behind impacted canine:(mentioned before)
a. bone around the crown
b. Thick tissue
860. A picture of ball attachment, one straight one tilted and asking why the over denture isn’t stable or something
like that:
a. implants are not parallel to each other
b. distortion of the rubber
861. pedo pt missing lower E AND D with permenant molar and incisors which space maintainer to use
LLHA


862. pedo pt with multiple caries and bad oral hygiene
a. neglect
863. Long case about onlay after you check occlusion within something ship i cant remember you found other teeth
are not in same occlusion what you will do
864. Which drug of hypertensive drugs will increse blood pressure if you use LA with epinephrine
a. Nonselective bblocker


865. pt after extraction you prescribed for him amoxicillin , he came after days complain of diarrhea and stomachache


a. Stop drug and prescribe for him floconazole
b. Stop drug and prescribe metroniazole


866. While your preparing access cavity through PFM (picture provided and tooth was broken ) what's your treatment
Depend on options and the pictures (if extraction or post place)
867. Main drawback of using MTA in DPC


868. Composite restoration and remaining thickness of dentin 0.5 choose optimum liner or base

869. disease has striking symptoms of no reaction of gingiva to trauma by inflammation


a. chediak higashi syndrome
b. granulocytosis


870. which muscle affected pt can not close her mouth and jaw shift 7mm to the right
a. medial petrgoid
b. lateral petrgoid


871. mass in post of tongue(give u histo description)
a. CHORISTOMAS
b. peripheral giant cell granuloma
c. ossifing fibroma

872. After you finish from teeth preparation You apply putty index to waxup cast and then do temp to the patient


intraorally
a. Direct
b. Indirect
c. Indirect direct


873. Muscle attach to the capsule
a. Lateral ptyrgiod
b. Medial ptyrgiod


874. What interference is this? Similar pic


a. functional cusp


b. centric
c. protrusive


875. trunk rash and fever and tongue and white tonsil, red and bumby tongue and ask for diagnosis?
a. scarlet fever
b. varicella

876. In which of the following disease accumulation of advanced glycation end products (AGES) in gingiva takes


place?
a. Diabetes mellitus
877. treatment of Exposed bone in pt under bisphosphonate ( they write zoledronate i think instead of zometa ) no
purulent no pain ??


a. Bone resection
b. Mouth wash (stage1)


878. Def. Of Thixotropic material:
a. a liquid that becomes less viscous and more fluid under repeated application of pressure


879. photo of crown on #21 high restoration (higher than occlusion level ) and there is gingival swelling localized to

🔁
the tooth , tooth is tender to percussion, isolated pocket ,what management ?
a. Adjust occlusion
b. Extraction
c. SRP
880. case scenario asking about management of hypertrophic filliform papilla of tongue and pt have gag reflux?


881. Big RL in the mandible and biopsy is planned. What should you do first ?
a. Aspiration
b. AB prescription
c. Bone resection
882. 20 years old female patient referred to periodontist for crown lengthening
related to #43 which has full coverage crown with subgingival margins,2mm keratinized gingiva is present. Which


type of flaps will you do?
a. Coronally positioned flap.
b. Apically positioned flap
883. Safe wrought wire cross section against dislodgement ?
a. Triangular


b. Square
c. Round
d. Half round


884. Dr saw lost pt searching for ER bc his temporary crown fell and he didn't help him wt did the Dr violate?
a. justice
b. nonmaleficence
c. beneficence


885. RPD components that give stability .
a. Reciprocal arm and minor connector
b. Retentive arm and minor connector
c. Plate and retentive arm
d. Rest and minor


886. Dentoalveolar fracture splint time in weeks


a. 2
b. 4
c. 6
d. 8


887. Pericoronitis pt and in scenario 3rd molar in correct position ask about management ?
a. Operculectomy
b. Other options are with extraction

888. restorable tooth need RCT and good prognosis but the pt and his mother refuse they want extract the tooth
a. take their informed consent and let them sign and go with their prefer ttt
This Q repeated and there’s option of getting other opinions from his father
889. Test needed with pt with warfarin
a. INR


890. Most toxic material ?


a. Ferrec sulfate
b. Formocresol
c. MTA
d. Calcium hydroxide


891. Pt With canine space infection and suddenly have a lot of happening and (tachycardia )
a. cavernous sinus thrombosis


892. Case Scenario pt have complaint of sensitivity in her lower anterior teeth during examination there is not enough
keratinized tissue and high frenum attachment , management ?


a. Frenectomy + CTG
b. Frenectomy +FGG
c. CTG
d. FGGk

e.
893. Female pt Scenario of bad habits scratching her gingiva by her nails , in the photo there is recession in #33 and
root caries asking what 1st treatment after habit stop ?
a. GIC restoration
894. how many embrasure between two adjacent teeth ?


a. 1
b. 2 (b/w tow teeth
c. 3
d. 4(if they say one tooth


895. Recession or Crowns i can’t remember and Food impaction under contact area best way to brush this area ?
a. Interdental brush
b. Dental floss


c. Dental brush
d. Super floss


896. Gen. Brown teeth discoloration and ubon ultra violet fluorescence is yellowish discoloration?
a. Dentenogenesis imperfecta
b. Amelogénesis imperfecta


c. Fluorosis
d. Tetracycline

897. very small pedo pt ingested tooth paste and his parent get him to clinic ?


a. Hospitalization
b. Give milk
c. Reassure his parents and give them instructions

🔁
898. Mouthwash containing High level of alcohol may cause
a. Lichen planus
b. Leokoedema
c. Stomatitis


899. CD prosthesis , pt sound Th instead of S sound what mistake ?
a. Upper ant. Teeth placed palatally
b. Reduced vertical dimention


900. What tipe of GIC is Luting agent
a. 1
b. 2
c. 3

901. Radiograph with RL in molar just between the roots ,
a. Interdental bone loss


b. Vertical bone loss
c. Interradicular bone loss
d. Horizontal bone loss
902. what is the minimum space for cemented crown implant for central incisor :


a. 5-6
b. 7-8
c. 9-10


903. edentulous patient want to make CD , the Maxillary tuberosity touch the retromolar area , what the management
a. surgical reduction of tuberosity
b. surgical reduction retromolar


904. what is the most common used for systemic fluoride:


a. toothpaste
b. water
c. mouthwash


905. year old woman complains of deposits on her teeth. Examination revealed yellowishcolored soft deposits which
could not be easily dislodged with a water spray. Which of thefollowing best describes the patient's complaint?


a. Pellicle
b. Plaque
c. Calculus
d. Material alba

906. picture of curret without and labeling and ask about the spacifc area used in (according to pictures)
a. mesia post
b. dista post
c. facil
d. lingual

907. Flasking of cast done by?


a. plaste
b. die stone
c. refectory something

908. Rheumatoid Arthritis pt came for fullquadrant treatment. How will you seat the patient?


a. Supine, bite block, rubber dam
b. However makes the pt comfortable
c. Supine with bite block only

909. Patient has lost all of his teeth posterior to 45 and 35 (Kennedy class 1) the undercut was not in the cervical part
and infrabulge is contraindicated which of the following would you do?
a. Mesial rest and distal i bar
b. Distal rest and mesial i bar


c. Distal rest and mesial circumferential
d. Mesial rest and distal circumferential
910. picture of tongue with red patch on dorsum of tongue surrounded by white border


a. erythroplakia
b. erythro something (I think they mean erythroleukoplakia)
c. lichen planus
d. lecnoid something


911. pt using topical corticosteroids on buccal side to heal the disease 90% in 2 months but in the last month it cause
this (showing picture)


a. chemical burn
b. candida
c. ichen planus

912. the case of rash in face and trunk on child


a. herpes zoster
b. varicella something (I think they mean VARICELLA CHICKENPOX)


913. : Xray of pedo with trauma and has horizontal root fracture of both primary incisors and said tooth had mobility,
the roots touched the permanent tooth bud, what is the management?


a. Observe and radiographic follow up
b. Extract coronal segment and leave apical
c. Extract both segments
d. Splint


914. 2/3 crown fractures and pulpal response was normal. What is best management?
a. Composite build up
b. Elective endo and post and core and crown
915. What is the maximum length that the curette can achieve in non non-surgical department?
a. 2.75


b. 3.75
c. 4.75
d. 5.75
916. Tooth which most commonly prone to caries ?


a. maxillary first molar
b. mandibular first molar
917. Amentia pt 80 yrs oldHe has no relative addmitted to Er has sever pain on 37 the dentist has two option
extraction or rct who will take the decision;
a. Judge


b. Hospital manager
c. Dentist


918. Dentist newly hired has only two dose of hepatitis and the hospital doesn’t request the vaccination record, they


violet:
a. Environmental preventative community
b. Staff health


919. At the end of the treatment of pedo pt the doctor want to reinforce the desire behaviour :
a. Positive reinforcement


920. For pedo pt to prevent injury something like he doesn't mention procedure what to use


a. Rabber dam
b. Bite block

921. Extracted amalgam where to discarded:


a. Infectious
b. Hazaroud


922. Dentist and needle stick he injured himself during session
a. Encourage bleeding and water for several minutes


923. Instrument kit with blue color indication
a. Return it cssd
b. Report
c. Use them


924. Alginate will pour it after one hour but the dentist kept wet towel on it what will happen
a. Imbibition


925. What is the healthiest finsh line for periodontal :
a. 0.5 supragingival
b. 1 supragnigival
c. 1.5 subrangigival
d. 2 subra gingival
926. Pedo 15 questions about appliance not common like ( blue grass thump coverage nance herpest ) also case
what is the appliance thar will use .
927. Patient had wire and multiple ulcer in mouth and tongue and mucosa and the next appointment with his Dr after
2 week what is the management


a. cut wire
b. Wax
c. remove the appliance
d. antibiotic


928. 7 mesiodistal space and you want to put the largest implant to fill this soace the implant will be :
a. 4
b. 5
c. 6
d. 7


929. Pic of implant very smooth surface aske about name
a. Coping for open tray


930. Cervico Occlusal molar length (Mentioned Before)
a. 7.5mm
931. Amalgam with spherical?

932. Strong phase ( gamma )?


Gamma is the strongest, Gamma 1 is less strong, Gamma 2 is the weakest.


933. Drawback of acrylic resin temporary crown is
a. Shrinking of the temporary


934. ESRD (end stage renal disease) oral finding ? (Mentioned Before)
a. Petechiae


935. case with the following details : Ferule 0.5 mm Reminding GP 5 mm Post length 2/3 or ⅓ And the Crown


fractured what’s the cause ?
a. Insufficient ferrule effect
b. insufficient GP
c. insufficient post length


936. case about Soreness on ridge and slopes of denture
a. hyper occlusal
b. candida (this also can be correct, please read below)
It’s important to know the full case to know the answer, as the information is very limited here, both answers
could be correct:
Hyper occlusal → If the soreness on the ridge of the denture is associated with irritation or pressure points.
Candida → If there are signs of inflammation, redness, or white patches on the oral mucosa.
But, option A (Hyper occlusal) may be more fitting as an answer because it is concerning ridgies and slopes,
while candida may be more fitting with the denture base.


937. Immediately noticed the disadvantage of increasing VD ?
a. Clattering of teeth
b. Old age appearance
c. Approximation of nose and chin
d. TMJ pain

938. Diabetic pt hba1C 9 or 8.5 and 50% Horizontal bone loss? Proper management?


a. refer to doctor to manage diabetes
b. SRP and doxy to reduce effect of diabetes
c. antibiotic only

939. irrigation Prevent contamination in surgery
a. Hydrogen peroxide
b. Tap water


c. Chx ?
d. Saline

940. resilience definition

a.

b.


941. SNB in class ii :
a. Increased
b. Decrease
c. Decrease then increased
d. Normal
942. medication given to pregnant to prevent causing cleft palate ?
a. analgesics


b. antibiotic
c. folic acid
943. scenario of trauma to athletic lost his upper #11, which type of implant is indicated ? pictures choose one :


a. tissue level implant
b. bone level implant
c. external hex implant
d. very wide implant


944. Patient having edge to edge occlusion which type of restoration ?
a. Full crown
b. Composite veneer
c. Porceline veneer
945. ortho appliance for young pt who have class II collusion and retroclined mandibular incisor ?
a. Headgear


b. Frankle
c. Twin block
946. Implant size 5mm in lateral incisor and there is bleeding. What is the case ?
a. No enough space from adjacent teeth
947. What type of collagen in pulp during development ?
a. type 1


b. Type 2
c. Type 3
d. Type 4
948. Pt have asymptomatic broken tooth , history of HIV , proper management ?
a. defer


b. Refer to infectious disease specialist
c. ask for new lab test


949. 2 years , having high carious tooth with radiolucency , what anesthesia you will choose ?
a. Lidocaine 2%
b. Lidocaine4%
c. Articaine2%
d. Atticaine4%
950. Case of pedo pt having thumb sucking habit what appliance you will choose?


a. W arch
b. Palatal crib
c. Hass


951. Case of pedo patient 7 years hve thumb sucking habit , what is the proper management? ‫االهل جربوا معه الحلول كلها‬


‫وفشلت‬
a. Fixed functional appliance


952. You will reduce from mylohyoid ridge which nerve you may injure ?
a. Lingual
b. Mylohyoid
c. Inferior alveloar


953. A patient requests extraction of all his teeth, despite there being no major problems and the possibility of repair.
How should you manage this case?


a. Extract all the teeth.
b. Refer to a psychologist.


954. A patient insists on having veneers for life. How do you address this request?
a. Explain why this is not a good idea.
b. Inform her that veneers may only last for about 5 years.


955. A patient with Down syndrome presents to the emergency department with a critical infection. From whom


should consent be obtained?(mentioned before)
a. The doctor.
b. The family.
c. The proxy.
956. A diabetic patient is developing a case of Ludwig's angina. What is the appropriate medical intervention?
a. Administer antibiotics.


b. Extract the tooth and perform incision and drainage.
c. Immediate transfer to the hospital.


957. What is the first sign that might indicate a patient has an underlying health issue?


a. Unexplained bleeding in the gums.
b. Hairy tongue with halitosis.


958. A patient experiences bleeding, but all their medical tests are normal. What could be the potential diagnosis?
a. Scurvy.
b. Hemophilia.
c. EhlersDanlos syndrome.
d. Von Willebrand disease.
959. A patient complains of mouth ulcers, eye problems, and recently developed skin ulcers. What could be the


possible diagnosis?
a. Behçet's disease.
b. Erythema multiforme.


960. A patient experiences pain in the right temporomandibular joint, with no clicking or trismus, but has restricted


movement to the left side and deviation to the right side. What could be the diagnosis?
a. Myofascial pain dysfunction syndrome.
b. Disc displacement without reduction.
c. Disc displacement with reduction

961. Pic of lateral incisor that fractured at gingival level + x ray showing long root , what is the best tx ?
a. extraction
b. Post , core, and crown


c. Gingivectomy
d. Extrusion + osseous


962. a 2 years child ingests toothpaste. What is the management ? ( depend on the amount )
a. advice to drink milk ( first step )
b. Induce vomitng


963. Propping depth of lower canine 5 mm , pt is medically free , no bleeding or sign of gingival inflammation ,
attached gingiva 4mm
a. Deficient attached


b. Healthy periodontium
c. Gingival overgrowth ( fibrotic type )


964. Palatal brown pigmentation 5mm , well defined , not changed in size since 3 months what the management ?
a. cryotherapy


b. electrosurgery
c. excisional biopsy

965. Pt having small amalgam restoration in #16 , need to be changed what you will choose ?


a. gold
b. Composite
c. Cast ceramic
d. Glass ionomer


966. Bulging band in orthodontic patient causing ulceration what to do:
a. Wax
b. Cut
c. Wait for orthodontic app


967. Patient has ulcerated in her lips and the disappear after for days( pic of ulcers on the lower lips
a. Recurrent herpetic ulcer
b. primary herpes ulcer


968. patient has rashes only in one side of his body and his face itches what virus caused it :
a. CMV


b. Herpes simplex
c. Herpes zoster


969. Removes and stops the growth on inanimate surfaces:
a. Disinfection
b. Seterlization
970. Patient has prosthetic valve and want to perform a procedure and to prophylaxis antibiotic , the other day and
came back to the clinic with hoarseness voice and swelling what to give him :
a. Epinephrine


971. Electric pulp testing tests what ?
a. doesn’t indicate the severity of the pulp inflammation only indicates the vitality


972. Patient doesn’t want to hear the side effects of her procedure she’s medically fit and refused and wouldn’t listen


to the doctor ?
a. Take the consent form her and tell her the side effect
b. Take the consent form her son and tell him the side effect


973. Patient is has some mental disability not fully and came to you to asses his mental capacity, form who do you


take the consent from ?
a. Parents
b. Doctors
c. Him


974. Patient admitted to the hospital with covid signs and the doctor didn’t inform the staff of the patient condition
what did he violate:


a. Infection control
b. Community
c. Collegues


975. What the fourth dental material aside from polymers, metal ,porcelain :
a. Stone


b. Cement
c. Composite
d. Alginate


976. Accidentally got blood and something on your hand what to do:
a. Wash with soap and water then alcohol
b. Alcohol scrubbing material only
c. Wash with soap only


977. Adjusting the try in of upper molar and it was noticed that it glides superior anteriorly :
a. Distal inclination of the mandible


b. Lingual inclination of the mandible
c. Mesial inclination of the upper
978. Patient came with white line on the side of her tongue asymptomatic what to do :
a. Incisional biopsy
b. Excisional biopsy
c. Topical steroid
979. Patient pedo came with her parent and said she ingested 50mg of fluoride :
a. Just observe
b. Give milk
c. Admit to hospital


980. Patient has kennedy class ii and want to use the abutment that was mesially tilted what to use:
a. Ring
b. Combination
c. RPI
981. Linear line bilaterally on the facial cervical third :


a. Abfraction
b. Abrasion
● * Abrasion: linear in buccal -cervical surface , * Abfraction: V shape in buccal cervical surface
982. Patient with class iii malocclusion :
a. Extract lower 1st premolar


b. Extract upper 1st premolar and lower 1st premolar
c. Extract upper 2nd premolar and lower 1st premolar


983. Patient came to the clinic that had ortho TX and took them off due to inflamed gingiva and there were no bone


loss present what to do:
a. Scaling and gingivectomy
984. Patient came the next day with pus and pain and (it looked like dry socket) what to do: (Mentioned before)


a. Antiseptic mouthwash , AB, curettage
b. antiseptic mouthwash,AB, irrigation


985. patient came with clicking in his TMJ:
a. with reduction
b. without reduction


986. patient suffered from night sweats, sudden weight loss, fatigue :
a. irregular widening of PDL
b. expansion of the bone
● *Signs in the question indicate malignancy. The malignant cells metastasis causes the irregular
widening of the PDL.


987. Histological there was C3,and prickle cell:
a. Pemphigus
● *Antibodies ( IgG, igM) and C3 , and it’s intercellular. Regarding the prickle cells:


988. Patient started to convulse and later lost consciousness what to do :(Mentioned before)


a. Semi supine
b. Supine
c. reclined
d. Semi reclined.


989. Patient edentulous came and stated that his retromolar pad touches the upper hamular notch what to do:
a. Increase the vertical dimension
b. Extend the metal to the retromolar pad ✅
● *adjusting the denture by covering all retromolar pad rather than making significant changes to the
vertical dimension.


990. Case of fibromyalgia how to diagnose it:


a. Test regarding muscle movement
b. By exclusion


991. Lesion on x ray that shows only white not super calcified with no RL rim what is it ?
a. Osteoma

992. Patient want to replace tooth 14 with implant there was 4mm interarch distance what to do:


a. Return the patient to the surgeon to adjust
b. Place implant screw retained implant


993. Mother came with her child with interproximal carious lesion on the lower molars , the mother stated that she
has her daughter on a very strict diet :


a. SSC
b. Adhesive resin restoration
994. Patient came with complicated fracture 3hr ago with open apex tooth what to do:
a. Pulpectomy


b. RCT
c. DPC


995. Dentist removed his gown and face shield how to remove the mask :


a. perform hand hygiene then removes the mask
b. remove the mask with the band around the ear


996. Doctor wants to reduce the croma:
a. Add violet


997. Patient came with something forgot but he's taking 325mg of aspirin what to do:


a. stop for 23 days
b. Extract


998. Most common tooth that’s missing:
a. Lateral maxilla
b. Lateral mandible
c. 1st molar of upper
d. 1st molar of lower

999. Patient placed to implant the distal one is failing what to do:
a. Removing the distal implant
b. Placing vertical bone matrix


1000. What the number of visits in implant patient the 1st year:
a. Every 3 months


1001. Pregnant patient came she's on her 2nd trimester , she has 2 teeth that there was severe pain radiating from them:


a. Single x ray
b. Start treating the 2 teeth In effort to relieve the pain


1002. What’s the thing that’s attached for Complete over denture:
a. Locator
b. Implant
c. Analog
d. Coping


1003. Most common occlusion ?
a. Class I
b. Class II


c. Class III
d. Class I with crowding


1004. pt want to to fixed crown and dr reduce the following, margin Supra gingival 0.5 Inter arch 1.5, occlusal gingival
axial wall 2mm ,What is violet?


a. marginal integrity
b. resistance and retention
c. tooth reduction


1005. 9 Y/O patient brought to clinic by her parents and they are worried about her orthodontic treatment , she has
crowding and impacted upper canine and maxillary deficiency and mandibular prognathism, what is the most thing


you will be worried about it in this time:
a. Maxillary deficiency.
b. Mandibular prognathism.
c. Crowding.
d. Canine impaction.

1006. 15 Y/O patient complaining of diastema, on x ray there is no mesodent, what is the most likely cause for
diastema (no other informations):


a. Open bite.
b. Low frenal attachment.
c. Normal with growth.


1007. 62 Y/O male patient while examination you found posterior recession and root caries that is arrested, what is the
best management:
a. Topical fluoride.
b. Systemic fluoride.


c. Composite restoration.
d. Glass ionomer cement restoration.


1008. 35 Y/O female patient received recent partial denture and came complaining from redness and erythema in the
palate ( i’m not sure if he said swelling), you take incisional biopsy and you found granulation tissue and


accumulation of blood vessels, what is the most likely diagnosis:
a. Peripheral giant cell granuloma
b. Cellular hemangioma.
c. Pyogenic granuloma


1009. What is the required distance from interproximal implant to crest of bone to preserve a healthy papilla:
a. 3.4mm.
b. 1.8mm.


1010. 4 years patient and down syndrome needs fluoride, what is the best method for applying fluoride:
a. 10% stannous fluoride solution.
b. 2% sodium fluoride gel.


c. 1.23 Acidulated phosphate fluoride foam.
d. 5% sodium fluoride varnish (not sure about concentration but it is varnish).
1011. 50 y/o patient with a history of coronary heart disease and hypertension, came with symptoms of dyspnea,
orthopnea and paroxysmal nocturnal dyspnea. Clinical signs you found jaundice and clubbing of fingers, what is the
most likely condition cause that:
a. Liver disease.
b. Hypertension.


c. Coronary heart disease.
d. Congestive heart failure.


1012. How can you locate posterior palatal seal on the cast:
a. Wards carver.


b. Le cone carver.
c. Kingsley scraper.
d. Surgical blade.


1013. What is the most critical thing to consider for a child with premature primary second molar loss:
a. Space maintenance

● .
1014. During metal try in for a patient the patient said “ i feel a seeds stuck between my teeth “What is the most likely
cause of that:


a. High occlusal contact.
b. Tight proximal contact


1015. Patient with squamous cell carcinoma what will you do as an adjunctive to surgical intervention:


a. Immunotherapy.
b. Radiotherapy.


1016. Patient have crown in tooth #26 and there is a protrusive interference, how can you manage it:
a. Mesial inclination of upper.
b. Buccal inclination of upper.
c. Lingual inclination of upper.
d. Buccal inclination of lower


1017. 35 Y/O female patient with lower RPD replace the posterior teeth in quadrant 3 which opposing natural teeth,
she complain from interference when closing to the optimum condyle on the glenoid fossa, what is the type of


interference:
a. Centric
b. Protrusive.
c. Working.
d. Non-working.


1018. Patient came and complain from pain and swelling in the area between #21 and #11, on radiographic


examination you find a unilocular 6mm radiolucency between 21 and 11, what is the most likely diagnosis:
a. Nasopalatine cyst.
b. Periodontal abscess.


1019. Pt had a car accident and came to ER with body and dental injury and difficulty in breath his vital sign is noraml
what will be your management?
a. chest x ray


b. tracheotomy
c. jaw thrust
d. head left and chin


1020. What is the step before sending the tray to autoclave
a. scrub with brush


b. manually clean with water
c. thermodynamic cleaning
d. soak in glutaraldehyde

1021. Pedo pt limited mouth opening , no deviation , asymmetrical lower jaw to right side, Tx? ( on OPG there was
susceptible Condylar ankylosis )


a. Arthroplasty
b. Jaw exercise ‫اول شي‬
c. Anterior bite plane
1022. patient have car accident and loss 3 lower incisor, came to replacement, you are uncertain if it was #32, what


anatomical morphology to confirm?
a. Incisal slope towards distal
b. Insical at right angle
1023. pt with 4 anterior missing upper with severe ridge resorption best esthetic prostho tx
a. Implant
b. FPD
c. RPD
1024. patient has thin gingival phenotype of prepared tooth , what should you do before taking impression ?
a. do gingivectomy
b. do crown lengthening

c. trough with laser for gingival retraction
d. Use smaller core


1025. The junction of the mandibular minor connector with major connector is.
a. butt type joint
b. acrylic forms the joint
c. ball and socket
d. there is no joint


1026. What is the sequence of conventional serial extraction:
a. Primary canines, primary molars only.

✅ (BCD4)
b. Primary canines, primary molars , second premolars.
c. Primary lateral incisor, primary canines, primary molars, first premolars.
d. Primary laterals, Primary canines, second premolars.


1027. 60 Y/O male patient came for clinical check up and you take a radiograph, you find that there is a radiographic
image was taken when he is 25 Y/O , while comparing two radiographs what is the most likely difference you will
find:


a. Increase thickness of alveolar bone.
b. Increased cementum deposition.
1028. 15 Y/O patient came with pain and radiolucency under tooth #46 after examination you decide to do root canal
treatment and preserve the tooth, his mother was not sure about the decision of treatment and she ask her son , he
requested extraction of the tooth, what is the appropriate management in this case:


a. Do the desire of them and take informed consent from the mother.
b. Ask for the permission of father and mother then do either extraction or RCT.
c. Refuse the extraction.
1029. 6 months child came with eruption cyst, what is the management:


a. Surgical incision.
b. Follow up.
1030. patient on inhalation corticosteroid came to the clinic (he provide the vital signs ) everything was normal and the
respiratory rate improvement during rest,
1031. (The oxygen saturation was 98%, and all other vitals normal), what is the most likely diagnosis for him:


a. Emphysema.
b. Asthma.
c. Chronic bronchitis.

1032. Healthy patient smoke 1 pac per day since several years suffers from deep pockets (there wasn’t any other
disease), he needs topical antibiotic, which antibiotic you will give him:


a. Metronidazole gel.
b. Doxycycline gel .
c. Minocycline gel.
d. Tetracycline fibers.


1033. psychiatrist refer a patient to you and she complains from swelling in parotid gland which one of the following
psychiatric conditions can contribute to it:
a. Schizophrenia.


b. Bipolar.
c. Anorexia nervosa.
d. Attention deficit hyperactivity disorder.


1034. patient complain of fibromyalgia, what is the most accurate method to diagnose it:
a. Diagnosis by exclude. ( eliminate any disease with dissimilar symptoms )
b. Electric muscle test.
c. Incisional biopsy.


1035. Female patient came to you and ask about the anesthesia and if it will affect her fasting:
a. Say “ i don’t know “.


b. Send her to health director.
c. Send her for asking religion affairs in the hospital.
d. Ask her not to exaggerate things.

1036. During crown lengthening procedure the dentist ask the assistant for blade, what is the appropriate method to
give him:


a. By assistant dominant hand.
b. Put it in neutral zone.
c. Put it in the working zone.
d. By hand and the blade away from the dentist.


1037. Slow mixing cement to reach the consistency
a. Resin
b. GIC


c. Polycarboxylate
d. Zinc Phosphate


1038. How long should you etch enamel for veneer cementation in Seconds:


a. 30
b. 20
c. 15
d. 40


1039. Patient complained of metallic taste and pain, He cemented a cast post and crown recently, no mobility or RL
detected, what is the cause:


a. Eccentric occlusal interference
b. Mobile crown
c. Vertical root fracture


1040. Patient with “diffuse” redness after ortho:
a. allergy from NiTi
b. Extended wire


1041. Def of Bioethics


A. This is the division of applied ethics that helps in defining, analyzing, and resolving ethical issues that
arise from the provision of health care or the conduct of healthrelated research.
1042. Def of Clinical (medical) ethics

A. This is the branch of bioethics that is related to the identification, analysis, and resolution of moral
issues that arise in the health care of individual patients.
1043. A physician in a local hospital agreed to be an investigator of a multicenter international trial sponsored by a
pharmaceutical company. He regularly completed and sent CRFs to the sponsor overseas. On one occasion, the
sponsor questioned the data submitted and insisted that the original patient‟s chart be shipped to him for inspection


and verification.
A. confidentiality.


1044. What instrument to use to remove calculus in the image


a. Hoe


b. File
c. Curette
d. Chisel


1045. Which major connector of choice for palatal torus
a. U shape
b. anterior stap
c. posterior strap
d. Anterior posterior palatal strap
If large Tori’s U shape if smal Anterior posterior palatal strap


1046. a doctor she cut her finger while she make dinner in thursday and she go to the clinic on sunday to check her
patient appointment

b. Use alcohol ✅
a. Cover the wound

note:No need to cover the wound cuz more than 24 hours. Just disinfecting
1047. Angle of blade in cutting instrument which number( did not specify 4 or 3 numbers formula)
a. 3

b. 4 ( If they did not specify select as if it’s 4 numbers formula )

1048. Pt limited mouth opening with vertical lines on buccal mucosa


a. tobacco smoking
b. Betel


1049. you have a deep cavity and want to preserve the health of pulp, what approach would you take? Would you start
removing the decay from:
a.
b.

The axial wall first, then the floor.
The floor first, then the axial wall.

1050. A patient presents with a picture of a swollen lip, stating that it has been like this since childhood and now seeks
treatment
a. Hemangioma
b. persistent arterial issue ✅


1051. functional impression in CD
a. Alginate
b. Impression compound
c. PVS

d. ZOE
1052. Pt complaining of difficulty in swallowing, ulceration and erythematous in non keratinised tissue. She had also
had a recent hematopoietic cell transplantation. What is the best treatment?
a. Antibiotic


b. Antifungal mouthwash
c. Corticosteroids mouthwash


1053. Fluoride Varnish concentration?
a. Sodium Fluoride 5%
b. Stannous Fluoride 0.05%
c.
1054. Pt came complaining that he is not comfortable with his old denture because when he where it he get “aged
appearance associated with wrinkles in the corner of her mouth”. Pt is requesting a new complete denture. What will
be your management?

🔁
a. To open up the freeway space
b. To close the freeway space
1055. Pt came complaining of pain related to #38, after examination of the tooth it was very close to IANB and tooth

🔁
#37 has root resorption. How to manage the case?
a. Extraction of #38
b. Extraction of #37
1056. What indicated periodontal disease progression?


a. Bleeding on Probing
b. increased CAL or when pockets get deeper
1057. Pt is complaining from gingival bleeding that happened after the dentist did gingivectomy by laser. What


happened?
a. violation of biological width
1058. Pt complained from his anterior 6 crowns. The gingiva is bleeding and painful. How to manage?


a. surgical crown lengthening
b. shallow margins of crowns


1059. Radiograph of supra gingival calculus. What instrument to use for scaling?
a. Scaler


1060. Treatment used for 8 years old pt who has retrognathic mandible?
a. Functional appliance


1061. Female pt has bulla. After taking a biopsy you found intraepithelial separation.
a. immunofluorescence testing


1062. Indicates the presence of intercellular autoantibodies of IgG, C3 . What is the diagnosis?
a. pemphigus vulgaris
b. Bullosa epidermolysis
1063. Tooth has an isolated wide pocket. Cold test indicates that the tooth is necrotic. Tooth is sensitive to percussion.
In the Radiograph tooth is restored and there is large radiolucency associated with the distal root. What is the


diagnosis?
a. primary endo secondary perio
b. primary perio secondary endo


1064. same as question 19 but asking about the management?
a. RCT then Perio tx
b. RCT only
c. Perio then RCT
d. Perio only
1065. What is the purpose of a liner in a casting ring?


a. prevent shrinkage
b. Allow uniform expansion

1066. while the use of surveyor, the Finish line of height of contour varies in width and position during surveying of
cast, why?
a. Poor Quality of carbon
b. Wrong angulation of the cast ·


c. Tip of carbon touching to sides
d. Carbon markers need resharpening
1067. Patient with a history of breast cancer is taking bisphosphonate and needs to extract 3 teeth. What's the best
treatment?
a. extract like normal
b. extract all teeth in the same visit


c. extract 1 tooth each visit with 2 weeks in between
d. extract quadrant wise with 2 months in between
1068. When the dentist did tooth preparation, Patient experienced pain even on the enamel layer this may be due to?


a. Enamel tufts
b. Enamel spindle
c. Enamel Lamellae


1069. pt wants to place implant in tooth #21. Mesiodistal space is 6.5. What is the implant diameter you'll place?
a. 3.1
b. 4


1070. Which of the following is associated with xerostomia?
a. Antihistamine

1071. Known case of HIV came for treatment of asymptomatic small fracture of tooth, what is the appropriate


management:
a. Ask for CD4 count and lymphocyte and viral load.
b. Refer to infectious disease specialists.
c. Double gloves and face shield.


1072. Picture of caliber persistent artery on upper lip and ask about the source of that:
a. Vascular
b. Spacious.

1073. Long scenarios of diabetic and hypertensive pt and take corticosteroids for other disease ( I don’t remember it ) ,
what is the effect of long term use of corticosteroids on him:


a. Increased Blood pressure.
b. Increased HBA1c.

1074. Pt after fixed appliance placing complaining of sever laceration & ulceration, on examination there is excess wire


is the cause:
a. Clip the wire
b. Put gauz
c. Put wax around it
d. Assure the pt & no treatment

e.
1075. xray of tooth #24 with metal crown and after examination Response to EPT but not cold?


a. Normal
b. necrotic (false positive response to EPT)
c. Irreversible
d. reversible
1076. Patient with Hemophilia extraction was done bleeding would not stop even with (he did procedure it didn’t stop)
is the management:
a. vit K aminocaproic acid


b. Frozen fresh plasma
c. Factor Transfusion


1077. autoclave 126
a. 10 psi


b. 15 psi
c. 20 psi
d. 25 psi

1078. Which finding will shift the diagnosis from bell's palsy to ramsay hunt
a. acute otitis media
b. nerve paralysis
c. herps ear infection ✅


1079.

Pt came complaining of diffuse redness and pain in left buccal mucosa she installed the ortho 6 weeks ago
a. Allergy to niti
b. Extended
c. wire Trauma
1080. Missing 17-14 , 34-37 and 44-47 all third molars are missing how should you record occlusion
a. Arbitrary mounting with CR
b. Bite block and face bow with CR
c. Arbitrary mounting with MIC

d. Bite block with MIC
1081.

What is the best way to clean your hand after a procedure?
a. Regular soap only
b. Alcohol only
c. Wash with soap and water then alcohol based hand rub
1082. What is the importance of the posterior palatal seal?

b. Improve stability.
c. Decrease gag reflex.

a. Increased durability of the denture


1083. First Ortho screening age?
a. 7


1084. How to store biopsy before sending it to the lab?
a. Formalin


1085. Extracted tooth with amalgam restoration . What container?
a. Biohazardous waste


1086. Dr with contact lens got a splash in his eyes. What to do first?
a. Remove lens


1087. Antibiotic prophylaxis for child with heart disease?
a. Amoxicillin 50mg

1088. After 6 weeks of ortho braces pt complains of diffuse redness and pain in the left cheek
a. Trauma


b. Extended wire
c. Nickel allergy


1089. Pic of white lesion and case about heavy smoker, he smokes 1 pack daily for 25 yrs what is the lesion?
a. Leukoplakia

1090. Chemo pt had his first cycle 18 days ago and he needs extraction. Lab results were all below normal what to do?
a. Postpone treatment after 2nd cycle with new lab test
b. Wait 3 days and request another lab test
1091. Pt was complaining of her smile and shy about it. After the appointment the assistant talked about it with her


colleagues. What violation ? (No confidentiality in options)
a. Privacy

1092. Pt was complaining of his CD that he received 3 weeks ago. When pronouncing S it sounds like th what is the
problem?
a. Upper incisors set too far palatally
1093. RPD #32 - 42 and pt lost #33 what is the ideal management
a. Implant with crown


b. Implant with locator
c. Add canine with wire


1094. Pt with gingival bleeding mobility
a. Scurvy

1095. 11 yr with avulsion of #11 more than 60 min. What to do


a. Place the tooth in stannous fluoride
b. Place the tooth in sodium fluoride
c. Place the root in sodium hypochlorite then fluoride

1096. Case about kidney transplant 2 months ago and pt need treatment
a. Postpone tx for 6 months after transplant


1097. Dr could not place rd in broken tooth what to do? Radiograph of R.R #15
a. Place it on #16 and stretch it to #14
b. Cotton roll isolation
c. No need to place rd


1098. Pic of bone resorption because of overhang resto and ask about the etiological factor?
a. Plaque

1099. Lab test needed for HCV pt before treatment


1100. Management of primary herpatic stomatitis?


a. Systemic antibiotics
b. Antiviral and home care

1101. Pic of hand vehicles and it mentioned that the pt have the same in his feet and mouth
a. Hand foot mouth disease
b. Chicken pox


1102. Gummy smile, 9 mm teeth show and normal lip what is the management?
a. Lefort impaction


1103. Why is it recommended to use a cool glass slab while mixing zinc oxide?
a. Shorten setting time
b. Increase solubility


1104. Severe retrognathia glossoptosis and cleft lip
a. Pierre robin


1105. 70 years pt come for treatment then get tremors bradykinesia what diagnosis


1106. pedo pt 6 years low caries risk good oral hygiene and carbohydrates fluoride supplements
a. 0✅
b. 0.25
c. 0.5
d. 1.0


1107. Pedo 3 years with white patches
a. high Caries risk
b. Low
c. moderate risk


1108. cause of cleft lip
a. medial nasal and maxillary
b. lateral nasal and maxillary
c. medial nasal
d. lateral nasal


1109. who is focus toward clinical system
a. Ptc
b. Doctor
c. family pt


1110. pic of pt with bone exposure and asymptomatic
a. mouthwash and antibiotics
b. surgical department

1111. pt with blood borne infection how to ttt


a. Treat under special precaution and infection control
b. treat as normal and universal precaution


1112. pt trismus after multiple injections of local anaesthesia which space


1113. pulmicort inhaler use in which respiratory disease


1114. pt with new complete denture well done but he feel the denture will fall down
a. chapmer suction
b. section bumper


c. magnetic
d. Adhesion
1115. diabetic pt feel dizzy after set in dental chair he take daily dose of insulin and appointment in the morning


a. insulin shock
b. Vasovagal
1116. pic where to put indirect retainer in class I cast


1117. Atrophy of vermilion border


a. surgical removal
b. Laser

1118. Clamp use in class v
a. A12
b. A13


c. W8
d. 212


1119. osteomyelitis appear as
a. expansion of bone


b. root resorption
c. Sequestr


1120. number of blade angle in gv


1121. revascularization of immature teeth which material put in clot immediately
a. Mta
b. Tribiotic


1122. extremely anxious asthmatic patient allergic to diazepam. How to do tx?
a. LA


b. GA
c. nitrous oxide
d. allergy test for another type of benzo


1123. diabetic patient on sulfonylurea came to you for dental treatment, you gave him ibuprofen. he called you after tx
later that night with weakness, increased heart rate? (not sure but something with his heart beats) and numbness.
what's the possible situation?
a. hyperglycemia
b. drug intereaction ✅
c. forgot other two


1124. patient with addison’s and liver cirrhosis coming for minor oral procedure. vitals WNL. steroid
supplementation?
a. no need


b. double dose
c. 25 mg hydrocortisone
d. 5075 mg hydrocortisone


1125. case of skin lesions and perioral crustings and oral ulcers.
a. erythema multiforme
b. lichen planus
c. pemphigus vulgaris


1126. a patient with uncontrolled diabetic presented with a case of fast spreading odontogenic infection with purple


skin.
a. necrotizing fasciitis


1127. healthy gingiva histo?
a. mild lymphocyte
b. abundant PMNs
c. plasma cell
d. no inflammatory cells at all.


1128. which type of occlusion is a contraindication of anterior bite plane?


a. normal
b. open bite
c. deep bite
d. crossbite
1129. 9 yo with insufficient arch size in mandible, which tooth is most likely to be malpositioned?
a. Lateral (closest answer?)
b. Canine (not erupted in 9yrs old)


c. first prem
d. second prem(just erupted or maybe not in 9yrs old)

1130. garre’s osteomyelitis asking about the first treatment step?


a. ab
b. ⁠rct

1131. patient with excised adrenal glands became subconscious in dental clinic. no signs of apprehension were
mentioned. Treatment?
a. atropine


b. thyrotoxin
c. hydrocortisone

1132. arch size discrepancy with amount of tooth structure. what diagnostic information is relevant?
a. pan
b. taking photographs


c. dental hx
d. study cast (bolton analysis)


1133. case of bridge 34x36. Patient complaining of malodor. Bubbles under retainer of 34 found. cause?
a. space between abutment and retainer
b. food impingement under pontic
c. fractured connector
d. fractured abutment or retainer (cant remember)

1134. 9 yo with insufficient arch size in mandible, which tooth is most likely to be malpositioned?
a. lateral
b. canine
c. first prem
d. second prem

1135. Which clamp is used for the upper molar?


a. W7(universal for both jaw)
b. W8
c. W2
d. W4

1136. Lethal dose of fluoride,patient was 6 years old.


a. 200
b. 300


c. 400
d. 500

1137. You treated #26 with deep caries and in the next day patient comes complaining from numbness. What is the
cause of feeling that?
a. breakage of anaesthesia


b. Injection to maxillary sinus
c. Injection to grater palatine foramen

1138. ANB=6 , SNA= 80 ,SNB= 74 , Upper incisors to SN = 110



a. Class II
b. Class II Div I
c. Class II Div II
d. Class III

1139. 16 female pt with erythematous lesion surrounded by white hallo in the buccal mucosa and the palate , what is
the tests to order ?
a. Anti DNA
b. ANA and Rf


c. ANA and AntiRo
d. Anti DNA and Anti Smi


1140. ( OPG) patient taking zometa has exposed bone and pathological fracture. What is the appropriate management?
a. Mandibular resection
b. Mandibular debridement
c. Iv antibiotic
d. Open reduction and internal fixation

1141. Which of the following is the classic picture of a minor salivary gland biopsy with sarcoidosis?
a. Fibrosis
b. Acanthosis


c. Acinar hypertrophy
d. Granulomatous inflammation
1142. Which of the following is the recommended isolation method when a root canal treatment is planned for a
maxillary anterior tooth in an asthmatic patient?


a. Use cotton rolls throughout the endodontic procedure
b. Apply rubber dam throughout the endodontic procedure
c. Start with cotton rolls, then apply rubber dam before obturation
d. Place rubber dam, then change to cotton rolls before obturation
1143. During the fabrication of a maxillary complete denture, the dental surgeon examines the palate for recording the
posterior palatal seal. However, he is unable to identify the exact location of the pterygomaxillary notch. Which of


the following instruments is the best to locate the posterior palatal seal?
a. T Burnisher
b. Indelible pencil
c. PKT instrument
d. William's probe


1144. The factor make primary tooth extraction difficult?
a. Long divergent root
b. Long convergent


1145. Difference between primary and permanent tooth crown?
a. More bulbous
b. Converge apically
c. Wider mesiodistally


1146. Dentists take money from implant companies to recommend it ? Considered
a. Illegal it's a bribe
b. Legal it's not conflict of interest

1147. Patient came for CD, he has mild bilateral posterior undercut( tuberous) and anterior undercut? How to manage it
?
a. Remove both undercuts ✅
b. Keep all as it's needed for retention
c. Relief for posterior

1148. Cement that has potential for fluoride influx?


a. Resin cement


b. Polycarboxylate
c. Zinc phosphosilicate

1149. Best way for the community to prevent caries?


a. Topical fluoride
b. Systematic fluoridated water
c. Diet counseling

1150. Patient has COPD and hypertension, and came with a productive cough for routine dental tx . What's the
proper management? Bp normal O2 saturation 90 HR 80
a. Treat him in upright position


b. Treat him and monitor with pulse oximetry
c. Reschedule


1151. While you are doing IANB and retracting with your finger, you stick your finger. What is the first thing to do?
a. Wash the site
b. Report
c. Check pt medical history
d. Encourage bleeding second


1152. While you apply the matrix you cut your finger. What is important to do?
a. Wash it with antiseptic
b. Apply plaster when you reglove
c. Apply sanitizer on glove

1153. mechanism of action of chlorhexidine
a. disrupts microbial cell membranes
b. cell wall inhibition


1154. Child falls while playing in park and his tooth avulsed what is the most influential factor for reimplantation
a. time
b. antibiotic
c. tetanus


1155. While you are doing IANB and retracting with your finger, you stick your finger. What is the first thing to do?
a. Wash the site
b. Report
c. Check pt medical history
d. Encourage bleeding
1156. Pt. taking cyclosporine and causes malalignment of teeth and facets and bad breath, Cal 5-6 with two pictures.
What of the following if treated first will aid in treating other problems. The clinical pictures have severe gingival


enlargement and no plaque will evident
a. Gingival enlargement
b. Bad breath
c. Maligned teeth
d. Oral hygiene

1157. A patient underwent a successful kidney transplant 7 years ago and has a history of diabetes and high blood
pressure. He has periodontitis. What is the first step in treatment?


a. Scaling and root planing
b. ask for medical report


1158. Diabetic patient, hb1ac in the last period 8,9,11. He have treatment plan include multiple implant and crowns and
he need bone graft in #46 area and implant:


a. Change treatment plane optional
b. do phase I treatment and defer other until disease control
c. keep treatment as it
d. do phase I and II and do something
1159. which of the following is contraindicated in I bar attachment of RPD
a. severe abutment undercut and low frenum ✅
b. parallel abutment
c. highly esthetic
d. tilted abutment

1160. patient came with clinic with severe pain and he was MI 3 years ago , incomplete pulp extirpation was done due
to difficult access to the canals , after that 1 g acetaminophen when needed, and 500 mg amoxicillin was per day 5-7


days INR test reading is 2.9 and the norm is 1.8-2.1 , what could be the risk from that case?
a. elevated INR
b. reduced INR
c. severe angina
d. Congestive heart failure
1161. 20 yo female patient came with herpes lesion that last for 4 days (no more details) with pic , what is the dx ?


a. primary herpes
b. Recurrent herpes


1162. 11 yo male px with deficient mandible and retroclined lower anterior which appliance is suitable ?
a. twin block
b. herpest
c. frankle

1163. suitable obturation technique for internal resorption ?


a. carrier based
b. warm vertical compaction

1164. most congenitally missing teeth?


a. third molar,second premolar,lateral incisors
1165. standard treatment of lower missing teeth ?


a. overdenture with 2 implants (minimum)
b. overdenture with 4 implants
c. complete denture

1166. px with recession of upper first molar only in MB cusp with deep pocket what is your management?


a. GTR
b. root amputation
c. hemisection

1167. implant with recession buccally and asking about the treatment
a. change crown


b. remove whole implant and add new implant
c. CT Graft

1168. pt placed class 1 amalgam restoration 3 days ago, came to the clinic complaining of pain when clenching,
clinically TMJ was normal. What is the problem?
a. galvanic reaction


b. over contour restoration
c. high point contact
d. undercontoured restoration
1169. why do we disinfect impressions and appliances?
a. To protect pt from lab personnel


b. protect lab personnel
c. prevent cross contamination
d. prevent lab and equipment from contamination.


1170. pt with dental hx of excess bleeding after extraction? How to manage?
a. condense homeostatic something inside and close with figure 8 incision.
b. prescribe b… pre and post operative
1171. implant placements in relation to mental foramen!?


a. 5mm posterior
b. 5 mm anterior

1172. warfarin and INR = 3.5 ?


a. Processed without modification ✅
1173. Open bite (apertognathia) asking what are the aetiology?
a. Cleft palate,
b. hypo plastic maxilla,


c. retrognathic maxilla
d. Maxillary vertical excess.


1174. pain unilateral of the face with multi episodes asking about the etiology?
a. Herpes zoster

1175. y.o hypertensive patient has hyperthyroidism and you will give him LA with epinephrine which condition the


patient will have?
a. Thyrotoxic crisis

1176. dose of acetaminophen for children? (missing info weight, syrup dosage?160mg\5ml)
a. 15-10 mg/kg 46 h

1177. Dose of ibuprofen for children (missing info weight, syrup dosage?160mg\5ml)
a. 4-10mg/kg 68 h


1178. pt bleed since childhood everything was in normal range?
a. Scurvy
b. Haemophilia
c. Von willebrand
1179. Pic of ulcer in the labial mucosa it was very small in size less than 2mm. Pt got it recent 2 years and come back
every month
a. Recurrent herpetic ulcer


b. Labial hepatic something
c. Minor aphthous ulcer
d. Major herpetic ulcer


1180. What is the other name of clear gingival sulcus?
a. Gingival crevice
1181. Pt came with white pigments all around his teeth after he removed his ortho appliance. What is the most suitable
thing you can do?
a. Florid varnish


b. Florid gel
c. Emphasise on Oral hygiene instructions
d. Restoration
1182. Pedo with anterior open bit and posterior cross bite and thumb sucking habit?
a. Palatal crib (if he ask about the appliance to stop the habit)

1183. Elderly with osteoarthritis plan to place 3 unit bridge posterior upper area, not showing during smiling, it is


difficult for the pt to clean his teeth, which design for the pontic is suitable for this pt?
a. sanitary.
1184. Mouth breather patient what type of periodontal disease he will has in upper anterior area


a. recession
b. chronic inflammation and gingival enlargement.

1185. Shank in scaling


a. parallel to the long axis of the tooth
b. parallel to the surface being treated
1186. patient has new sets of max/mand CD came complaining of pain and reythema in Both ridges what could be the


reason
a. insufficient freeway space
b. insufficient vertical dimension
c. premature contact in latrotrusive

1187. patient has hx of SCC, dry mouth and oral ulcer in the palate that used to be treated by fluconazole,patient want a


definitive tx for his condition
a. Pilocarpine
b. steroids
1188. case pt with good OH and sound abutments want to replace missing central incisor due to trauma, what tx option
you will give her


a. Cantilever FPD
b. conventional FPD (best option)
c. removable
d. implant supported FPD (if it is a single implant will be the correct answer ?)
1189. case of 14 month old pedo pt with purple filled cyst in the gingiva that increase with eating the cc drooling saliva
what is the dx
a. eruption Hematoma (because it’s the Time of primary 1st molar eruption)
b. Bilateral ranula


1190. pic ulcer in the labial mucosa ,patient reported it came during her exams what tx you will give
a. 0.05 fluocinonide
b. acyclovir


1191. case patient with periocornitis with Impingement from the opposing tooth how you will treat the acute symptoms
a. irrigation and extract the opposing tooth

1192. case scenario patient with MI 3 years ago INR 2.9 and you decide to extract for her, what medical condition she
might has (previously mentioned)


a. severe angina
b. congestive heart failure


1193. another question with the same scenario what you will give tx
a. monitor ECG and nitroglycerin prophylactic
b. give antibiotics prophylactic
1194. patient reported last visit during LA administration she has Slurred speech,disorientation and dizziness ,how you
will prevent this from occurring again


a. don’t give LA
b. aspirate before anesthesia
c. change the type of LA
d. reassure patient before and during LA administration


1195. case scenario male patient heavy smoker has bilateral swelling
a. Warthin tumour

1196. case scenario separated instrument in Middle third and you can bypass it what is the prognosis
a. good
b. questionable
c. you can’t predict
1197. case scenario patient just received her crown and she complains about food accumulating in the Cervical third
what colud be the cause
a. crown emergence profile
b. open contact
1198. pic and case scenario patient with multiple missing teeth received her new denture
a. Hybrid


1199. Missing 35,36 and you will constrict RPD where you will place your indirect retainer
a. no need (b.c it is class3)

1200. recommend major connector for torus palatines(mentioned before )
a. U shaped
b. AP palatal strap


1201. what is contraindicated in kennedy class I rpd
a. Circlet clasp ( another name for aker )


1202. Kennedy classification the only remaining teeth #13,#14,#24
a. class I modification 1
b. Class I modification 2
c. Class II
■ Mostly class III modification 1
1203. pic and case scenario patient has deep distal pocket in #35 and he was referred to periodontist to do surgical #36


where missing what is the name of this surgical procedure
a. guided tissue regeneration
b. guided bone regeneration


1204. pic and case scenario patient 66 years old with necrotic #12 what is the cause ( xray showing bone loss to mid of
the rooth and there was no dens invaginates )


a. Dens invaginates
b. Periodontal defect


1205. what is the name for all quantitative enamel defects
a. hypoplasia
b. hypomaturation
c. hypo calcification (qualitative)
d. enamel defects


1206. mother concerned about her child missing tooth ,she also has the same tooth missing(mentioned before)
a. lower 5
b. upper lateral
■ Note:most common missing teeth -third molar ,lower 2nd premolar then upper lateral


1207. patient want to do perio surgery in #11 where you should place the vertical incision
a. mesial 13
b. distal 12


1208. case scenario of 0.5-0.6 cm radiolucency under the mandibular canal ,what is your tx
a. annual follow up ✅
b. incisional biopsy
■ Stafne cyst


1209. missing 46 and tilted 45 and supraerupted 26 how you will manage the case
a. ortho tx to open the space
b. Ortho tx to close the space
1210. case scenario tooth #27 with good crown in the xray has overall pocket 3-4 except in one side 7 , there is no pain
on percussion or palpation what is your tx
a. extraction
b. non surgical RCT


c. apical surgery
d. amputation
1211. Pt haa #17,21 missing and U will do rpd , what type of articulation?
a. hand mounting


b. mount on CR
c. mount on MIC


1212. Recent test: Positive Tb patient needs emergency tx with 10/10 pain?
a. consult physician then decide


b. refer to physician
c. Isolated room


1213. PT with class l needs RPD, you decide to make a patrix and matrix, where you will put the matrix?
a. Mesial to abutment
b. Distal to abutment

1214. Pt with kidney failure came with Whiteyellow lesion, what is the cause ?
a. Ammonia in blood
b. Lichenoid drug reaction


1215. Pt with smooth surface and ulcer on the lip,manegment?


a. antiviral
b. laser ablation
c. Steroids


1216. Geographic tongue with dysphagia,deficiency in ?


a. Vitamin A
b. b
c. C


1217. Pt with sickle cell needs simple extraction?
a. make sure his hydrated
b. Do the extraction
c. Blood test


1218. what is the treatment


a. pulp revascularization ✅
b. apexogenesis
c. Apexification


1219. Autoclave 132 what is the minimum time ?
a. 1
b. 2


c. 3
d. 4


1220. Unilateral headache 2-3 times in 3-8 hours with dizziness and vomiting?


a. paroxysmal hemicrania
b. Migraine


1221. Pt has diffuse swelling mass and not fluctuating but is erythematous ?
a. tumour


b. abscess
c. cellulitis


1222. Pt after scaling in 2 days you have a fever and inflamed gingiva what u will do ?


A. local ABX +scaling
B. systemic ABX +scaling
C. Scaling
1223. Normal implant Bone Loss after 1 year ?
a. 0.2 ✅
b. 1-1.5
c. 3-4


1224. Short RCT but the apical end seems calcified and you want to do a survey crown ?


a. re RCT first
b. do the crown
1225. Sharp pain which nerve
a. A


b. B
c. Ad
d. Aδ


1226. When is the minimum days required to extract before chemotherapy
a. 4


b. 14
c. 7
d. 21


1227. 34 with periodontists stage ll, after you finish scaling and root planing what you will tell the patient to expect the


future visits ?
a. he will have root sensitive
b. he will feel the tooth loose
c. root caries


1228. patient with 1 mm recession and feeling sensitivity in his tooth, what is the first action?
a. describe to him anti sensitivity agent


1229. teeth with open apex and you will do RCT, what is the action that you will consider?

a. sided vent needle (always)
b. ⁠low concentration of sodium


1230. when to do frenectomy after space closure


1231. How to differentiate between Ramsey and bell palsy?


1232. White lesion on the lateral border of the tongue of an HIV PT what you should do to the pt
a. Reassure as it’s a benign lesion
b. Give local antifungal


c. Give systemic antifungal
d. Refer to a specialist


1233. mucocutaneous lesion in histology there’s antibodies attached to prickle cell layer
a. Pemphigus
b. Oral lichen planus
c. Mucous membranes pemphigoid


1234. a 33 year old pt came for extraction of four 3rd molars, he is 50kg, you planned to give him 2% lidocaine with
epinephrine, the carpule is 1.8, how many carpules can you give safely?
a. 6.3


b. 8.3
c. 9.7
d. 12

1235. During cleaning and shaping the dr broke 2 instruments apically, and while trying to retrieve or bypass the
instruments got pushed beyond the apex (Xray provided) what is the management?


a. Follow up
b. Surgical


1236. What make crown lengthening distal to #37 very difficult or nearly impossible?
a. External oblique ridge
b. Lingual nerve


1237. Pt came with recession and treated with this incision, what is the name of the flap? Similar pic
a. Double papilla flap
b. Semilunar


1238. Healthy pt came with white lesion along the gingival margin non scrapable, he is a smoker for 20 years,


management?
a. Biopsy
b. Follow up

1239. Healthy pt with excellent OH came with discomfort around #35 (implant), bleeding all over and 5mm PD buccal
to the implant, management?(previously mentioned in december)
a. Change crown
b. Soft tissue graft
c. Remove implant and place new one
d. Inject antibiotic in the pocket
1240. Saucer Like around implant?
a. Fistula


b. Peri Implant mucositis
c. Periimplantitis


1241. Heart disease pt with prosthetic heart valve, you will do periodontal graft?
a. Preoperative Ab prophylaxis
b. Ask for INR and preoperative Ab prophylaxis

1242. The muscle that limits mandibular lingual border molding?


a. Mylohyoid
b. Genioglossus
c. Superior constrictor


1243. What is the mandibular limit of the distal extension of lingual flanges?


a. Mylohyoid
b. Superior pharyngeal constrictor

1244. Pt came with heavily decayed #36 and large extraoral swelling (similar pic) and buccal vestibular fullness, and
ask about management?


a. intraoral drainage, extraction and Ab
b. Extraoral drainage, extraction and Ab (Probably submandibular space infection)
1245. 5mm from gingival margin to bone crest, 2mm from restoration margin to bone crest and 2mm tooth structure


above margin?
a. Sufficient ferrule and supracrestal attachment
b. Insufficient supracrestal attachment and sufficient ferrule
1246. Epileptic pt had seizure in the clinic?


a. Midazolam 1mg
b. Diazepam 10mg IM

1247. Dr wants to replace multiple units with screw retained implants (#25 to #26) What is the complication of this
type of treatment?
a. Difficult to retrieve
b. Excess cement


c. Interocclusal space
d. Lack of passivity of something


1248. Hypertensive and diabetic pt taking Metformin, recently diagnosed with rheumatoid arthrirtis and taking


methotrexate and corticosteroid, now having ulcer in lateral tongue what is the cause?
a. Methotrexate
b. Candidiasis
1249. Patient came to you in the ER complaining from labial fracture of recently placed porcelain fused to metal
crowns replacing the lower anterior, patient has missing lower posteriors and known case of deep bite, what is the


cause of the fracture?
a. Occlusal overload
b. Incorrect anterior guidance
1250. 4yr pt came complaining of Malaligned teeth, examination reveals unerupted upper canine, what is the tx?(The
photos attached shows class III with moderate crowding in lower and no crowding in U but no space for canine to


erupt
a. Expansion
b. Extraction
c. Nonextraction
d. Distalization
1251. Pt came want to replace #21 with implant, pic shows bad OH and plaque accumulation, caries mesial to #22,


missing lower anterior teeth, and asks about the first thing to do?
a. Scaling AND root planing
b. Restore #22
c. Do the implant
d. Do the lower RPD
1252. Pt with hx of Jaundice 2 years ago and hepatitis, lab results shows +ve antiHBs and antiHBe only, what does that
mean?
a. Low infectious


b. Highly infectious
c. Not infectious

1253. Pt came with pain, fever and limited mouth from tooth #36, which space infection?

a. Submandibular
b. Submassetric
c. Ptyregomandibular


1254. Most common tumor of parotid gland?
a. Pleomorphic adenoma


1255. Pt with geographic tongue have burning sensation how to treat?


a. Steroid
b. Antifungal
c. Antivirus


1256. Xray of vertical bone loss mesial to premolar and ridge resorbed and another pic of graft above the ridge, asking


what is the name of the procedure?
a. GBR
b. GTR


1257. Bur for prep of V Shape cingulum rest on #13 ?
a. Round
b. Fissure


c. Tapered
d. Inverted cone

1258. Clear X Ray of simple bone cyst and ask about management?


a. Radiotherapy
b. Surgical


1259. Purpose of opaque layer in crown?
a. To mask the oxide layer

1260. Patient has ulcerated in her lips and the disappear after for days( pic of ulcers on the lower lips


a. Recurrent herpetic ulcer
b. primary herpes ulcer
1261. patient has rashes only in one side of his body and his face itches what virus caused it :
a. CMV


b. Herpes simplex
c. Herpes zoster


1262. Removes and stops the growth on inanimate surfaces:
a. Disinfection
b. Seterlization
1263. Patient doesn’t want to hear the side effects of her procedure. She's medically fit and refused and wouldn’t listen


to the doctor ?
a. Take the consent form her and tell her the side effect
b. Take the consent form her son and tell him the side effect
1264. Patient is has some mental disability not fully and came to you to assess his mental capacity, from whom do you


take the consent from ?
a. Parents
b. Doctors
c. Him
1265. Patient admitted to the hospital with covid signs and the doctor didn’t inform the staff of the patient condition
what did he violate:


a. Infection control
b. Community
c. Colleagues
1266. Adjusting the try in of upper molar and it was noticed that it glides superior anteriorly :


a. Distal inclination of the mandible
b. Lingual inclination of the mandible ( mostly a protrusion interference)
c. Mesial inclination of the upper
d. There was a fourth choice
1267. Patient came with white line on the side of her tongue asymptomatic what to do :
a. Incisional biopsy ( if there is no exfoliative)
b. Excisional biopsy


c. Topical steroid
d. Exfoliative cytology

1268. Patient pedo came with her parent and said she ingested 50mg of fluoride :


a. Just observe
b. Give milk
c. Admit to hospital


1269. Patient has kennedy class ii and want to use the abutment that was mesially tilted what to use:
a. Ring
b. Combination
c. RPI
1270. Linear line bilaterally on the facial cervical third :


a. Abfraction
b. Abrasion


1271. Patient came to the clinic that had ortho TX and took them off due to inflamed gingiva and there were no bone


loss present what to do:
a. Scaling and gingivectomy
1272. Patient came the next day with pus and pain and (it looked like dry socket) what to do :
a. Antiseptic mouthwash , AB, curettage


b. antiseptic mouthwash,AB, irrigation
c. Irrigation


1273. Patient started to convulse and later lost consciousness what to do :


a. Semi supine
b. Supine
c. reclined
d. Semi reclined.
1274. Patient edentulous came and stated that his retromolar pad touches the upper hamular notch what to do ::
a. Increase the vertical dimension
b. Extend the metal to the retromolar pad
1275. Lesion on x ray that shows only white not super calcified with no RL rim what is it ?
a. Osteoma
b. Forgot the rest
1276. Patient came with crown that keeps coming off what to do thigh of the tooth is 5mm and forgot the rest: (my


assessment was that the tooth height was accurate and it’s a retention problem)
a. Add retention grooves


1277. Patient came with central intruded what will happen to the permanent :


a. hyperplasia of the root
b. palatally displaced
c. buccally displaced
d. devitalization of the permanent


1278. Prophy jet is used:
a. On implant
b. Removing calculus
c. For patients with haemodialysis
d. Polishing amalgam and composite


1279. Nodules on the anterior palate of patient using CD:
a. Epulis fissuratum
b. Nodular hyperplasia


1280. what is the procedure
a. Guided bone regeneration (GBR)


1281. Patient has hemophilia A , had severe bleeding after 3rd molar extraction, should you give him?
a. Vitamin K
b. aminocaproic acid


c. Platelet transfusion
d. Factor IX

e.
1282. component that attaches the prostheses to the abutment tooth?


a. Pontic
b. Retainer

1283. Photo of provisional restorations on upper incisors implant were made to have favourable healing of the tissue
Asks what kind of impression coping should the dentist used to register the exact tissue configuration after healing?


a. Open tray coping
b. Customised impression coping

1284. excellent oral hygiene but traceable sinus tract from the mid buccal of the cement retained implant, with Pa of
implant asking about the cause (mild bone loss, with a RO projection from the cervical part of the implant, for me it
does not look like a cement it’s radiopacity similar to the crown
a. Excess cement (most probably)
b. Loose screw (he will say mobile crown, bad odor)
1285. a patient came for construct a new set of CD, came with inflamed traumatised mucosa, what to do?


a. Rebase denture
b. Reline denture with tissue conditioner
c. instruct patent to not wear denture during sleep time

1286. ceramic onlay after you insert shims stock only the inlay grips , no contact with other teeth, this means:
a. good contact


b. deficient contact
c. occlusal interference, needs adjustments of only
d. slight hyper occlusion of onlay but it is acceptable

1287. characteristic feature of setting reaction of PVS?
a. Addition
b. Condensation
1288. RCT done for 11, 12 send back for final restoration, treatment steps?
a. preparation and provisionalism only


b. CL, post and core and crown
c. mounting, diagnostic wax up, preparation, provisionalism


1289. 9y/o came after 30 mins of intrusion trauma 3-4mm #11, best treatment?
a. Follow up
b. ortho reposition


1290. Most commonly reported trauma?
a. Avulsion (in pedo)
b. Root fracture
c. Complicated crown fracture
d. uncomplicated crown fracture (in adult)
1291. Smoker patient asking about perio effects?


a. decreased both gingival inflammation and BOP
b. increased gingival inflammation and decreased BOP

1292. pic of dry socket management?


a. antibiotics


b. mouthwash, analgesics and curettage
c. mouthwash, analgesics and irrigation


1293. treatment sequence?
a. Endo,ortho extrusion, crown
b. Ortho extrusion, Endo, crown


1294. protruded archwire and ulceration in buccal mucosa, lip tongue, management?(previously mentioned)
a. Clip the wire
b. Cover the wire with wax (said wire not bracket)
c. paracetamol
1295. Xray essential for facial asymmetry
a. cephalometric
b. ⁠Posterior anterior


c. ⁠OPG
d. CT


1296. an adult with tongue retained appliances for what?
a. sleep apnea
b. Tongue thrust

1297. Caries free 2 years old pedo with bottle feeding at night, should use?


a. Smear size non fluoridated tooth paste
b. Smear size fluoridated toothpaste
c. Pea size non fluoridated tooth paste
d. pea size fluoridated tooth paste

1298. pedo ingested 50 mg ingested, what to instruct her mother?


a. Observe for symptoms


b. Observe then immediately go to ER
c. Give milk and come to ER
d. Give acidic solution and come to ER
1299. patient complain from discomfort when eating and brushing around implant, excellent OH, reason (picture of


implant with insufficient keratinized gingiva)
a. insufficient keratinized tissue

1300. Components in CD that can cause allergy/inflammation and reaction to some patients?
a. benzoyl peroxide
b. unreacted monomers (i think this is the closest_)
c. PMMA

1301. has a patient has multiple punch out lesions?


a. gorlin syndrome
b. multiple myeloma


1302. signs of biologic width violation, margin 1.5 mm away from bone. horizontal or vertical bw?
a. Vertical bitewing
1303. OM mesial restoration 2 days ago, grade 1 mobility and pain with cold, radiograph shows “funnelling in pdl of


mesial side”.
a. 1ry occlusal trauma
b. 2ry occlusal trauma

1304. a patient with hx of HIV AIDS upon examination, oral hairy leukoplakia was noted on the side of the tongue.
what to do
a. Reassure it is benign in nature
b. Refer immediately to the physician because it is a sign of disease progression


c. systemic antifungal
d. topical antifungal

1305. Clinical picture (occlusal view) of 2 molars with fallen crowns, mentioned that the patient is angry about the
repetitive fallen of the crowns every few months, what is the best long term treatment?


a. Recement with RelyX
b. Crown lengthening
1306. intraepithelial separation ?


a. MMP
b. Pemphigus vulgaris
1307. patient is taking 40mg prednisone everyday, needs scaling and 2 restorations, management ?
a. Give 35mg prednisone, prohy, scaling, restorations


b. Give 20mg prednisone, prohy, scaling, restorations
c. Prophy, scaling, restorations

1308. irregular elevations at the middle of anterior maxilla?


a. residual ridge
b. palatal rugae
c. median raphe

1309. tooth bonded posterior maxillary RPD, mentioned that the patient has a palatal torus in the midline but not
extending to the soft palate, best major connector?
a. U shape


b. anterior bar
c. Anterior Posterior palatal strap


1310. stafne defect location?
a. angle
b. ramus

1311. usual sequel after extraction of lower 3rd molar?


a. inadvertent injury to the lower 7
b. paresthesia to the IAN


c. fracture of the mandible
d. edema and discomfort
1312. Pt complains of pain and limited mouth opening, upon examination, pt has clicking sound, limited mouth
opening, deviated to the right side, most likely diagnosis?

a. Internal derangement
b. Functional dislocation with reduction
c. Functional dislocation without reduction


1313. missing 12, protruded 11 with proximal caries, 13 is present with proximal caries , want to replace 12:
a. Implant
b. Conventional fpd with abutment 11,13
c. Resin bonded fpd
1314. Previously TB infected pt, was taking his medication for 6 months (i think), now he is clear


a. Treat normally
b. Mask

1315. Pic of white lesion in lateral border of the tongue:


a. Laser ablation


b. Excisional biopsy
c. Incisional biopsy
d. Cryco
1316. Definition of class I malocclusion?


a. Perfect aligned teeth
b. Malaligned teeth

1317. location of the second canal in lower central ?


a. Mesial
b. Distal


c. Labial
d. Lingual
1318. Missing 11, 24,25, best material for base..?


a. Acrylic resin
b. Cobalt chromium


1319. Uncontrolled diabetic pt, poor oral hygiene…?
a. Consult physician
b. Scaling and consult physician


1320. pediatric pt , you gave him 2.5 mg diazepam, …develop symptoms couldn’t remember them..?
a. Overdose
b. Anaphylactic reaction

1321. A patient has a metal ceramic crown on tooth #45, which requires endodontic treatment. The patient refuses to


remove the crown. What approach would you take to create an access cavity through the crown
a. Dimond for ceramic, carbide for metal
b. Carbide for ceramic, Dimond for metal
c. Ultrasonic

1322. old pt with remaining lower anterior teeth, grade 1 mobility, labially inclined, 12 mm from gingival margin to the
vestibule, which major connector…?


a. Lingual bar
b. Lingual plate

1323. a 13 years old boy, medically fit, came to the clinic with a complaint of enlarged gingiva, upon examination,
gingiva appeared pink, firm, stippling present (they used different terminology for stippling), the reason for


enlargement?
a. Idiopathic gingival enlargement
b. Drug induced gingival enlargement
c. Systemic…

1324. Diabetic pt, good oral hygiene, gingival swelling apical to the 46, there was pocket buccal side (couldn’t
remember the depth in mm), i think there was horizontal bone loss…? Pic provided


a. Gingival abscess
b. Periodontal abscess

1325. Long scenario about pt with rpd, difficult to insert and remove the denture, occasionally the retentive are


fractured, reason for fracture..?
a. Cyclic fatigue of the retentive arm
1326. PA of #25 that has a relatively short root and Periapical radiolucency with around 2 mm above the bone” they
mentioned that the tooth is equigingival clinically and asked what is the best way to isolate the tooth.
a. no need


b. cotton roll
c. extract and implant
d. Clamp the #26 and extend the dam to #24
1327. incisal reduction of 11 metal ceramic crown…?


a. 1 mm
b. 2 mm


1328. skin lesions, oral ulcers, peri oral crusting lesion, fever and myalgia, pt says it happened to him previously with
complete remission…?


a. Erythema multiforme
b. Primary herpetic gingivostomatitis


1329. A dentist, sustains a needle stick injury from a patient with HIV. The patient has been adhering to medication,


and the virus is not active. What should the dentist do?
a. Visit the infectious disease clinic.
b. Induce bleeding and clean the finger with soap.
c. Refer the patient to the infectious disease clinic


1330. Scenario with Lab work, MCV 90, HB within normal range…? Type of anemia


a. Iron
b. G6PD
c. Folate
d. B12


1331. Growing pt (they specify the age) with class iii, appliance …?
a. Facemask .
b. Twin block
c. Headgear


1332. How to accelerate stone setting without jeopardising the quality..?


a. Sluury water
b. Hot tab water
c. . Podwerliqued ratio


1333. pic of lesion in lateral surface of the tongue, biopsy revealed squamous cell carcinoma, you remove the lesion,


what should you do next…?
a. Radiotherapy
b. Hyperbaric oxygen


1334. multiple missing primary teeth, permanent molar and incisor already erupted, best space maintainer..?
a. Lingual arch
b. Removable denture
c. Bilateral crown and loop
d. Bilateral band and loop


1335. pic of pt, she is unsatisfied with her smile, good occlusion, short clinical crown, 12 mm sulcus depth…. How to


improve appearance (pic shows discolored teeth)
a. Crown lengthening
b. Ortho extrusion
c. Gingivectomy


1336. in lateral cephalometric, you will assess…?
a. Anteriorposterior skeletal relation
b. Transverse relation


1337. pt unhappy with new cd biting ?
a. Offer overdenture .


1338. rounded bur finsh line?
a. chamfer
b. Bshoulder
c. shoulder with bevel


1339. complication of mesial tilted 37 for 3 unit bridge ?


a. pulp exposure
b. short mesial axial wall
c. irrivirsble pulpitis


1340. Combinations syndrome complications?
a. Lower post bone resorption


1341. denture base for limited inter arch space ?
a. Metal


1342. Class V resto in sjogren syndrome ?
a. GI


1343. What caries are associated with xerostomia ?
a. Cervical RCT
b. Cervical extraction


1344. Salivary flow in Sjogren syndrome?
a. <5 per 5 min


1345. class 1 amalgam on lower and cusp fracture ?
a. Undermined tooth structure
1346. Mild amalgam OD on premolar ? Replace by?


a. composite
b. only


c. crown


1347. Reciprocal system ?
a. Endo eze (wave one)


1348. How to locate a calcified canal ?
a. Only bubbles test (no CBCT OR Blue stain, also take multiple pa)


1349. Half of the face pain with rash and vesicles ?
a. Post herpetic neuralgia

1350. Case of short clinical crown with 2 mm between CEJ and crest of the bone ?
a. gingivectomy !


1351. a 28 year old patient noticed her hair getting less and tachycardia. What are the tests required?
a. TSH
b. ⁠testosterone
c. ⁠alh
1352. Patient with low levels of phosphate, and something else


a. hilum syndrome
b. ⁠phosphatasia or something


1353. mass look like dom shape in floor of Mouth
a. Ranula

1354. Pt has diffuse swelling mass and not fluctuating but is erythematous ?
a. cellulitis

1355. pedo pt uncooperative had deep stain in permanent


a. varnish and pt&fissure selant on premanent teeth
b. Prophy with flouride varnish


1356. pt did extraction to upper 16 after that he felt numbness
a. Ans of greater palatine
1357. Class v remaining thickness is less than 0.5 mm what will us as lining?


a. RMGI (this is only a base)
b. GIC

1358. class 1 with deep caries RDT is 0.5
a. RMGI
b. GIC
c. RMGI base with GIC liner


1359. How to increase the working time of zinc oxide ?
a. Mix in cold glass

1360. 37 years old PT case scenario (pic clinical and radiograph ) heavily destructed lateral incisor ,endo treated with
history of fractured post and core ,asking about long term treatment ?
a. Ortho extrusion followed by osseous surgery (will compromise the tooth\root ratio)


b. Maryland resin bridge
c. Extraction and implant
1361. Pt after trauma 20 years ago,Opg with very short root but the tooth is not mobile, what type of resorption?
a. internal


b. surface resorption
c. replacement resorption

1362. What are the factors that determine the forces directed to the alveolar ridge and tooth ?
a. Direction and magnitude of force


b. Type of denture base
c. Use multiple rest and clasps with broad denture base

1363. missing 35,38,48 and u will not be able to replace the third molars. What is the Kennedy classification ?


a. class 1
b. class 3
c. class 1 mod1
d. class 2 mod 1
1364. ptt has RPD ,came to the clinic complaining of discomfort on the abutment tooth , what is the reason?
a. occlusion


b. exposed dentin under rest
c. galvanic shock due to amalgam
d. VRF
1365. When denuded cementom there will be exposed dentin and there will be communication between pulpal tissue
and PDL Which of the following will be the cause ?
a. VRF
b. developmental defect


1366. after removing the CD there is roseberry like protection?
a. denture hyperplasia
b. epulis fissuratum


1367. if u want to retract anterior teeth and you have space, what would you use ?
a. Hawley with labial bow

1368. Pt has rescission on #13 ,she will do C.T gravy and coronal placed flap Which incision will be used ?
a. external bevel


b. internal bevel
c. sulcular

1369. case about lesion in lower lip since 6 weeks , transparent and patient reports it reduced in size by tima What is
the contents of it ?


a. Fat
b. Mucus

1370. What is the length of the implant The missing tooth is number 15, Given that the distance from the bone crest to
the sinus is 12 mm
a. 11 ✅


1371. What makes pit and fissure sealant more retention?
a. Acid etch
b. Air abrasion


1372. You did condensation for amalgam for 8 min what will happen
a. Amalgam will be set
b. Easy to carve
c. Increase mercury

1373. Who to differentiate between nug , desquamative gingivitis

1374. impression for cd?(best is polyether but not there)


a. Polysulfide
b. Addition silicone (Pvs)
c. Reversible elastomeric (agar-agar)
d. Irreversible elastomeric (alignate)


1375. which material is reversible?
a. Agar
1376. Difference between Osteoarthritis and Rheumatoid arthritis

1377. Which disease is associated with this lesion (same picture) most probably its fibrous dysplasia?

a. Epilepsy
b. Diabetes

1378. to extract badly destructive lower 3rd molar , the incision should be ?


a. At least until 2nd premolar
b. Base is wider than crestal
c. incision periostealsulclar

1379. Pt present to you with clubbing fingers, he tell you that he can’t tolerate exercise and gets tired immediately,
what is the management? (congestive heart failure)


a. give NSAIDS
b. Treat him in upright position
c. Give him 34 LA with Epi
d. Treat him as normal
1380. Pt came to you for a new extraction he tell you that he has continuous unstoppable bleeding in the last extraction
, what you will do after consultation?


a. Blood perfusion
b. Put hemostasis and suture after extraction
1381. The phosphoric acid percentage of the abutment of the Ceramic crown is ?
a. 15


b. 20
c. 37

1382. a 3 years old child with teeth with multiple caries , what is the management?


a. GIC
b. RMGIC
c. Composite
d. Amalgam
1383. 37 years free of caries and fluoridated water at his living area and he brushes twice daily, when will you call him
for the next visit?


a. 3 month
b. 6 month
c. 8 month
d. 1 year

1384. 14 years old with bluish lesion in the gum


a. Eruption cyst
b. Peripheral Giant cell granuloma
1385. 42 female pt present to clinic with sudden upper central mobility (the picture display a good OH and only
erythematous gingival margin of #21 #11)
a. Periodontitis
b. Necrotizing Gingivitis


c. Gingivitis
d. Pathological migration

1386. Girl presents to the clinic with tissue covering the lower 3rd molar, the dentist decides it needs an extraction but
he can remove the tissue and irrigate as a temporary treatment, she decides to remove the tissue only, what should the
dentist do?


a. Remove the tissue
b. Refer for other Consultation
c. Take a consent and extract
d. Take a consent and remove the tissue
1387. Antibiotic for oroantral communication
a. Penicillin


b. Metronidazole
c. Amoxicillin with clavulanic acid
d. Clindamycin
1388. Pt take amoxicillin and came to you after 6 days with diarrhea and abdominal issue, what you will do?
a. Complete the antibiotic course


b. Replace with Clindamycin and complete the course (if in the beginning days )
c. Stop the course (because he almost finished the course 6 days this is the closest answer)


1389. Pt present to clinic with 3 unit ceramic bridge #11#22 with half incisal of #11 chipped, what will you do?
a. Remove and take a new impression (because huge defect as half of it gone)
b. Composite repair at the clinic (if small chipping)
c. Repair
1390. Female pt present to you with soreness on the occlusion area and premature contact, why?


a. Decreased vertical dimension
b. Decreased Freeway space

1391. Intrusion of primary incisor what will cause?
a. Possible palatal displacement of permanent
b. Hypoplasia

1392. Pt have denture that cover 1\3 of the retromolar bad area, what is the correct>
a. Good extension


b. Overextension, it should not cover it
c. Underextension it should cover 2\3 of the area
d. Underextension it should cover th whole the area
1393. Assistant get injured by scalpel blade while she take it from the surgeon and she got the infection with 15 days
holiday to sit at her house, what is her legal duty ?
a. Come to the work the day after
b. Don’t come to work until the infection is resolved


c. Report the uncaring surgeon
d. report the incidence to the person in charge
1394. 42 years old female pt came to you with Cc: I’m not happy about my teeth appearance (the same picture but on
all her teeth)

a. Enamel hypoplasia ✅
b. Amelogénesis Imperfecta
c. Dentinogenesis imperfecta
1395. Pedo pt avulsion tooth less then 1 hour should immerse before splitting
a. 2% sodium fluoride 20min (if close apex)
b. Minocycline (if open apex)
c. Extraction ( if primary tooth)
1396. preventive resin components
a. RMGI
b. Compermer


c. Glass ionomer
d. composite
1397. Atraumatic restorative treatment


a. Microabrasion
b. Conservative treatment
1398. reason in chipping porcelain on pfm


a. bonding b/w metal and porcelain
b. Insuffeinat porcelain thickness
c. occlusal truma


1399. pt has Vlll less than 1%
a. haemophilia a
b. Haemophilia b
c. Haemophilia c
d. Thrombocytopenia

1400. pin in amalgam fracture why


a. On the level of cej


b. parallel axial wall
c. 0.5 on enamel or dentine ( it should be 2 mm in depth)


1401. Inflammatory papillary hyperplasia happen in palate why
a. Not removed denture at night

1402. pt has erythmatues and burnings sensation but when remove denture he is feeling comfortable and doctor check
oral hygiene and smooth and apply indicator paste and check everything it was good what is reason


a. Pressure palate
b. Allergy from material of denture
1403. pt extracted 36 I don’t remember when but he said he has pain in tooth and when you check you found dry socket
and exposed boen


a. analgisic antibiotic and currtae
b. Irrigation, antibiotics, follow up
1404. teeth 41,31 dentist position sets on :


a. 9 o’clock
b. 12 o’clock
c. 11 o’clock
1405. Pt with history of renal transplant 1 year ago and he developed multiple white and red lesions in the buccal
mucosa and pain, what to prescribe ?
a. paracetamol


b. Prednisone
c. Acyclovir
d. Penicillin


1406. pt after scaling pt eat fish then come next day with swelling
a. gingval abssess
b. Periodontal abssess


1407. what is area to locate fovea palatin and vibrating
a. Posterior palatal seal


1408. pt has nikolysky signs sub epithelial
a. Pimphegoid

1409. Center of rotation of single tooth ??


a. Apex of tooth
b. Middle tooth and apical
c. Middle tooth and crown
d. Furcation
1410. Ovate pontic, tissue surface cause irritation? “ NOT SURE”
a. gold
b. Resin ..


c. Porcelain
d. Nickel chromium
1411. Part of denture responsible for stability?


a. Minor connector and rest
b. Major connector and proximal plate
1412. primary stress bearing in mandible

1413. mechanism of action of penicillin?


Work on the cell wall membrane

1414. how to to border moulding in lower lingula “READ ABOUT IT”

✅”BCD4 according to Moyer“


1415. Traditional extraction?
a. Primary lateral > primary Canine > primary molar > 1st premolar
b. Primary lateral > p canine > 1st molar > second molar
c. Primary canine > first molar > second molar
1416. What is the primary route for spread infection?
a. Airborne

b. Contaminated instruments
c. Hand and Clothes ”direct”
d. Blood borne


1417. Bilateral lesion in buccal and involve tongue and has purple polygonal papules in the skin
a. Lichan planus
b. Pemphigus vulgaris
c. Erythema multiform
d. Mucous membrane pemphigoid

1418. Director of Government clinic his friends wants elective treatment and ask you to expedite his appointment
because of the long waiting list: What is the most appropriate thing to do???
a. Report friend to patient relations
b. Do fast appointment to those who need it


c. Take him to emergency room and treat him
IF THERE’S “ Treat his pt following the schedule then treat this pt at the end as Extra”
1419. wear of teeth + island of amalgam what type of wear?


A. Abrasion
B. Erosion
C. Attration
D. Abfraction

1420. Red dot in the palate


a. Hhv8
b. Cmv
c. Ebv
1421. Patient has a prosthesis ( I think complete denture I can't remember) has bruxism, what should you do to fix this
problem?
a. Reline
b. Rebase
c. Make teeth cusps inclined 30 degrees
1422. Patient 75kg you want to give anaesthesia What the maximum recommended dose for this patient lidocaine 2%


with epinephrine
a. 525 (if adult multiple weight by 7 if pedo multiple in 4.4)
b. 600


1423. Advantage of GIC
a. Leaching fluoride
b. Thermal expansion more than dentine
c. Thermal expansion more than enamel


1424. 9 years old patient had trauma in central incisor 3 hours pulp exposure?
a. Pulpotomy
b. Dpc
c. Pulpectomy
d. RCT


1425. Purpose of post?
a. Provide core retention
b. Provide canal retention
1426. Patient has hypersensitivity you did scaling and after 2 days she came with localized ulcers in the buccal surface
of teeth #35 #36 #37 and she has very good oral hygiene, what is the appropriate management?
a. Redo scaling
b. Place localized antibiotics ( in places where he has ulcer )


c. Place desensitising agent and reassure patient
d. Apply topical steroid (to reduce inflammation and ulcer symptoms)

1427. Patient came for check up, clinical examination shows abscess in the buccal of posterior tooth, the tooth does
not respond to cold or percussion, what is the best management?
a. Scaling
b. .Surgical RCT


c. Give antibiotics first for this case
d. Nonsurgical RCT
1428. Xray of deep caries in premolar close to pulp, you did cold, heat and percussion test and there was no response,
what's the next step?


a. Take EPT test
b. Remove caries then reassess

1429. Patient has bleeding when he brushes and on examination gingiva bleeds easily during probing which stage?
a. Neutrophil


b. Leukocyte B (if he said bleeding with gingival enlargement of change in colour)
c. Leukocyte T

1430. What is the difference between dentinogenesis and amelogenesis imperfecta?


a. Hereditary


b. Brown color of tooth
c. Dentinogenesis has calcified pulp chamber and short roots


1431. Distance between calculus and bone
a. 1.97
b. 2.97
c. 3.97
d. 4.97

1432. Pulp exposure after 3 days


a. Cervical pulpotomy
b. Partial pulpotomy with MTA


c. Pulpotomy
d. RCT


1433. Patient complains of a denture fall when he pulls the lip, what is the reason for this?
a. Labial notch
b. Buccal notch
c. Buccal flange
d. Over extended posterior
1434. Xray of implant severe bone loss all the way to the apex ( should be removed ) there was excess cement in the
xray in crown marginWhat is the appropriate management?
a. Remove implant ( ‫)اتوقع هنا المقصد انه يشيل االمبالنت‬
b. Remove cement
c. Replace crown
d. Scaling and debridement
1435. Why do we remove overhang in restoration?


a. Allow accumulation of calculus
b. Prevent gram -ve bacteria to grow
c. Allow gram -ve bacteria to grow
1436. Patient with Liver cirrhosis needs extraction of mobile tooth
a. Delay treatment


b. Give antibiotic
c. Take INR before treatment

1437. Smoker obese patient you do extraction and he has chest pain
a. Finish procedure quickly
b. Place in supine


c. Give chewing aspirin 500mg
d. Give glyceryl trinitrate spray under the tongue

1438. Patient came for dental check up, on clinical examination there's 2mm between CEJ and bone, what does this
mean?
a. Gingivitis


b. Periodontitis
c. Normal alveolar bone level
d. Reduced alveolar bone level

1439. Tooth with large amalgam restoration and caries and you need to change it and the patient is bruxer
a. Ceramic onlay
b. Amalgam


c. Composite
d. Metal crown ( if there is gold onlay it will be the best answer)
1440. Patient had trauma in her anterior teeth and on clinical examination dentist see class IV in mesial side involving
the contact area the dentist decides to restore it immediately. Where should the dentist place the mesial contact?


a. Middle third
b. Incisal third
c. Whole mesial contact surface

1441. Child came to the clinic complaining from pain in primary tooth, has large, extensive caries. The diagnosis is
irreversible pulpitis, which type of x ray should you take?
a. Bitewing


b. Panorama
c. Periapical ( to see furcation involvement and eruption of succedaneous teeth)
1442. Patient came with interproximal caries, what is the best method to diagnose this lesion?


a. Transillumination (for crack diagnosis)
b. Bitewing
1443. patient with replanted tooth came after 4 months has resorption and in apical area has “scooped out” appearance
in xray, what the appropriate management?
a. Extraction (if we did rct and then he came with scooped out will go with this option)
b. Apical surgery and retrograde filling with MTA
c. Place intracanal medicament for 4 weeks ( if we only replanted we will start rct with caoh)
1444. Patient has a crown and during checking the margin there’s space between tooth and crown, what can this cause?


a. Crown fracture
b. Prone to caries
c. Dissolution of cement


1445. Best method to diagnose plaque?
a. Disclosing agent
b. Swap with cotton role


1446. Resonance frequency analysis measures what?
a. Implant stability
b. Implant mobility
1447. Multiple radiopaque and radiolucent lesions in all quadrants
a. Osteoid osteoma


b. Cemntoma
c. Florid cemento osseous dysplasia


1448. Non scrapable white lesion on the buccal and when stretch it’s not disappear
a. White spongy nevus

1449. You gonna increase filler in the restoration , what will happen


a. Increase hydrophilic
b. increase wear resistance
c. increase shrinkage

1450. Erythematous , genital ulcer Mouth ulcer , lymphadenopathy Ttt:


a. nystatin
b. Penicillin (because it is syphilis )
c. Prednisone ( if there is ocular lesion it will be behcet's disease)
d. Antiviral vir
1451. How to measure gingival suppuration?
a. Periopaper (blotter)
1452. What material will cause sensitivity to the patient if left without clearing of denture from the lab ?
a. residual monomer
1453. case of patient patient opening and right condyle moving more forward with pain dx?
a. anterior disc displacement with reduction
b. anterior disc displacement without reduction
c. myofascial dysfunction syndrome


1454. young adults with ortho and enlargement of gingiva what is done first ?
a. SRP and re-evaluating
b. Gingivectomy
1455. prominent collagen type during tooth formation
1456. local anaesthesia calculation of medically free female patient 75 Kg If we will use 2%lidocaine with 1:100000epi
1457. Patient came for dental check up Upon radiograph examination you found overhang amalgam restoration without
any complain from the patient what you will do?


a. Reratoration
b. Remove overhang
c. No Tx
1458. Which type of fluoride is the most cost-effective?


a. gel
b. varnish
c. mouth wash
d. supplement
1459. what to give for a caries free Pedo patient with cardiac issues?
a. fluoridated toothpaste


b. Non - fluoridated toothpaste
c. fluoridated toothpaste with biannual varnish
d. Non fluoridated toothpaste with biannual varnish
1460. Diabetic and hypertensive patient taking Metformin and beta blockers and having multiple amalgam restoration


came with metallic taste, which of the following is the reason of his complaint?
a. Metformin


1461. 6 YO Pedo Patient , low caries assessment came for follow up appointment which X-ray should we take for him?
a. 2 bitwing

b.

1462. Mixed dentition patient with unilateral cross bite , how to treat him?
a. unilateral expansion

b.

1463. Patient with stable MIC , came for replacing one crown, what is the most important to achieve when dealing
with single crown cases?
a. centric


b. eccentric
c. MIC
1464. female patient done with perio treatment but still have multiple deep pocket , the periodontist decided to do


periodontal surgery, what is the goal ?
a. pocket reduction

b.


1465. asthmatic patients develop bronchitis progression. What to give him?
a. Salbutamol


1466. Which is the most affected teeth by bone resorption during ortho treatm
a. Upper incisors


1467. What is the best prognosis type of horizontal root fracture?
a. apical
1468. Patient with immediate denture for 6-7 months, came with a complaint of poor retention. What is the best


treatment?
a. ⁠laboratory reline
1469. Deep carious lesion and indicated for extraction. Which of the following criteria help in choosing type of
extraction?


a. Gingival swelling in buccal
b. 3 site caries
c. Pulpal pain
d. Percussion
1470. Cement that has potential for fluoride influx?
a. Resin cement


b. Polycarboxylate
c. Zinc phosphosilicate


1471. While you are doing IANB and retract by your finger, you stick your finger. What first thing to do?
a. Wash the site
b.
c. Report
d. Check pt medical history
e. Encourage bleeding
1472. most fracture occur in:
a. Coronoid
b. Body
c. Angle
d. Symphysis
1473. Patient has a problem with eye, can't wrinkle face?
a. a. Needle went to parotid gland
1474. Old pt present with ridge resorption atrophy of tongue and loss of all the teeth she complains from burning


sensation of tongue ?
a. Improve nutrition


1475. pic of upper lateral broken it was build up with post ask about the treatment (depend on the picture but mostly


extrusion due to the aesthetic demand) ?
a. Extraction and implant
b. Re-do the post and crown
c. Ortho extrusion
d. maryland bridge
1476. fruit laden pranchless tree and shcrimer test 3 mm in 5 min? ( ‫‏‬case scenario , they are more lab test I just


remember the schirmer test )
a. Sjorgen
b. Behcet


1477. U broke a file and bybass it what is the best management?
a. Disclusre to the pt


1478. pic case extruded calcium hydroxide outside canal and no pain or symptoms?
a. Follow up
1479. which bur you will prepare a cingulum rest?


a. round bur
b. inverted cone
c. fissure


1480. (long scenario ) Pt girl has swelling on both arches( starry sky) . What disease?
a. burkitt lymphoma
b. Cherubis syndrome


1481. lf the force used for central incisor 75 for tipping what the force used to intrusion:
a. 125
b. 75


c. 100
d. 25

1482. Patient had trauma in her anterior teeth and on clinical examination dentist see class IV in mesial side *involving
the contact area* the dentist decides to restore it immediately Where should the dentist place the mesial contact?


a. Middle third
b. Incisal third
c. Whole mesial contact surface


1483. Patient came for check up, clinical examination shows abscess in the buccal of posterior tooth, the tooth does
not respond to cold or percussion, what is the best management?
a. Scaling
b. Surgical RCT


c. Give antibiotics first for this case
d. Non-surgical RCT


1484. Erythematous, genital ulcer Mouth ulcer, lymphadenopathy?


a. nystatin
b. Penicillin
c. Prednisone
d. Antiviral -vir


1485. Patient complaining of change size of dentures ( long scenario with lab results )? (mentioned before
a. Paget's disease

1486. a patient with replanted tooth came after 4 months has resorption and in apical area has "scooped out"
appearance in x-ray, what is the appropriate management?
a. Extraction


b. Apical surgery and retrograde filling with MTA
c. Place intracanal medicament for 4 weeks
1487. Missing 35-36 only indirect retainer will be on?
a. 45&46


b. 45&46 embrasure
c. No need
d. 43&45


1488. What are the components of rc-prep?
a. EDTA, urea peroxide, glycol
b. EDTA, NACIO, glycol
c. EDTA, CAOH, glycol


1489. Bacterial of infective endocarditis?
a. Streptococcus
b. Staphylococcus


1490. If we have pt did RCT with ferric sulfate what do think happened?
a. internal tooth resorption
b. external tooth resorption
c. Apical tooth resorption
d. Coronal tooth resorption

1491. A doctor cut her finger on Thursday, on Sunday she went to the clinic to see her patients and was wearing a


diamond ring and wedding ring. What should she do before washing her hands?
a. remove diamond ring
b. remove wedding ring
c. put waterproof plaster
d. wipe her finger with alcohol

1492. 7 yers pedo present to clinic with asymptomatic deep caries lesion , during caries removal you did a pulp
exposure , when you try to perform pulpotomy there is no bleeding, why ?
a. Healthy pulp


b. inflamed pulp
c. Necrotic
d. it’s normal


1493. 12 years old with an ankylosed primary molar with no successor, what should you do?
a. Refer to pedo
b. Wait until 18
c. Extraction
1494. Pt complained of anaesthetic metal appearance on the upper central implant , POB and 3 pockets were there with
saucer-shape bone loss around the implant, what is the diagnosis?


a. Peri-mucositis
b. Peri-implantitis
c. Buccal perforation
1495. Antibiotic for oroantral communication
a. Penicillin


b. Metronidazole
c. Amoxicillin with clavulanic acid
d. Clindamycin



1496. Pt with genital and skin lesions and oral involvement , what is the diagnosis?
a. Behcet's disease

1497. 12 year old child with 4 mm diastema, unerupted mesiodens, unerupted laterals, low frenal attachment. best tx?


a. frenectomy then extract mesiodens
b. Extract mesodens then fixed
c. Wait till canine erupt then frenectomy
d. Wait till canine erupt then extract mesiodens


1498. Where should the obturation stop: -
a. Minor apical constriction.
b. Major apical constriction.
c. Radiographic apex
d. Anatomical apex


1499. child ingest full toothpaste what is the management ?
a. advice to drink milk
b. Induce vomiting
c. Hospitalization
1500. A patient had pain in the upper right teeth early in the morning, after examination all upper teeth are sound .


What could be the diagnosis?
a. Myofascial pain dysfunction syndrome.
b. Sinusitis

1501. Pic Pericoronitis and in scenario 3rd molar in correct position ask about management ?
a. Operlectomy
b. Other options are with extraction
c. Antibiotics
d. extraction
1502. treatment of Exposed bone in pt under bisphosphonate ( they write zoledronate i think instead of zometa ) no
purulent no pain ??


a. Bone resection
b. Mouth wash
1503. Pocket depth equals Clinical attachment when ?
a. Pocket depth 1 mm above CEJ
b. Pocket depth 2 mm above CEJ


c. Pocket depth 3 mm above CEJ
d. Pocket depth at the level of CEJ


1504. pic of implant retained crown with recession asking about the reason?
a. Remaining cement

1505. Patient has sarcoidosis and you take a biopsy from swelling of a minor salivary gland. What will you see?
a. Fibrosis
b. Acanthosis


c. Acinar hypertrophy
d. Granulomatous inflammation

1506. Patient presented with non-scrapable White corrugated patches on the buccal mucosa close to the corner of the


mouth that does not change when stretched. What could be the diagnosis?
a. White sponge Nevus
b. Biting sign

1507. Ortho patient came with elongated wire with multiple irritations to buccal, tongue, lips. What will you do if he
have appointment after 2 weeks ?


a. Wax
b. Cut the wire
c. Wait for the appointment
1508. Pedo with multiple caries and bad OH and had pain from 3 days and fever and cough from 1 week what type of
abuse?

a. medical
b. Neglected
c. ‫‏‬physical
d. Emotional

1509. Female patient came complaining from rough teeth surfaces, upon IOE , she has staple occlusions with mild
attrition and linea alba, pt is bruxser, what is the immediate management?


a. Reassure the patient
b. Educate the patient and construct night guard
c. smooth the teeth roughness
1510. Pt long scenario bald tongue? (Atrophic glossitis)


a. Serology
b. exfoliative cytology (biopsy)
1511. pt 7 years has Ant open bite , Always tongue out , occasionally thumb sucking and rarely tongue thrust while


swallowing what is the cause of open bite ?
a. Thumb sucking
b. Always tongue forward position
c. Swallowing while tongue thrust


1512. Cause of internal resorption


a. Dento alveolar abscess
b. Inflammation in coronal pulp


1513. Subluxation :
a. increased mobility, no displacement
b. No mobility , no displacement


1514. 50 years old male pt came for recall annual of his implant with pain , bleeding on probing and radiograph
showed bone loss form crest 2.5 mm
a. Peri implantits ✅
b. Peri implant mucositis


1515. Burning sensation on upper complete denture cause pressure on
a. Invasive foramen
b. Palatal rugae
1516. 55 years old male , with leukoplakia on tongue, what is your management ( not mentioning if he is smoker or
time ) ?


a. Biopsy
b. Observation


1517. Working length of k file
a. 16
b. 25


1518. enamel , dentin fracture name
a. Uncomplicated fracture


1519. caseating granuloma in lung
a. TB
b. Sarcoidosis


1520. pedo patient did SSC complaining of pain and bleeding clinically showed 2 mm high of occlusion , xray showed
overhang (no xray provided)what is cause of complaint: (Mentioned Before)


a. Inadequate occlusal reduction
b. Improper Size selection

1521. Most common bacteria cause IE?(Mentioned Before)
a. Streptococci
b. Staphylococcus
c. Enterococci

1522. Which of the following values is a major consideration in dental treatment of patients with HIV


infection?(Mentioned Before)
a. CD4
b. CD8
c. Basophil
d. Eosinophil


1523. side effect of HAART of HIV? (Mentioned Before)
a. Xerostomia
b. Hairy leukoplakia
c. oral Wart

1524. picture of clinic light and asked type of disinfection :


a. Put plastic sleeve and disinfectant between patients


b. Put plastic sleeve and disinfectant at end of day
c. Disinfection between patients (plastic sleeve if used should be replaced witch every patient)
d. Disinfectant at end of day

1525. you will placed resin composite restoration and remaining dentin thickness is 1 mm? (Mentioned Before)
a. RMGIC base


b. calcium hydroxide liner
c. Calcium hydroxide liner and RMGIC base
d.
1526. case of patient has osteoporosis and taking drug , came with severe pain and need extraction,how you will give


antibiotic? (Mentioned Before)
a. If there is sings of infection
b. 7 days before and continues to 14 days after
c. Prophylactic 1h before procedure

1527. type of resorption of using intra canal bleaching? (Mentioned before)


a. Internal


b. External replacement
c. External cervical
1528. Alveolar fracture splint?


a. 1-2 weeks
b. 2-3 weeks (Best answer is 4 weeks)
c. 7-8 weeks

1529. Best occlusal scheme used for complete denture and provide aesthetic?(Mentioned Before)


a. Bilateral balanced
b. Lingualized
c. Monoplane


1530. Pedo patient 20 kg , paracetamol 160/5 ml ,what is dose in ml
a. 6-9ml
b. 11-12

1531. Pt.have cl II relationship and retroclined lower incisor :


a. Frankel
b. Twin block
c. Herbset


1532. primary with gingival abscess the pt took antibiotics what is the condition of the following


a. reversible pulpitis
b. irreversible pulpitis
c. hyperemic pulp
d. obliterated pulp


1533. Most important for functional appliance
a. skeletal age
b. stage of dental development


1534. Scaling of implant


a. Carbon instrument
b. plastic instrument


1535. Signs of incomplete pulp removal in pulpotomy


a. Pain
b. uncontrolled haemorrhage


1536. 13 years old with cross-bite thumb suking open bite


a. hass
b. quad helix


1537. #22 had trauma 20 years ago no signs or symptoms it does not respond to cold test from x ray the canal is very
calcified what is the prognosis of non-surgical end
a. Good
b. fair✅
c. poor
d. Hopeless


1538. What is the muscle precied during IANB
a. Lateral pterygoid
b. buccinator


1539. Child swallowed a whole adult toothpaste what is the immediate management
a. advice to drink milk
b. axminster ipecac syrup
c. administer activated charcoal


1540. 8x5 cm radiolucency in body of mandible
a. Incisonal biopsy
b. Follow up


1541. 7 years old limited mouth opening mandible is not moving


a. arthroplasty
b. jaw exercise


1542. What is considered a parafunctional habit?
a. Erosion


b. Chewing
c. Clenching
d. Abfraction


1543. Nosocomial infection common Route?
a. Direct contact
b. Door handle
c. indirect contact


1544. Advantage of GIC with SCC ?
a. Fast setting
b. Low compressive strength
c. Soluble


1545. Property of zinc oxide eugenol in pulpectomy of primary molar:
a. slow resorption


1546. external resorption?
a. The radiolucent move with angle change


1547. Pinpoint pulp exposure :


a. CaoH liner
b. Or MTA


1548. relevant information, and knowledge of patients and families, colleagues, and other professionals


1549. apertognathia (open bite) usually common with?
a. cleft palate
b. hypognathia of maxilla


c. prognathia of mandible
d. maxilla vertical excess


1550. Implant in lateral 5 mm bleeding and deep pocket 5 mm The crown is stable no mobility but slightly with
horizontal percussion , good OH What cause ?


a. poor OH
b. space between implant and tooth
1551. DM ,facial right swelling , badly deca #46 , allergy from penicillin


a. RCR
b. RCT + clindamycin
c. RCT +amoxicillin


1552. Severe periodontitis with ortho
a. force more apically
b. force in the centre of the tooth


1553. Lowe primary central loss , low alkaline phosphatase


a. papillon lefèvre syndrome
b. hypophosphatasia


1554. Mild facial staining and md incipient caries Preparation for more esthetic
a. facial
b. lingual
c. occlusal gingival


1555. Bur for groove when tooth preparation


a. inverted bur
b. tapered fissure


1556. Which of the following treatments of paroxysmal hemicrania help its diagnosis?

1557. White yellowish lesion in oropharynx area, Dx?
1558. The patient is scheduled for orthognathic, had apertognathia what made him eligible for surgery.?


1559. Vertical maxillary excess ?


1560. Maxillary hypoplasia?


1561. Patients have multiple painless growth in Buccal mucosa, palate and tongue mother said that the family has the


same thing?
a. Heck's disease
b. Squamous papilloma


1562. Vital tooth Which material to avoid


a. bis Acrylic
b. poly meth meth
c. poly eth meth
d. poly vin meth meth

1563. Moisture contaminant powder will result in


a. tear in impression
b. grainy impression
c. chalky cast
d. distorted impression


1564. Pt have grade 2 furcation Generalized horizontal bone loss?
a. Extraction
b. Plasty


c. Resection
d. GTR


1565. Patient had gum recession, on the anterior tooth, and had a very small resto the same tooth and pt is what would
be aesthetically important to consider?
a. Vertical hight of gum
b. Bucca lingual of gum
c. Mesiodistal width of restoration
d. MIC
1566. Pt wants to restore centrals In which scheme they will touch?

✅🔁
a. MIC
b. Protrusive ( restorations could be too weak to go through protrusive )
c. CO


1567. Isolated pocket, negative cold test Xray Extensive bone loss Minimal resto?
a. Perio endo
b. Endo perio
c. True compained

1568. X Ray missing 6 and 4mm bone height, the doctor noted to the patient that he needs bone argumentation which
was done.. Next step


a. internal sinus lift
b. external sinus lift


1569. Dr was called to check in 1 day old child with a tooth
a. Natal
b. Neonatal


1570. malocclusion، and ask about the most prevalent?
a. class I malocclusion


1571. submandibular space infection
a. Extra oral drainage
b. Intra oral drainage


1572. The radio opacity of the MTA is related to which?
a. Bismuth oxide


1573. Most commonly child abused type that is related to orofacial?
a. physical abuse
b. sexual abuse
c. psychological abuse
1574. class III restoration which approach is more esthetic,


a. Facial
b. Lingual


1575. minimum metal thickness of metal rest in mm
a. 1
b. ⁠2
c. ⁠3
d. ⁠4
1576. Pic of missing 14,15,16, and 47, there is only 3 mm inter arch space


a. fixed fpd
b. ⁠removable
c. ⁠implant
1577. patient complaining of discoloured 12 how to tx


a. internal bleaching “vital tooth!!’
b. ⁠veneer
c. ⁠crown

1578. supracrestal perf and asking the prognosis of the tooth


1579. Pt has end stage renal what is the most sign you'll see or smell like that
a. halitosis
b. aphthous ulcer

1580. Pt came to the ER with pain 37 x ray showed difficult anatomy. What is your management?(Pulpotomy and
partial pulpotomy were not in the options)


a. rct
b. nsaid
c. extraction
(if there is pulpotomy and referal it will be the best option)
1581. restoration with the most glass substrate


a. lithium disilicate
b. feldspathic
c. leucite reinforced ceramic


1582. what type of anemia Mcv 90 (within normal) Reticulocyte ( either below or more than normal)
a. iron
b. folate


c. vit b 12
d. g6pd

1583. 14 y Crown fracture involving enamel, dentin and pin point expose the pulp 1 day , treatment ?
a. RCT
b. DPC


c. IN DIRECT PULP CAPPING
d. Cvek pulpotomy


1584. Pt diagnosis TB 4 day ago ER need to prio treatment
a. Hand scaling root planing
b. Ultra scaling and root planing


1585. stage 4 cancer patient with questionable teeth what is the treatment?
a. RCT
b. extraction


1586. Dens Evaginatus occurs mostly on which tooth ?
a. Permanent Lower 2nd premolar


1587. ISO Diameter of K file :
a. 0.02


1588. Patient came to you and he has NUG he is very feverish and his high temp :
a. Supra scaling and antibiotic
1589. Pic of a very very small point of pulp exposure before three days in the central of 7 years child ?


a. Pulpotomy
b. Cervical pulpotomy


1590. How should the technician set up posterior teeth for patient with skeletal class II ?
a. upper slightly palatal


1591. In Gow-Gates technique, anesthesia is directed towards?
a. Lateral side of condylar neck

1592. 16 yo female pt with swelling extra orally after a dental procedure yesterday (with picture buccal to sub
mandibular swelling girl) With sudden pain and normal vital signs
a. incision and Drainage


b. incision and Drainage and antibiotic
c. antibiotics + analgesia
1593. asthmatic pt we want to give him LA WHAT is the substance that make asthmatic attack from LA MATERIAL
a. articain


b. lidocaine
c. Na sulfate
d. epinephrine

1594. pt came with mucocele what is the management


a. marsupialization
b. Enucleation
c. follow up
d. surgical incision


1595. pic of anterior implant with coping and patient want aesthetic to represent the same teeth
a. customised coping impression

1596. pt that make coronary stent 3 years ago And taking 325 aspirin want to do extraction
a. antibiotics 1 h before the appointment


b. stop aspirn 5-7 days
c. just extraction

1597. pt with low social status came with clinical picture ( gingivitis ) and histological findings was Granulomas tissue


on apical central incisor
a. pemphigus vulgaris
b. crohn's disease


1598. -if you want to evaluate the carbide burs survival and effectiveness (previously mentioned)
a. high speed before contact the tooth
b. low speed before contact the tooth
c. high speed out of oral cavity
1599. The pt have anterior implant with black line and she concern about aesthetic What is the abutment material
a. titanium


b. alloy
c. zirconia abutment (UCLA)
1600. pt came with loss of sensation and not able to open the mouth after a dental procedure Yesterday What is the


case
a. injection LA inside posterior superior foramen
b. broke the needle
c. LA injection inside the maxillary sinus
1601. lower 6 tooth they have done perio surgery for it and they need to do again based on periodontist (xray with
distal bone resorption and there is no second molar )What is the incision
a. semilunar


b. submarginal
c. intra sulcular
1602. Pt came with several pain and the Tx is RCt and crown the pt said that the crwon not covered under assurance
and he can't afford and he want to save the tooth ?


a. dismissed pt
b. take other options
c. treat the pt just the procedure that covered by assurance
d. pulpectomy and instrumentation and dismiss pt


1603. pt had trauma the tooth moved palatal with bone fracture ? What is the fracture?
a. intrusion
b. extrusion
c. lateral luxation
d. subluxation

1604. pt on bisphosphonate have pain and pus discharge and exposed bone


a. mouthwash
b. Antibiotic


1605. pt pedo with missing 85 and distal shoe space maintainer on 46 with inflammation and mesial eruption And 36


didn’t erupt
a. reverse band and loop
b. lingual arch
1606. Patient came for emergency treatment , he has symptoms of fever and cough , the next day the doctor and
assistant develop the same symptoms? What should they do next time to avoid it ?


a. Aspiration
b. Wash hands , wear gloves and PPE
c. Avoid sharp instruments
1607. Pt is complaining from gingival bleeding that happened after the dentist did a gingivectomy by laser.What


happened?
a. violation of biological width


1608. Where should the obturation stop: -
a. Minor apical constriction.
b. Major apical constriction.
c. Radiographic apex.
d. Anatomical apex


1609. child ingest full toothpaste what is the management ?
a. advice to drink milk
b. Induce vomiting
c. Hospitalization
1610. A patient had pain in the upper right teeth early in the morning, after examination all upper teeth are sound .


What could be the diagnosis?(mentioned previously)
a. Myofascial pain dysfunction syndrome.
b. Sinusitis
c. TMD


1611. ( Case scenario and lap test ) Rheumatoid factor more than 12 and ‫‏‬pain in joints, what’s the diagnosis?
a. Rheumatoid arthritis


1612. Pic Pericoronitis and in scenario 3rd molar in correct position ask about management ?
a. Operculectomy
b. Other options are with extraction
c. Antibiotics
d. extraction


1613. ( Case scenario ) Gagging pt how to accelerate alginate impression? (mentioned previously)
a. Hot water


1614. cleft lip?(menstioned previously)
a. Failure of fusion of medial nasal process and maxillary process
b. Failure of fusion of lateral nasal process and maxillary process
c. Failure of fusion of medial nasal process and lateral nasal process
d. Failure of fusion of medial nasal and lateral nasal process and maxillary process
1615. the process of killing microorganisms ?(mentioned previously)


a. Disinfection is reducing microorganisms but not spores
b. Sterilisation killing all microorganisms including spores
c. Antisepsis


1616. post crossbite and open bite and thumb sucking appliance?
a. Quad helix
1617. pic of periodontal abscess with pus discharge (mentioned previously)


a. RCT
b. incision and drainage
1618. To have a good antimicrobial effect from using calcium hydroxide it should be placed in the canal for
?(mentioned previously)
a. 1 Hour


b. 1 days
c. 1 weeks
d. 1 month


1619. Ask about the cyst between molar teeth with vital teeth ? (mentioned previously)
a. Stafne bone cyst
1620. Patient presented with non-scrapable White corrugated patches on the buccal mucosa close to the corner of the


mouth that does not change when stretched. What could be the diagnosis?(mentioned previously)
a. White sponge Nevus
b. Biting sign
1621. Patients with rheumatoid arthritis and DM takes metformin, methotrexate and corticosteroids have oral lesions
Which of these causes the lesion?


a. Corticosteroids candida infection
b. Methotrexate ulcer
c. Metformin
1622. Ortho patient came with an elongated wire with multiple irritations to buccal, tongue, lips. What you will do he
have appointment after 2 weeks ?(mentioned previously)


a. Wax
b. Cut the wire
c. Wait to tha appointment
1623. .Pregnant patient came she's on her 2 nd trimester , she has 2 teeth that there was sever pain radiating from them:


a. Single x ray
b. Start treating the 2 teeth In effort to relieve the pain
c. Wait to after Birth
1624. Pt long scenario bald tongue?(mentioned previously)


a. Serology
b. exfoliative cytology


1625. long scenario mention that the lesion when stretched disappear when stretch and he is smoker?
a. follow up
b. Smoking cessation


1626. Ortho instruments need sterilisation?
a. Ortho plier


1627. Chronic exposure of fluoride affects?
a. Dental
b. skeletal


1628. 121 times of sterilisation?(mentioned previously)
a. 15
b. 10
c. 20
1629. Rest above occlusal level?


a. Rotation
b. Tipping


1630. Patient with Stent do you give Ab ?
a. No we don’t give
b. Amoxicillin


1631. Patient wants to place an amalgam of the remaining dentine 2 mm. What is the linear to use ?
a. No need
b. GIc
c. Caoh
d. RMGlic


1632. Major aphthous ulcer And the patient did not respond to local steroid what to give?
a. Systemic steroid
b. Local delivery of steroid
c. Systemic Anti fungal
d. Local antifungal
1633. Material for functional imp?
a. alginate
b. Pvs


c. Tissue conditioner
d. Compound


1634. Pedo mixed dentition with deep caries:
a. horizontal bitewing
b. Vertical bitewing
c. PA
d. OPG
1635. Pt anxious and history of stage 3 COPD, and severe dental pain he started amoxicillin 3 days ago but no
improvement and he is on medications: bronchodilator theophylline
a. add erythromycin


b. Add clitthrom
c. Low does diazepam
d. Nitrous oxide
1636. Pt after one day of the procedure of MOD amalgam complained about numbness in the upper left:
a. needle breakage


b. Allergy
c. Greater palatine
1637. Pt missing 21 and does not want extensive treatment:
a. cantilever fixed


b. Fixed 3 unit
c. Maryland


1638. Class V and RDT <0.5 what liner will use?
a. calcium hydroxide
b. Rmgi
1639. X-ray, homogenous radiopaque no RL rim the tooth is asymptomatic and vital.


a. osteoblastoma
b. Idiopathic osteo.
c. Condensing ostitis
1640. Differentiate between Bell's palsy and Ramsay Hunt syndrome?


a. Acute otitis media
b. Varicella zoster infection in the ear


1641. Poorest prognosis of root fracture:
a. apical


b. Middle
c. Cervical


1642. Better prognosis of root fracture:
a. apical
b. Middle
c. Cervical
1643. Gutta-percha extruded, why?
a. short root


b. Type of obturation tech
c. No control on material
d. Long root
1644. Pt taking Zometa and has pain and pus & purlient from the extracted socket, what is the management?
a. AB


b. Mouthwash
c. Resection
1645. Implant with recession and probing depth 4-5 mm and 1st thread clear, diagnosis?
a. Peri implants
b. Mucositis
c. Mucosal recession
1646. the patient used betel nut came with pain and limited mouth opening what to do?
a. surgery
b. intralesional steroids (if mild)
c. split fibrous tissue (if moderate to severe)
1647. Premolar endodontically treated and has full coverage crown, they mention the diagnosis on the Q ( vertical root


fracture ), and ask about the cause and there is an x-ray:
a. wide and short root
b. Poor quality RCT
c. Overload
1648. Patient missing 38,37,36,35?


a. direct retainer on 43
b. Indirect retainer on 43
c. Horse shoe major connector
1649. Diabetic pt controlled and recently diagnosed with rheumatoid arthritis and takes corticosteroids and ...., what


happens if he continues on these medications:
a. increase HbA1C


1650. MCV below 80:
a. Iron deficiency anemia


1651. The is just going through final exam week and has foul older what is the 1st thing u do:
a. remove pseudomembrane
b. Superficial scaling
c. AB
d. Measure Pd
1652. Pt wants crowns but she has bleeding and pocket depth ranges from 2 to 3 what is the appropriate to do?
a. polishing and prep


b. SRP every 4 week
c. SRP and polishing and reevaluate


1653. Doc took bitewing and measured the distance from CEJ to crest of bone was up to 2mm. What does that mean?
a. normal

1654. Lower molar with grade III furcation involvement what is the best way to preserve tooth?
a. odontoplasty


b. Tunneling
c. Hemi section
d. Root amputation

1655. During endo procedure file No. 35 broke in the apical part of the mesial root and bypassing & retrieval were
failed, what is the appropriate management?
a. extraction


b. Apical surgery
c. Obturate to the level of broken file

1656. 14 Y.O. pt wants to do a diamond on her tooth and says all her friends have it but her parents, how to deal with
it.(previously mentioned)
a. give her an alternative option as bleaching
b. Do it as she wants it
c. Don't do it until she brings their signed consent form
d. Don't do it until one of her parents is with her✅

1657. How deep Toothbrush bristles can get into the sulcus:
a. 0.5 - 1 mm
b. 1 - 2
1658. Pt wants to extract the lower 3rd molars and there is bone loss around 7:
a. extract 3th


b. Extract
c. Coronectomy of 8
1659. 2 implants, in 2 stage one of them was good, and the distal one exhibited mobility and crestal bone loss. What is


the recommended treatment?
a. Remove the distal implant
b. Put implants on functional load
c. Do vertical bone graft by using .. mesh
1660. Pt has had anterior fixed crowns for 6 years and recently before 3 months, she noticed recession and metal


appearance, cause?(previously mentioned)
a. hard brushing
b. BW violation
c. Excess cement


1661. U want to replace 14, 15:
a. 4 unit (16, 13)
b. 5unit fixed fixed
c. 5-unit non-rigid connector
d. 6 unit


1662. Cbct of symphyseal fracture:(previously mentioned)
a. 2 plates
b. 1 superior
c. 1 inferior
1663. 14 Y.O Pt had trauma on the central with a complicated fracture and about 2/3 of the crown ( they don't mention
how many hours before he is seeking the treatment or the size of the exposure) and what is the appropriate treatment
option:


a. DPC
b. Pulpotomy
c. RCT
1664. Well-controlled diabetic pt, came for an annual follow-up for her complete denture and doesn't have any
complaints during IE there is pinpoint elevation in the palate, what is the treatment?


a. antifungal
b. Instruct pt to wear it only during daytime
c. Remove it until it heals


1665. Pic of odontoma and ask about treatment:
a. excisional
b. Excisional with safety margin
c. Resection


1666. Pt on prolonged use of cortisone will undergo simple extraction and he isn't afraid:
a. daily dose
b. Double dose of
c. 25 mg
d. 50 mg
1667. Pt very anxious and becomes dizzy and dilated pupils and is unconscious, how you sit him:


a. upright
b. Supine

1668. What is the systemic condition that affects chewing on complete denture wearers?


a. xerostomia
b. Diabetes
1669. What is the effect of Sjogren syndrome on oral health?
a. doesn't have an effect


b. Increase rate of root caries
c. Increase periodontal disease and caries


1670. What syndrome comes with rheumatoid arthritis?
a. Sjogren syndrome

1671. Most common route of infection in dental practice?


a. air borne
b. Direct/indirect
c. Something with vehicle
1672. The child will receive liver transplantation and has visible plaque and enamel defects, what will give him


varnish?
a. every 3 months
b. 6 m
c. Contra
1673. Pt will do whitening and has discoloured class III restorations, when will replace it?(previously mentioned)


a. after 2 days
b. After 2 weeks
c. Same visit
1674. Endodontic Hand operator instrument:


a. gates gliden
b. barber broach
c. ⁠lentilospiral
d. pesso reamer
1675. Why use lentulo spiral in post and core procedures?

1676. apply separating medium
a. For cement
b. Remove gutta-percha


1677. PPE:
a. gloves, mask, gown, eye protector
b. Head covers, mask, gown, eye protector
c. gloves, face shield, gown, eye protector

1678. Pt came to the clinic and had a cough and yellow sputum, latent TB, The doctor suspected TB infection. How


many hours will be in the clinic?(previously treated)
a. up to 4 hours
b. Up to 2 hours
c. Min
d. Sec


1679. Pt with the reduced vertical dimension with pic Ask about the cause? Pic short teeth due to attrition
a. Tooth-to-tooth contact


1680. ART is considered what?
a. Conservative
b. Preventive


1681. what is the relationship between stress and psych with periodontitis?
a. Stress leads to poor oral hygiene
b. psychic pt are more prone to periodontitis


1682. full mouth rehabilitation for pt, what type of occlusion?
a. Canine guidance
b. Group function
c. bilateral


1683. Pt did a new crown 1 week ago, and came because of pain related to the tooth, what is the management?
a. Remove crown
b. RCT
c. Desensitizing agent
d. DPC
1684. pt did new upper anterior crowns and came back after 4 weeks with erythema , bleeding and inflammation and


back odor what is the cause
a. Biological width violation
b. Allergy
c. plasma cell gingivitis


1685. Pt suspected VRF what is the initial radiograph?
a. PA
b. CT
c. BW
1686. long case pt had sinus tract in his upper teeth and after doing percussion test to 25,26,27 the only one with pain


was 26 , what are you going to do?
a. sinus tract tracing
b. PA
c. Ept


1687. Pt has pain in his teeth with cold drinks what is the first thing u will do?
a. cold test
b. EPT
c. percussion
1688. pt had 3 unit fpd try in , while trying it there was torque in the retainer , what is the management ?
a. fix the retainer part

b. Redo the whole fpd
c. sectioning of the fpd
1689. What are the factors that determine the forces directed to alveolar ridge and tooth?


a. Direction and magnitude of force
b. Type of denture base
c. Type of clasp
1690. cancer pt taking IV bisphosphonate , hypertensive drug and other drugs , what is the absolute contraindication for


implant?(previously mentioned)
a. IV bisphosphonate


1691. HIV pt that is taking HAART , and had hairy leukoplakia, what does that mean?(previously mentioned)
a. HIV is not controlled ‫يعني المناعة ضعفت‬
b. HIV is controlled
1692. lung cancer pt that finished 4 cycles of chemo , he had a tooth that is carious ( I forgot if it needed extraction or


RCT),blood test was done Platelets and WBC were low, what is your management?
a. prophylactic antibiotic
b. platelet transfusion


1693. pink color on the crown and little on the tooth(previously mentioned)
a. internal resorption
b. pulpitis
c. external resorption
1694. long case and vital signs were shown to have low heart rate and low respiratory rate, what is your management?
a. Epi 100:000…..
b. diphenhydramine


1695. Pt had deep bite and his upper anteriors are showing 100% what is your management?(previously mentioned)
a. lower anterior intrusion
b. upper anterior intrusion
c. upper posterior extrusion
d. lower posterior extrusion
1696. nosocomial infection common route?(previously mentioned)


a. door handle
b. direct contact
c. indirect contact


1697. tooth with 5mm Pd and 2mm recession what is the cal?(previously mentioned)
a. 7mm
b. 3mm
c. 2mm


1698. ovate Pontic , tissue surface cause irritation?(previously mentioned)
a. nickel chromium
b. acrylic
c. gold
d. resin
1699. picture of inflamed gingiva (gingivitis)Asking about type of bacteria?(previously mentioned)


a. gram -
b. gram +
c. staphylococcus
1700. case of a smoker (17 years of smoking) with grey, red and white color on the palate , what is the


management?(previously mentioned)
a. follow up (nicotine stomatitis)
b. excision biopsy

1701. space analysis for pedo (previously mentioned)
a. Tanaka and jhonson


1702. gow gates , were to inject?(previously mentioned)
a. lateral condyle neck


1703. Flabby ridge impression (previously mentioned)
a. Mucostatic
b. Muco-compressive
c. admix


1704. burs sterilisation(previously mentioned)
a. perforated trey submerged with sodium nitrate 2%
1705. pt 9 years old fully erupted 21 , 11 missing/delayed eruption, what is the cause of benign
delayed/missing?(previously mentioned)


a. congenitally missing
b. fibrous tissue
c. delayed eruption


1706. long case about seizure , how to diagnose? (previously mentioned)
a. EEG
b. ECG
c. Doppler test
1707. A patient came for constructing a new set of CD, came with inflamed traumatized mucosa, what to


do?(previously mentioned)
a. Reline denture with tissue conditioner
b. Rebase denture
c. instruct patent to not wear denture during sleep time


1708. VRF is what(previously mentioned)
a. deep narrow isolated pocket
b. deep wide isolated pocket


1709. pontic design hard to clean(previously mentioned)
a. ridge lap
b. modified ridge lap
c. ovate
1710. Facial nerve motor intervention?(previously mentioned)
a. Masseter


b. Temporal
c. Buccinator
1711. Hierarchy of control: what is the next step If the administrator can't afford elimination of hazard?(previously


mentioned)
a. Substitute hazards

1712. old pt came with amalgam restoration and wants to replace it with esthetic restoration. He has abnormal salvation
and, deep bite, what do you consider to prevent the failure of the final restoration?
a. pulp health


b. occlusal adjustment
c. impression tech
1713. What type of ex-ray you will do in avulsed pt? (previously mentioned)
a. occlusal


b. opg
c. cbct and 2 pa
1714. most retentive pins in amalgam(previously mentioned)


a. cemented
b. self threaded
c. friction


1715. Rest seat prep(previously mentioned)
a. 1mm
b. 2mm
c. 3mm


1716. Sodium hypochlorite ph(previously mentioned)
a. 5


b. 7
c. 11


1717. Prep of ceramic crown how much to remove from incisal (previously mentioned)
a. 2mm in functional and 1.5 in nonfunctional
b. 1mm
c. 0.5


1718. Pseudo class 3 (previously mentioned)
a. -skeletal 1
b. -molar 1


1719. During pulpotomy there is dark blood thus indicate?(previously mentioned)
a. irreversible pulpitis
b. necrotic pulp


1720. pic of upper lateral broken it was build up with post ask about the treatment (previously mentioned)
a. Extraction and implant
b. Redo the post and crown
c. Ortho extrusion
1721. Facebow consider (previously mentioned)
a. Semi critical
b. Non critical ✅
1722. you have three canal , MB ML D at the end of instrumentation you couldn't negotiate the ML And in the
radiograph there was nothing?(previously mentioned)


a. Perforation
b. Ledge
c. Separated file


1723. Use of prophy jet(previously mentioned)
a. For implant
1724. Pt came complin of broken tube and irritating wire and his appointment after 2 weeks?(previously mentioned)


a. remove the wire
b. cut the excess
c. tell him to wait


1725. How to check the effectiveness of IANB?(previously mentioned)
a. Canine
b. First molar
c. Second molar
d. Third Molar


1726. Which tooth has a consistent number of canals?(previously mentioned)
a. Maxillary Canine(if there is maxillary central will be the most correct)


1727. Lesion 4*4 cm how to treat?(previously mentioned)
a. Marsblization
b. excision
c. Enucleation


1728. Chest ex-ray shows granuloma what is the diagnosis?(previously mentioned)
a. Tb
b. Copd
c. Asthma
1729. Child have infection and he will do an extraction (previously mentioned)


a. 2%lidocaine
b. 4%articaine
1730. lower anterior recession and high frenum and no keratinized gingiva?(previously mentioned)


a. frenectomy with free gingival graft
b. frenectomy with ct graft
1731. Pt with multiple with discolouration white patch covering all the teeth poor oral hygiene and diet with
carbohydrate mother said she has the same:(previously mentioned)
a. Amelogénesis imperfecta


b. Dentinogénesis imperfecta
c. Dental caries
1732. Splint for dentoalveolar fracture?(previously mentioned)


a. 2 weeks
b. 3-4weeks
c. 1 week


1733. Gag reflux will cause?(previously mentioned)
a. Erosion
b. Abrasion
c. Abfraction


1734. Myxedema coma?(previously mentioned)
a. Hypothermia
b. Hyperthermia
c. Irritablity


1735. Why remove the smear layer before obturation ?(previously mentioned)
a. To allow the sealer to go into open dentinal tubules
b. To remove the sealer easily
c. To increase the antimicrobial activity of the seale


1736. Appliance used to retain crossbite after treatment?(previously mentioned)
a. Hawley
b. Quadhelix
c. Essex


1737. Scenario of sudden allergy after rubber dam(previously mentioned)
a. type 1 allergy


1738. Biopsy to the lab?(previously mentioned)
a. Put it in a biohazard bag with formalin
b. Put in in a biohazard bag with saline


1739. Pedo fall in his chin, what probable fracture?(previously mentioned)
a. Condyle
b. Angel
c. Symphysis
1740. Referral from prosthodontist for crown lengthening , when he did the flap the margin was 1mm from the bone
What should he do?(previously mentioned)


a. Close it and suture
b. Remove 2mm more


1741. Inflammatory periosteal reaction(garres syndrome)(previously mentioned)
a. Onion appearance
1742. Pt extracted a tooth 2 days ago and bleeding is still present even after closure of suture, what is the appropriate
management?


a. Gelfoam
b. Surgicel
c. Alveogyl
1743. Human papilloma virus associated with cancers.(previously mentioned)(previously mentioned)
a. Salivary gland
b. Skin


c. Bone
d. Oropharyngeal
1744. Pt with thyrotoxic crisis, most common cause ? (previously mentioned)


a. Nystatin
b. Epinephrine
c. Lidocaine
d. Amoxicillin
1745. Question about undercut in anterior region and maxillary tuberosity and you want to fabricate complete denture,
what is your management? (previously mentioned


a. Surgical removal all undercuts
b. Retain all undercuts helps in retention
c. Relief of posterior undercuts
1746. Clamp for Upper molar? (previously mentioned


a. W7 (this is universal)
b. W8
1747. Pt came with white wrinkle buccal mucosa and had history of chewing tobacco for 10 years what is the


appropriate management? (previously mentioned
a. Reassure pt Immediate biopsy Quit habit and reassess
1748. Pt has headache happens at night for 10 minutes behind the orbit and anterior maxilla with temporal area


(previously mentioned
a. Cluster headache
b. Giant cell arteritis
c. Migraine
1749. Needle stick injury dentist and after all procedures were done (encourage bleeding, washing wound, applying
plaster) what is the next step? (previously mentioned
a. Ask the pt to bring blood tests next appointment


b. Continue treatment
c. Report the incident
1750. HBV stays at room temperature for;
a. 1 month


b. 4 hours
c. 7 days
d. 8 months
1751. Pt has inflamed gingiva, bleeding on touch, foul odour (other things I don’t remember) The main difference


between necrotising ulcerative gingivitis and desquamative gingivitis?
a. Foul odour
b. Bleeding
c. Inflammation
1752. 4 years old had trauma and both primary incisors intruded and touched the permanents, what is your
management?


a. Allow for spontaneous eruption
b. Extract both teeth
c. carefully Reposition and splint
1753. Picture with crown on #46 and an open margin mesially and distally, what is the cause?


a. Over tapered preparation
b. Shrinkage during casting
c. Distortion of final impression
d. Expansion of core
1754. Case scenario, then asks a question, during lithium disilicate cementation what is the first step?
a. Sandblasting


b. Etching with phosphoric acid
c. Etching with hydrofluoric acid


1755. Internal résorption obturation technique?
a. Carrier
b. Single cone
c. Warm vertical
1756. Minimum days for extraction in cancer pedo patient?


a. 7
b. 14
c. 21


1757. Patients has erythematous bleeding on probing gingival, deep pockets 7-11mm all over teeth (attached


radiographs) management?
a. Scaling and root planning Scaling and root planning with antibiotics
b. Antibiotic then scaling and root planning after 1 week
1758. Percentage of accessory canal in apical third?


a. 15
b. 74
c. 11


1759. Most common bacteria in waterlines?
a. Legionella
b. Pseudomonas
c. Mycobacterium tuberculosis
1760. During instrumentation, file was separated but then you managed to bypass it
a. Miss Near ✅
b. Miss
c. Extreme miss
1761. Radiograph asking about the mishap:
a. Ledge
b. Short obturation
c. Stripping
1762. Most common clinical picture of thin biotype
a. Mccall festoons


b. Stillman clefts
c. Fenestration and dehiscence
1763. Dry heat sterilisation


a. 120 F for 30 minutes
b. 320 F for 120 minutes
c. 250 F for 60 minutes


1764. Case about hypertensive pt takes 2 types of drugs (forgot their names but one of them diuretic) and bp was
160/89, you prepped the tooth last visit and this appointment was for final impression What is your management?
a. Change dose of diuretic


b. Change dose of other drug
c. Take impression with retraction cord phenylephrine
d. Take impression with epinephrine
1765. Pt came for routine dental examination, lower premolar was missing and after you obtained x-ray there was a
cyst:


a. Enucleation and removal of premolar
b. No treatment (cuz its eruption cyst)
c. Marsupialization and decompression
1766. Pic of MMP

a.
1767. Pic of the Apthous ulcer and their tx


a.
b. Fluocinonide 0.05%
1768. Pic of mucocel

a.


1769. Unicystic ameloblastomas tx
a. Enucleation
1770. - solid ameloblastoma and tx
a. Removal with marginal resection
1771. X ray strip perforation

a.
1772. Monomer to polymer 1:1
a. Excess polymiraztion shrinkage
1773. V - shape attrition
a. Bracket
1774. First sign of chemical burn
a. Necrosis
1775. shock absorption characteristic of which of PDL
a. Physical
1776. Dolichcephalic
a. Long skull
1777. Conservative Access cavity on 36 on MO amalgam restoration after RCT done which is nest ?
a. Cast post ceramic crown
b. Composite core and Ceramic crown
c. Cast post and PFM
1778. xray about like hug radiolucency and said that when Dentist do cleaning and shaping excessive bleeding comes
from canal and he noticed that plastic stopper has been moved
a. Perforation
b. Sodium hypochlorite accident


1779. Diabetes pt come with pain from 46 and pus discharge and deep isolated pocket depth?
a. RCT first then perio treatment


1780. which of the following can we do standard precautions on it?
A. Infectious diseases pt


1781. how to determine the gingival suppuration
a. Place the finger on apical margin of swelling and puch coronally ✅


1782. Hypertension 135/ 80
a. Prehypertention

b.


1783. Blood pressure 67/57
A. Put head lower than feet


1784. 5 mm recession and non keratinized tissue
a. Root coverage


1785. Deciding endodontic Tx and implant
a. Tooth and periodontium related factor
1786. Pic of implant hybrid

a.


1787. Vertical release incision while GTR on # 11
a. Distal angle to #13


1788. Sensation of obstruction
a. CT
1789. 16 years old girl came with these findings: class II, excessive lower facial height, incompetent lips, excessive
maxillary growth, wide buccal corridors, excessively showing gingiva. What is the problem?


a. Crouzon syndrome
b. Mouth breather
1790. Most common of HAART with pt HIV
a. Hairy leukoplakia
b. Oral candidiasis


c. Oral warts
d. Dry mouth


1791. Case about RPD and u will replace the anterior teeth which teeth?
a. Porcelain
b. Porcelain facing acrylic
1792. Sequences after RCT and tooth need CL and RCT done on 1 mm of sound tooth structure and u have 1.5 BW
a. P/C , Crown lengthening and Crown


1793. Atrial fibrillation and he takes warfarin and the INR is 3 and he give u the normal results of INR is 0.8 - 1.2
a. Proceeds with extraction


1794. U did conservative access opening on 11 after RCT
a. Composite restoration
1795. advantage of using disposable flow tips for flowable composite
a. Minimize the possibility of air trapping in restoration
1796. Pt cone to ER with emergency and he have symptoms like cough and runny nose and the Dr and nurse second
day have the same symptoms how to avoid this?
a. Wash and use PRR


1797. Which material is preferred for base of RPD
a. Acrylic resin
1798. Supracrestal perforation how to manage ?


a. Immediately seal it with GIC
b. Immediately seal it with MTA


1799. man attended with a loose implant crown #35 torqued to 35 Newton centimetres using a torque control device
and Light occlusal contact was present in maximum Occlusal contact was also present with lateral excursions. Which
is the cause of implant crown screw loosening?
a. Centred occlusal contact
b. Undesirable lateral excursion


1800. CD and the pt is pronounced Th instead of S ?(previously mentioned)
a. Upper incisors placed more palatally


1801. minimum space for cemented retained implants (previously mentioned)
a. 7-8 mm


1802. features of active caries (previously mentioned)
a. Matte, soft and chalky


1803. Pt with every thing with normal limits except the saturation of O2 90%(previously mentioned)
a. Reschedule (should not be below 95%)
b. Treat with Treat
1804. Pt pedo with multiple surfaces lesion and heavy plaque which restoration(previously mentioned)


a. SSC
b. RMGIC
1805. U explain to the pt the complications of Tx what is it?(previously mentioned)


a. Veracity
b. Autonomy (if there is informed consent will be the most correct answer)
1806. ask about age(previously mentioned)


1807. severely resorbed ridge with tissue firm on on it(previously mentioned)
a. Admix
b. Selective pressure displacement


1808. which muscle is attached to Condylar (previously mentioned)
a. Lateral pterygoid


1809. Pt with numbness after placement of 2 implants(previously mentioned)
a. Neurovacscular bundle interferes (mental canal)


1810. mechanism of CHX ?(previously mentioned)
a. Cell membrane


1811. Pt after RPD had hypersensitive to percussion?(previously mentioned)
a. Defective occlusion


1812. most caused of Infective Endocarditis (previously mentioned)
a. Viridian streptococcus
1813. palatal tori not mentioned if small or large which connector?(previously mentioned)


a. U shape (if large)
b. Anterior posterior strap (in general)


1814. Severe COPD which position no upright position (previously mentioned)
a. Semi- supine position


1815. Position avoided with pregnant women (previously mentioned)
a. Supine


1816. patient class l but has procline anterior ?(previously mentioned)
a. hawley retainer with labial bow


1817. During instrumentation, file was separated but then you managed to bypass(previously mentioned)
a. Near miss
1818. case use medication causes gingival enlargement (Dilantin) and she has heavy plaque and calculus what is the
initial management?(previously mentioned)


a. Consult with her physician to change the medication
b. Scaling


1819. Pt with 90 MCV and Reticulocytes high?(previously mentioned)
a. GB6D (80-100 is normal)
1820. OPG with bubbles on ramus ( very clear ameloblastoma ) what is the next step?(previously mentioned)


a. Biopsy
b. FNA
c. CBCT


1821. Pt came with necrotic #45 and tooth has a plug in occlusal surface what could it be(previously mentioned)
a. Dens evaginates
1822. 13- 5mm from gingival margin to bone crest, 2mm from restoration margin to bone crest and 2mm tooth


structure above margin?(previously mentioned)
a. Sufficient ferrule and supracrestal attachment


1823. Implants and there is bleeding with probing the pocket will be? (previously mentioned)
a. Deeper than natural teeth
1824. Case about pedo pt has a Trauma before 3 days or 3 hours not sure and EPT is negative no response? (previously


mentioned)
a. Follow up
1825. Case about ortho class 2 malocclusion and have good face profile and just need to correct the deep bite and
overjet(a lot of data missing)
a. Surgery
b. Extract upper 4


1826. Indication of tissue conditioner (previously mentioned)
a. For inflammation tissue and ill fit denture


1827. The manager’s relatives or friends treated first or make their appointments(previously mentioned)
a. Use advantage his position


1828. Case about swelling and histo relieved deposits of hemosiderin pigment ?(previously mentioned)
a. peripheral giant cell


1829. 12 Pt have unilateral crossbite and reduced facial height(previously mentioned)
a. Rapid expansion


1830. Girl with neck scratches and also in palate (previously mentioned)
a. Sexual


1831. a patient with upper and lower complete denture has ulcer in frenum. What is the cause? (previously mentioned)
a. Overextended labial flange
1832. synthesise relevant information, and the perspectives of patients and families,colleagues, and other
professionals?(previously mentioned)
a. Health Advocate


b. Researcher/ Scholar
c. Communicator

1833. Avulsed tooth came after 3 days and was in plastic bag (previously mentioned)


a. rct and reimplantation
b. ⁠implant
1834. What is the best posture for a copd patient in the clinic?(previously mentioned)


a. Supine
b. Semi-supine
c. decubitus
d. Prone
1835. HIV patient with Linear gingival Erythema that didn’t resolve ,what to give him ? (previously mentioned)


a. Valacyclovir
b. Anti fungal
1836. Heavy smoker for 20 years , generalised ?brown staining ?(previously mentioned)


a. intrinsic Stain
b. Extrinsic stain
1837. Patient with white patches on teeth , consuming a lot of carbohydrates, mother mentioned she has the same


problem ?(previously mentioned)
a. Dental Caries
b. Amelogenesis imperfecta
c. Dentinogenesis Imperfecta
1838. When placing implant and you want to have interdental papilla regrowth you should leave between base of
proximal plate & crestal bone ? (previously mentioned)
a. 3.4 ✅
b. 4.4
c. 5.4


1839. Avulsion for 45min splint time ?(previously mentioned)
a. 2-3 weeks functional splint
b. 8 weeks rigid splint
1840. 7years old has 5mm intrusion whats to do?(previously mentioned)
a. Surgical repositioning


b. Ortho repositioning
c. No repositioning until 3 weeks
1841. patient came 2 days after extraction of lower right second molar with a loss of sensation of his right tongue,
injury happened to which nerve?(previously mentioned)


a. IAN
b. Lingual
c. Glossopharngyeal
d. Facial


1842. Picture with a trismus and swelling, they’re asking which space is this?(previously mentioned)
a. Sub masseteric
b. Sub pterygomandibular


1843. most restoration will have polymerization shrinkage?(previously mentioned)
a. ⁠class I


1844. Facebow considered what(previously mentioned)
a. Nont critical
1845. While doing surgery, doctor noticed his gloves was torn what should he do(previously mentioned)
a. Change torn gloves
b. Alocohol habd rub


c. Wash it with water
d. Wash it with soap and water


1846. when do we check occlusion of SSC? (previously mentioned)
a. before cement


1847. which of the following require glazing to avoid dehydration?(previously mentioned)
a. Glass ionomer


1848. Most side effect with chemotherapy?(previously mentioned)
a. Mucositis
1849. Factors which indicate the failure of implant in posterior upper maxilla ?(previously mentioned)


a. high remodelling
b. Insufficient primary stability
c. high occlusal force
d. increase blood supply


1850. The reason for acid etching before restorative treatment?
a. prevents micro leakage
b. prevents thermal exchange


1851. patient has been diagnosed with Myocardial infarction before 3 weeks and taking aspirin and he has a sever pain


in a tooth tx?
a. ‫‏‬Treat now RCT and give nitroglycerin
b. ‫‏‬Extraction with AB
c. ‫‏‬Treat after more than 90 days of MI

1852. The indication for growth modification in patient with class III skeletal ?
a. protraction of maxilla
b. protraction of mandible
1853. Which of the following materials has potential toxicity?
a. calcium hydroxide


b. ferric sulfate
c. formocresol
d. MTA
1854. An impression was made in the clinic, which of these should be done before sending it to the lab? (didn’t specify
the material that was used)
a. clean with chlorine compounds for 1 minute


b. clean with CHX
c. clean with Idophor
d. clean with Glutradlehyde
1855. A gypsum break during flasking, which of these could be attributed?
a. Exothermic reaction that led to it


b. Shrinkage of the gypsum
c. Insufficient bulk of the gypsum material
1856. A pt coming to restore his 3 missing lower anteriors, the dentist reckoned the tooth to be #32, upon what did he
decide?
a. bilaterally symmetrical


b. same mseiodetal in each side
c. insicsal slopes are inclined distally
d. tooth is off the center, and the root toward the mesial


1857. Difference between AI and DI?
a. -Genetic inheritance
b. Calcification in the root canal
1858. Dr decided to do RCT for the patient and gave him IANB + long buccal, during the procedure the patient felt


pain and the dr gave another IANB, when the dr reach close to the pulp the patient felt pain, what to do?
a. Intraligamentary


1859. Sever COPD case what to consider during RCT?
a. Not placing rubber dam


1860. What measure Modulus of elasticity?
a. Resilience


1861. Resistance of denture to horizontal force?
a. Stability


1862. Impression technique for flappy ant maxillary ridge?
a. Mucostatic
1863. Pt came to ortho clinic requested to extract #53, upon exam 53 is good with good crown size, impacted


permanent canine with poor prognosis, what is appropriate tx?
a. Leave 53
b. Crown for 53
c. Extract 53 and space closure
d. Extract 53 and implant


1864. Offset angle curette?
a. Gracey
b.


1865. Shank of the gracey curette during scaling?
a. Lower shank parallel to long axis of the tooth

b.


1866. Incision for coronally displaced flap to cover the root?
B. External bevel


1867. Simplest way to manage broken retentive arm?
a. Wrought - wire


1868. Liver cirrhosis which one is safe medication?
a. Paracetmol


1869. Liver dis pedo pt has multiple caries and plaque accumulation, asking about F supplement?
a. Every 3 mon


1870. Renal dis pedo pt has multiple caries and plaque accumulation, asking about F supplement?
a. No need


1871. Clinical pic of morsicatio or white spongy nevus and asking about tx?
a. Follow up
b. Remove the irritant


1872. Large dentigerous cyst tx?
a. Marsipulization
1873. Which is less likely to be correct about smoking and perio dis?


a. Smoker has more red and orange complex bacteria
b. Smoker has more plaque
c. Smoking causes imbalance bt bacteria and host response


1874. Pocket depth 6mm from CEJ to base what does that mean?
a. 6 mm cal
b. 6 mm psuedo
c. 2 Pd, 4 cal
d. 4 pd, 2 cal
1875. Diabetic pt (take sulfonylurea) you gave him ibuprofen, at night he called you complaining of numbness all over


his body, what happened?
a. Drug interaction
b. Hyperglycemia


1876. Pt have lesion on the lateral surface of the tongue, ulcerated and indurated what is the best management?
a. Incisional biopsy
b. Excisional
c. FNA
d. Follow up in 2 mon


1877. Pt 3y ago had nasopharyngeal carcinoma, now came with irregular radiolucency in the mandible?
a. Osteoradionecroais
b. Osteomyelitis
c. Sequestration


1878. Pic of rest on lower central without rest seat preparation, asking what’s wrong?
a. Rest should be in rest seat prep

1879. Pt on prolonged systemic steroid therapy, not anxious, came for minor surgery?
a. Usual daily dose
b. 25mg steroid pre op
c. 75 mg steroid pre op


1880. Resonance frequency analysis for what?(previously mentioned)
a. Implant stability


1881. Cancer patient what supplement to give to decrease mucositis? (previously mentioned)
a. Zinc


1882. Gp percentage in gp cone? (previously mentioned)
a. 20%


1883. Gate glidden size 2? (previously mentioned)
a. 0.7
b. 0.5
1884. pt with asymptomatic lesion on checks can’t rub off but disappear if you stretch the cheeks?(previously


mentioned)
a. leukoedema


1885. Flexure strength of 3 units bridge: 3 pontics ?(previously mentioned)
a. 27


1886. CD Patient complaining of cheek biting, why?(previously mentioned)
a. Improper horizontal overlap
1887. What will cause failure in acrylic?(previously mentioned)


a. low water absorption
b. low modulus of elasticity
c. high thermal coefficient


1888. Ibuprofen with MI?(previously mentioned)
a. arrhythmia
b. tachycardia
1889. 16 Slightly Supra-erupted and you want to place lower crown?(previously mentioned)


a. no treatment
b. enameloplasty


1890. Pt did third molar exo after 4 weeks came with pus swelling limited mouth opening?
a. -buccal cortical fracture
b. -subperiosteal abscess


1891. Pic failure of implant bcz of what?(previously mentioned)
a. insufficient keratinized tissue
b. implant size
c. recession in 34
d. not seating crown in 35


1892. tooth paste for high risk 13 years patient(previously mentioned)
a. 1450


1893. instruments used in determining the guide plan(previously mentioned)
a. analysing rod


1894. What is the main disadvantage of silver points?(previously mentioned)
a. Inadequate seal
b. Toxicity
1895. ceramic crown what core(previously mentioned)


a. Amalgam
b. Zirconia
1896. Class 1 prep how to do bevel(no burs in the opts)(previously mentioned)


a. Enamel hatchet ,
b. chisel
c. bevel hatchet
1897. standard treatment of dentigerous cyst?
a. surgiacal removal of cyst


b. surgical removal of tooth
c. surgical removal of tooth and cyst


1898. Upper bilateral missing premolars(previously mentioned)
a. Anterior palatal strap


b. A-P palatal strap
c. Palatal plate
d. u-shaped
1899. Pt uses smokeless tobacco and came to u, u noticed he has fibrous bands?? or something like that,he has limited
mouth opening, what to do(previously mentioned)
a. Topical steroids


b. Intralesional steroids
c. Surgery removal fibrosis band


1900. patient did inlay before 2 days and came with severe pain in biting
a. tooth in high occlusion
b. periodontal ligament involvement


1901. Mesiolingual gold onlay deflection when closing what is the problem or interference


a. centric relation
b. Centric stop
c. Protrusive


1902. tooth displacement buccal , lingual,mesial,distal


a. subluxation
b. lateral luxation


1903. cement needed slow mixing ?
a. Zinc phosphate
b. zinc Polycarboxylate

1904. How to disinfect the prosthesis before sending it to the lab
a. Steam autoclave


b. Dry heat
c. Glutaraldehyde


1905. Pt present to clinic with pain on biting on 45, 44 upon examination you found large amalgam restoration in
relation to two teeth best diagnostic?


a. PA
b. Percussion
c. Palpation
d. Examination and transillumination
1906. Pt with palatal recurrent ulceration and peri ocular crust …
a. Erythema multiforme


b. Pemphigus vulgaris
c. Mucos membrane pemphigoid


1907. Blood supply of floor of the mouth
a. Lingual artery


1908. Pt using Pulmicort inhalation corticosteroid his vital signs was monitored (values for BP 130/80, respiratory rate
within normal, oxygen saturation 98%) the respiratory rate improved after rest
a. Emphysema


b. Chronic bronchitis
c. Asthma


1909. pt with liver disease what test could be useful
a. PT


1910. third molar lost in the soft tissue which space(previously mentioned)
a. Submandibular
1911. severely resorbed ridge what is the impression technique the tissue of the ridge is firmly attached(previously


mentioned)
a. Admix
b. Mucostatic
c. Neutral zone
d. Selective … impression


1912. pt with diastema between to central with blenching of IDP what is theManagement(previously mentioned)
a. A.frenectomy


1913. Most common bacteria water than line(previously mentioned)
a. Legionella
b. Mycobacterium
c. Staphylococcus aureu
1914. Pt extracted a tooth 2 days ago and bleeding is still present even after closure of suture, what is the appropriate


management?
a. Gelfoam
b. Surgicel
c. Alveogyl

1915. 14 year old child has avulsed tooth , what antibiotic recommend to prescribe?(previously mentioned)
a. Amoxicillin


b. Penicillin
c. Doxycycline
1916. Patient high caries risk and depression and socially deprived asking about the toothpaste fluoride concentration
in ppm ?(previously mentioned)


a. 1480
b. . 2800
c. .500
1917. pic of space infection what is the cause of the abscess


a. 45
b. 46
c. 43
1918. year-old woman complains of a painful ulcer on her right cheek. Clinical examination showed a single
yellow-white lesion covered by fibrinopurulent membrane, which is encircled by erythematous hallo. The lesion is
about 4 mm in diameter and is located at the anterior aspect of her right buccal mucosa. Which of the following is the
best treatment for this lesion? (previously mentioned)
a. 0.50% acyclovir ointment
b. 3.00% benzocaine ointment
c. 2.00% metronidazole cream
d. 0.05% of betamethasone gel ✅
1919. Gate glidden size 2 (previously mentioned)


a. 0.5
b. 0.7
c. 0.9


1920. Pic of infra orbital fracture in CT symptom? (previously mentioned)
a. a. Double vision


1921. patients have isolated recession in lower incisor... and high frenum...what to do?(previously mentioned)
a. Connective tissues graft and frenectomy
b. Free gingival graft and frenectomy


1922. 2-Avulsion open apex after 2h ? (previously mentioned)
a. Functional splint for 4 -6 week
b. Functional splint 2 week


1923. levelling curve of Spee in growing pt, proper management (previously mentioned)
a. Anterior bite plane
b. Posterior bite plane
c. TAD


1924. 3 years Pediatric patient with multiple surface caries, what type of restoration? (previously mentioned)
a. SSc
b. RMGI
1925. How to refine the access for C shape canal ?(previously mentioned)
a. 169 L
b. #1 round bur


c. #2 round b
d. Ultrasonic
1926. Veneer falls down from the tooth and pic (all cement retained on the veneer surface) What is the cause?
a. Contamination of Porcelain surface
b. inadequate etchant for porcelain


c. old cement was used
d. dentine substrate
1927. Patient is complaining of pain in his mouth for 2 days. Medically he has a fever, accompanied by
lymphadenopathy and malaise. He is also noticing a foul odor from his mouth. Radiographs taken show normal bone


levels.what is diagnose(previously mentioned)
a. NUG
1928. .In a mandibular Kennedy class I, the usable undercut is located at the middle of the facial surface of the most


posterior abutment. Which of the following is the clasp of choice? (previously mentioned)
a. RPI
b. RPA
c. Ring
d. Back action
1929. Perio case pt has an implant. Upon examination, there is tissue inflammation and pockets and BoP. Moreover, the
buccal first serration of the implant was shown. DX? (No xray and didn't mention if there is bone loss or not)


(previously mentioned)
a. peri implantitis
b. peri-implant mucositis


1930. X-ray of endodontic mishap ( separated instrument in middle of canal ) ask about cause :
a. improper access
b. Manfactuery erro


1931. radiographs, associated with non-eruption impacted third molar: (previously mentioned)
a. Enucleation with extraction of tooth


1932. Adult Class 3 with normal mandible and deficient maxilla (previously mentioned)
a. Lefort 1
1933. Patient wants an implant for #46 interdental space of 14mm.(previously mentioned)
a. Two implants ✅
1934. question about a pt he has deep overbite that came with 5 amalgam restoration with wear facet and gonna be


replaced. What is the important step? (previously mentioned)
a. Occlusal assessment
b. Adaptation of the marginal walls
c. Impressive faulty
1935. patient received his mandibular overdenture, during follow up visit he was complaining that's in the first days
after over denture delivery he faced difficulty during wearing and removing the denture. however, the denture now is
not retentive at all. What’s the cause of this problem? ( picture of one of them was tilted) alveolar ridge
resorption(previously mentioned)


a. loose ball attachment
b. distortion of the rubber the implants are not parallel
1936. healthy female patient came to her first visit in dental clinics she feels dizzy, pallor , Nausea,(previously


mentioned)
a. Syncope
1937. What makes caoh superior to the other linear material ?(previously mentioned)
a. Thermal insulator to pulp
b. Chemical insulator to pulp


c. Sedative to the pulp
d. formation of secondary dentin
1938. 15 years old female came with extensive proximal Caries. What is the most appropriate to?(previously


mentioned)
a. Caries excavation and temporization before complete of treatment plane
b. Diet control delay treatment and revelation
c. cast metal crown for all teeth
d. Amalgam restoration for all teeth 73


1939. Pic of look like hypercementosis ask about management (previously mentioned)
a. Follow up (if it is general in all teeth)
b. Extraction
c. RCT ( if in single tooth and restorable condensing osteitis)
1940. Pic of pericoronitis and it’s asymptomatic and the patient is asymptomatic and there is a history of reoccurrence
(previously mentioned)


a. excise the lesion and allow the thirds to fully erupt -
b. .extraction of the third molar
c. excise the lesion and give antibiotics
d. give the patient antibiotic and extract after 1 week


1941. Pt bites suddenly on olive seed and feels pain this is due to?(previously mentioned)
a. Acute trauma from occlusion
b. Chronic trauma from occlusion
1942. Patient have breast cancer and take (biophostanate ) since 2 years we want to avoid osteonecrosis , he has a


hopeless tooth to extract What’s the best management?(previously mentioned)
a. Grind the tooth till subgingival level
b. Change to oral medication
c. Give 2 months holiday of the medication
d. Give antibiotics


1943. Most effective method for caries prevention? (previously mentioned)
a. Water fluoridation
b. Toothpaste fluoridation
1944. Patient instructed to say “Ahh” , noticed one side of the uvula raises while the other doesn’t. What nerve causes


that? (previously mentioned)
a. Vagus nerve
b. Glossopharyngeal nerve
c. Hypoglossial nerve
1945. posterior palatal seal function (previously mentioned)


a. Support
b. Prevent gagging reflexes


1946. Proximal plate function (previously mentioned)
a. Stability
b. support
c. Retention


1947. Instruments come into contact with mucous membranes (previously mentioned)
a. Semicritical


1948. polysulfide should pour in(previously mentioned)
a. 1h
b. 1 week
c. 15 min
1949. A patient had pain in the upper right teeth early in the morning, after examination all upper teeth are sound .
What could be the diagnosis?(previously mentioned)
a. Myofascial pain dysfunction syndrome.


b. Sinusitis
c. TMD


1950. Pic of pericoronitis. What inital treatment is appropriate(previously mentioned)
a. irrigation and antibiotic and analgesic
1951. Pic of 25 Patient have ( smoothing like attrition and white pigmentation with little brown pigmentation) since
birth she is like this what she had ?(previously mentioned)
a. Amelogensis imperfects


b. Dentenogensis imperfects
c. enamel hypoplasia
d. Flouroisis
1952. crown and the next day he came complaining of slight deflection in rest this is in(previously mentioned)


a. Rest
b. Centric


1953. X ray deep pocket and large radiolucent between 32-33? (previously mentioned)
a. Lateral periodontal cyst


1954. Hypertension patient with tongue silicone appliance, for what?(previously mentioned)
a. Sleep apnea


1955. Pt with one side paralysis with rash which virus?(previously mentioned)
a. varicella-zoster
1956. Case about pt having ulcer heal in one week and with systemic involvement ( fevers , flu what is your diagnosis


have same last month ? (previously mentioned)
a. Recurrent herpes labialis


1957. Long case scenario about patient have Xerostomia what is best prosthetic replacement? (previously mentioned)
a. Implant
b. RPD
c. FPD
1958. Pic Female patient have trauma in She has loss 21 adjacent teeth are intact what is best prosthetic replacement


(previously mentioned)
a. Implant
b. RPD
c. FPD
1959. Which of the following considers caries high risk ?(previously mentioned)

a. Fluorosis
b. open contact
c. using miswak
1960. Third year resident OMFS have patient need extraction for third molar and he afraid if he tell the patient
complications will not agree and that will Lead to complications reach to resection of mandible what he should
do?.(previously mentioned)


a. Tell patient’s family
b. Tell patient and let patient decide


1961. Long scenario about major connector Half pear how to minimise trauma for tissue ? (previously mentioned)
a. distribution of for occlusal force
b. Wax
1962. which cement irritates the pulp (previously mentioned)
a. Resin
b. GIC


c. Polycarboxylate
d. Zinc Phosphate
1963. ptt has RPD ,came to the clinic complaining of discomfort on the abutment tooth , what is the reason?(previously
mentioned)
a. occlusion


b. exposed dentin under rest
c. galvanic shock due to amalgam
d. VRF
1964. Case that has aplastic anaemia,All results show normal Haemoglobin, normal WB and normal platelet and need


Endodontics procedures? (previously mentioned)
a. Do tx without modification
1965. patient who is healthy and has a healthy periodontium. He had anterior crowns placed a while ago and now
presents with spacing and other issues indicating trauma from occlusion. What type of occlusal trauma does he


have(previously mentioned)
a. Primary
b. Secondary
c. Combined
d. Persistent


1966. Gingival enlarged after ortho(previously mentioned)
a. gingivaectomy with External bevel
b. gingivaectomy with internal bevel
1967. Pt medically fit or mentally stable what is the thing that may determine the type of floss to be used manual


dexterity (previously mentioned)
a. pt preference
1968. Patient came to you while measuring probing depth, there is recession 4 mm. And pocket depth 3 What did


express?(previously mentioned)
a. 7 mm attachment loss
b. 7mm pseudo pocket
c. 2 pocket depth ,
d. 4 mm attachment loss
1969. pt 30 years old with little mental retardation and his psychologist said that he is capable of making his own


decisions, how while sign the consent ?(previously mentioned)
a. Patient


1970. .Patient with prosthetic heart valve need tx what in your management?(previously mentioned)
a. Antibiotics before 1 hours from procedure
b. No need
1971. Patient came with proximal white spot and in x ray there is nothing what should we do for him(previously
mentioned)
a. GIC restoration
b. Fluoridation ✅
1972. Clinical picture (occlusal view) of 2 molars with fallen crowns, mentioned that the patient is angry about the
repetitive fallen of the crowns every few months, what is the best long term treatment? (previously mentioned)


a. Recement with RelyX
b. Crown lengthening
c. Splint teeth together
d. Long post
1973. Case scenario about patient with generalised inflamed gingival pocket depth 3 recently he do implant with
pocket depth 5 what your management


a. Scaling and root planing , OHI
b. inject topical antibiotic on implant pocket and Chx mouthwash
1974. We use flux with soldering to?(previously mentioned)
a. to absorb hydrogen
b. To improve the flow


c. For wetting
d. reduce oxide layer
1975. new patient with no relevant history using inhaler, vital signs measured measured at beginning of the visit (
pressure 140 all other normal ) respiratory rate improving with rest, what is the medical condition :(previously


mentioned)
a. Asthma
b. Emphysema
c. Chronic bronchitis
d. TB


1976. child ingested whole bottle of mouthwash brought by mother to emergency:(previously mentioned)
a. give milk
b. Use activated charcoal
c. Induce vomiting
d. Admit the patient
1977. Implant for high risk periodontists pt, good results and stable after one year of recall visit, what is the recall
interval after the first year?(previously mentioned)


a. 1-2 months
b. 3-4 months
c. 6-7 months
d. 8-9 months
1978. preapical x-ray taken after one year of implant placement, resorption in mm consider alarm sign ?(previously
mentioned)
a. 0.5
b. 1


c. 1.5
d. 2
1979. maximum implant size to replace #14 when available space is 12mm from ridge to maxillary sinus?(previously
mentioned)
a. 9


b. 10
c. 11
d. 12


1980. 19 y.o presented with maxillary construction how to treat?
a. surgical assisted maxillary expansion
b. LeFort ...
c. Rapid expansion
d. Ortho treatment
1981. immediate RPD seated perfectly, pt remove it before sleep ,next morning patient unable to wear it. What could
be the reason?

a. patient lack of skills of placement
b. Because of inflammation and swelling after extraction
c. Contact defect
1982. positive ( Hbs-Ag , HB-igM ) and negative ( Hbc-Ag ) ?
a. susceptible


b. Chronic infection
c. Acute infection
d. Immune by recent vaccination
1983. Doctor having up-to-date vaccination, still with great risk infection from?


a. HIV
b. HBV
c. HCV
d. TB
1984. down syndrome what type of fluoride?


a. 10% stannous fluoride
b. 22.6% Sodium fluoride varnish
c. gel


1985. instruments sterilized by Type B autoclave instruments can be stored for?
a. 30 days
1986. x-ray of #17 ( not clear ) , asking how many expected root?


a. 2
b. 3


1987. expected canal numbers for #34.
a. 2
1988. definition of hemisection
1989. patient with history of oropharyngeal cancer treated three years ago, came with L defined radiolucent lesions in


mandible.
a. Osteoradionecrosis
1990. while giving IAN patient move suddenly and Dr stick his finger, first thing to do:


a. report incident
b. Encourage bleeding
c. Wash
1991. child after motorcycle accident, pic of complicated crown fracture with pinpoint pulp exposure what is the
treatment?


a. Pulectomy
b. Partial Pulpotomy with MTA
c. Dental pulp treatment with MTA ( didn't mention caping)
1992. generalize brown teeth appear yellow under ultraviolet light?
a. Amelogenesis imperfecta
b. Dentinogenesis


c. Fluorosis
d. Tetracycline staining
1993. Patient has Jaundice, increased heart rate, increased respiratory rate, hyper tension and finger clubbing, what’s
the cause?
a. Liver


b. Hyper tension
c. Congestive heart failure
d. Coronary artery disease


1994. Patient referred for extraction of 15, you extract 25, what is next step?
a. tell patient
b. tell staff member
1995. Pedo patient finished extraction, after one week he came again with his mother with swelling in the lower lip
a. Hematoma
b. Masticatory trauma ✅

1996. In which kennedy classification the altered cast technique can be used?
a. Class 1 Mandible
b. Class 1 Maxilla
c. Class 3 Mandible
d. Class 3 Maxilla
1997. Patient with CD, which kind of impression material can be used in functional impression?
a. Addition silicone


b. Compound
c. Alginate


1998. Which kind of occlusion is used with Wit’s analysis?
a. Functional occlusion


1999. Patient has blood oozing, and petechiae in the palate, which test can be used?
a. ALT (suspecting liver disease)
b. aPTT
c. INR


2000. What is the drug that can cause Thyrotoxicosis?(previously mentioned)
a. epinephrine


2001. What is the reason to do caries risk assessment before periodontal treatment?(previously mentioned)


a. To check deep pockets
b. To check any root caries
2002. Patient with Endotreared tooth #47, come with severe pain, he has isolated deep pocket in distobuccal area, in


x-ray he has bone loss distal to #47, what is your management(previously mentioned)
a. Extraction
b. GTR


2003. Patient has recession and reduced interdental papilla, what is the management?
a. esthetic periodontal surgery
b. Pocket reduction surgery
c. osseous surgery
2004. Which can affect the enamel and dentin and pulp in single tooth of the jaw?(previously mentioned)
a. amelogensis imperfecta


b. dentenogensis imperfecta
c. Regional odontodysplasia
d. Dental aplasia


2005. What is the difference between adult and children in orthodontics treatment?(previously mentioned)
a. movement of teeth in adult is slow
b. Adult need big bracket
2006. 10 years patient, has super facial caries in all first permanent molar, clinically it’s not cavitated, and also not clear
in x-ray ,what is the management?(previously mentioned)


a. Acid fissure sealant
b. preventive resin restoration
2007. 5 years patient, there is lateral shift of mandible when he close, what is the management?(previously mentioned)
a. expansion


b. delay until permanent erupt
c. disking interfering tooth
2008. Patient with breast cancer, and is under zometa IV, she finished extraction before few weeks, she has purulent


and pain, (the x-ray shows that necrosis reaching the ramos) what is the management?(previously mentioned)
a. Mandibular resection
b. Antibiotics
c. Mouth wash
2009. Asthmatic pt has an asthmatic attack and his bronchodilator inhaler is ineffective , what the management ?


a. Long acting beta2 adreng
b. short acting beta2 adrenergic
c. 02 flow
d. Call ER


2010. Patient has endo treated tooth #21, x-ray shows bad obturation and radiolucency, what is management?
a. re-rct
b. Lesion is large and indicates cyst enucleation
2011. What instrument is used to measure gingival thickness?
a. perio probe


b. ⁠explorer
c. ⁠casterviego caliber
2012. 4 years patient had intrusion trauma what could be the consequence to permanent? (previously mentioned)


a. devitalization of permanent
b. ⁠crown of permanent tipped palatally
2013. Patient with tuberculosis and active what should you do and its urgent and in pain ?


a. treat like normal patient
b. ⁠use infiltration mask
c. ⁠treat in isolated rooms


2014. 17 years patient compalined about implant mobilty she did 3 years ago ? (previously mentioned)
a. she is too young and underdeveloped


2015. pitted enamel treatment(previously mentioned)
a. macroabrasion
2016. Renal transplant pt taking 2 meds with a lesion on floor of mouth what is your management
a. Stop 1st med
b. Stop 2nd
c. Intralesional steroids
2017. Trismus bc of 3 molar what to how to resolve(previously mentioned)
a. Abx


b. Mouthwash
c. Extract
2018. Young Patient came to clinic saying “my teeth are shifting” and said his father had this condition before.
Radiograph showing large radiolucency. What could be the condition?


a. Cherubism
b. Ectodermal dysplasia
c. Cleidocranial dysplasia
2019. A 85 yo grandmother came to clinic with untreated oral cancer. The doctors confirmed her condition was
hopeless and decided to activate non-resuscitation order (NRO). Her family objected the decision when they were
informed. Before the conflict were resolved the patient went through cardiac arrest duo to …(I can’t remember the


reason).How could the nurse act?
a. call the resuscitation team
b. Take the case to court
c. Act based on chef nurse decision
d. Follow the NRO
2020. scenario of patient with bleeding disorder and a table showing RBCs, WBC and platelets beyond normal values.
The question was what’s the reason that bleeding won’t stop?


a. Erythrocytopenia
b. Thrombocytopenia
2021. what could initiate scrubbing procedure from the following?


a. Before the start of implant surgery
b. Before and after every patient
c. After bare hands touched instruments that could be unsterilized


2022. What’s the reason for corrosion of carbide burs?
a. Autoclave


2023. What indicator to determine the status of hepatitis B carrier
a. HBsAg
b. HBsAb
2024. You were in the hall and saw a patient searching for the clinic to place her falling crown and continued walking
without directing her. What did you violate?


a. Justice
b. Beneficence
c. Autonomy
d. Maleficence
2025. Patient came for endodontic treatment for #11. After giving anesthesia and right before placing rubber dam the
patient refused to placed and said he feel uncomfortable with it. What should the clinician do?
a. complete the treatment and isolate with cotton roll


b. call the police to take away the patient
c. politely refuse to complete the treatment
2026. A diabetic patient came to clinic and his condition is controlled by using Metformin. He is welling to come for
another visit if needed (not sure if it was elective procedure or extraction). What’s the appropriate action for the
patient in the next appointment?


a. Have breakfast with regular medication and add insulin
b. Have breakfast with regular medication


2027. needle stick injury during giving anesthesia, what’s the appropriate action?
a. Force blood to come out and then wash for several minutes
b. Force blood to come out and then scrub with soap and wash for several minutes


2028. What form when blunt injury?
a. Hematoma
b. Hemangioma
2029. (diagram of incision) and question was what’s the disadvantage of this incision?
a. Bleeding
b. Poor visibility


2030. colour of H-file size 60?
a. Blue
2031. (Picture of mesially impacted third molar) and the question was what will causes difficulty during extraction?
a. Widened PDL


b. The tooth has a conical shape
c. Too close to the second molar
2032. mother came to clinic with her 2 old child and said she sucks her thumb during sleep. What’s the appropriate


management of this case?
a. Defer treatment until she reach 8 years old.
b. Apply removable habit breaking appliance
c. Apply fixed habit breaking appliance
2033. A patient presented to the clinic requesting replacing a missing lower right posterior tooth with a fixed
prosthesis. On examination, tooth #46 was missing, tooth #47 was mesially tilted (30 degrees to the occlusal plane).
The plan was to replace this missing tooth with a 3-unit fixed dental prosthesis. Which of the following
complications would most likely affect the long-term prognosis of this prosthesis?
a. Sensitivity
b. Pulp exposure
c. Irreversible pulpits
d. Short distal axial wall
2034. Pt. have a peg lateral tooth on the right side with localized gingivitis, Doctor fabricated provisional crown,What
is the main objective to fabricate the provisional crown in this case ?
a. Improve the OH


b. Addresses the esthetic concern
c. Protect the pulp
2035. 26-year-old woman presented to the clinic requesting to whiten her teeth for her graduation day. Her teeth were
otherwise healthy, so it was agreed upon to perform in- office bleaching. 3 minute after initiating the procedure, with
adequate isolation the patient complains of severe pain and wants to stop. Which of the following is the most likely
cause of the pain?
a. Tooth sensitivity due to vital teeth
b. Increase PH of the bleaching agent


c. High intensity of the light activating system
d. Bleaching of the gingiva due to incorrect placement of plastic dam
2036. Case senario Diabetes pt uncontrolled come to clinic with bilateral painless enlargement of parotid gland since 4
months …. What is most common feature of in parotid gland biopsy:


a. atrophy acini
b. acinar hypertrophy
c. Lymphocyte infiltration


2037. Retraction of canine impaction , forced applied 400 . What this force lead to
a. hyalinization
2038. Pt with a nodular enlargement in the clavicle, tender-less, fixed and firm what could it be?
a. Lipoma
b. Multiple myeloma


c. Brukitt lymphoma
d. Non-hodgkin lymphoma
2039. What happens with increasing age?
a. Decrease gingival thickness
b. Increase gingival thickness.


c. Decrease attached gingiva
d. Increase attached gingiva


2040. Cauliflower-like projection in the lateral border of the tongue which is painless and changed in size ?
a. Squamous cell papilloma
b. Squamous cell carcinoma
2041. Pt. Has severe pain related to infectious tooth. After giving L.A with Lidocaine the pt.still feels the pain, the
anesthesia still un-profound. What is ur next step ?
a. Prescribe Analgesics and Antibiotics
b. Continue the procedure anyway


c. Schedule another appointment until symptoms subsides
d. Change anesthetic agents and techniques
2042. White lesion in the lateral surface of the patient’s tongue, he complains he can’t eat. What is the best


management?
a. take biopsy
b. give antifungal drugs.
c. Intralesional corticosteroids
2043. Pt. With odontogenic infection what is the best imaging modalities to perform investigation:
a. Panorama


b. PA
c. Computed Tomography scan with contrast
2044. Pediatric patient who have severe asthma on examination you noticed moon face, nick hump, wide shoulders,
abdomen is larger than his age, lower extremities are smaller than his upper body, what is the cause?
a. Insulin resistance
b. High cholesterol
c. Thyroid disease
d. Excessive systemic corticosteroids ✅
2045. Pt. came with proximal caries and has gingivitis. The dentist decided to do class II with composite. Which of the
following could affect the composite ?


a. No direct effect
b. Polymerization shrinkage will be compromised
2046. Craze line treatment
a. Occlusal reduction
b. Veneer


c. Full crown
d. No need


2047. Pt. has bleeding, swollen gingiva, loosened teeth :
a. Scurvy
b. Beri Beri
2048. ( Long case scenario ) the pt. has endo treated tooth with post and metal ceramic crown. Has an isolated pocket.
The Pt. suspected with VRF. What is the recommended investigation initially ?
a. PA
b. Bitewing


c. Occlusal radiograph
d. CBCT
2049. What is the main criteria to decide for extraction of the tooth ?


a. Tenderness
b. Caries to 3 surfaces
c. Pain
d. Swelling extends to buccal attached gingiva
2050. Pt. has impacted canine #23. What is the future expectation to gingival level of #23 if the dentist decided to do


disimpaction ?
a. Recession
b. Same as #13
c. Normal
d. Overgrowth


2051. HAART with HIV with condition related?
a. xerostomia
b. Hairy lekoplakia
c. Oral candidates
2052. Pt missing upper 2nd premolars and all molars need RPD?
a. neutrocentric occlusion
b. Mutually occlusion


c. Unilaterla occlusion
d. Bilaterla occlusion
2053. pt came for extraction with severe pain, had myocardial infarction 3 weeks ago and on aspirin?
a. Stop asprin


b. Defer treatment 90 days
c. Monitor ECG and nitroglycerin prophylactic
d. Antibiotics prophylaxis and exctraction

🌝✅
2054. Dr vaccinated had needle stick injury with pt have positive HBV what to do?
a. no thing
b. Anti bodies vaccine
c. Anti immune
d. Anti immune + anti bodies
2055. 7 years pedo cant opening mouth and mandible deviation what to do?


a. arthroplasty
b. Jaw exercise
2056. you decide root amputation/resection what is important?
a. Root anatomy


b. Gingival phenotype
c. Amount of keratinized tissue


2057. pt with oral, ocular and genital ulcer which test?
a. pathergy
b. immunofluorescence
2058. psedo class 3 management?


a. protraction of upper
b. Retraction lower
c. Lip bumper


2059. pt came ER after car accident, what is detected if an expected facial fracture?
a. malocclusion
b. Edema


2060. band and loop where is the location of the loop?
a. at contact
2061. distal first premolar and mesial second premolar need restoration and doctor decide to do both of them in one


visit what is the advantage for his decision ?
a. Can make the contact good


2062. class I RPD and 8 mm which major connector ?
a. Lingual bar


2063. RPD and anterior teeth periodontal compromise which major connector ?
a. Lingual plate
2064. bilateral missing all posterior teeth and the anterior teeth having mobility which you will use as indirect retainer
and the ?
a. rest on 43


b. Rest on 33
c. Lingual plate
d. Lingual bar


2065. picture of lower incisor tooth with recession what treatment ?
a. Root coverage


2066. patient came with ulcer in lower lip same picture ?
a. Fluocinonide ointment 0.05 %


2067. case with cancer and tooth non restorable what is the management to the tooth ?
a. Grind the tooth until sub gingival


2068. flowable composite prefer than packable composite in preventive restorative?
a. Less microleakage
b. Low filler content
2069. patient came with pain lingering before the end of the day by 30 mins and tooth is not sensitive to percussion


and normal apical tissue ?
a. pulpotomy
b. Pulpectomy


2070. Buccal mucosa with ulcer or some thing like that the case was about Systemic lupus erythematosus which test ?
a. Anti DNA and anti SM


2071. patient came with ulcer to orthodontic clinic having ulcer at the end of the buccal mucosa what is the cause ?
a. Extended wire


2072. patient with class II and lower crowding by 9 mm what treatment ?
a. Extraction upper 4 and lower 5


2073. peso patient with quad helix appliance and irritate the tongue what management?
a. Assure them this is normal


2074. 5 years pedo patient with primary teeth having problem about cross bite or some thing related to ortho ?
a. Patient too young come when he has mixed dentition


2075. lower retrognathia and tongue glossoptosis ?
a. Pierre robin syndrome

2076. canal 21 mm after cleaning and shaping the file reach until 19 mm what wrong happened ?
a. Ledge


2077. Endo ice 3 tetrafluoroethane temperature ?
a. -26.2%


2078. one tooth affected in the one quadrant in the patient 's mouth . What is the case ?
a. Regional odontodysplasia
2079. HIV patient with white corrugated in the border of the tongue what is the best treatment?


a. radiation
b. Follow up
c. Surgical excision


2080. case about child with class II and mandibular hyperdivergent ?
a. High pull head gear
2081. 3 years child with abscess in tooth lateral incisor what anaesthesia to do your treatment ?


a. anterior superior alveolar nerve block
b. anterior superior alveolar nerve block + nasoplataine nerve block
2082. sickle cell anemia what from the options can safely given in case for surgical procedure ?
a. salicylates.


b. Narcotic
c. Local anaesthetic with Epinephrine
d. barbiturates
2083. x ray with huge radiolucency with superior scalloped not affecting the roots of the teeth ?


a. Surgical removal
b. Marsipulization


2084. x ray with overhangs all the restoration what is your management to control the progress of the inflammation?
a. Remove overhang of all restoration


2085. you did MOD cavity and you see the best restoration to do is composite but not available ?
a. Do temporary restoration and schedule for appointment


2086. x ray with long impression coping to long what to do next ?
a. Open tray impression


2087. x ray of three implant and one of them with pocket and resorption ask what is the diagnosis not cause ?
a. Periimplantitis


2088. which instrument to crack in the crown root fracture , he didn’t mention any thing only clinically ?
a. tooth sloth
b. D16 explorer
c. D 1 explorer
2089. dentist Receive patient referred from ortho department to extract 16 and he did extraction to 26 by mistake after


that he give post surgical instructions and dismiss the patient ?
a. Malpractice
2090. picture of Gracey curette along not in the mouth and ask for which side
a. mesial to 16
2091. patient came with trismus and swelling and limited mouth opening after multiple injection before 1 day with


other dentist ask which space is affected in the picture ?
a. pterygomandibular space
b. Submassetric space


2092. acrylic compression in which stage ?
a. Dough stage


2093. patient came with ulcer in lip and gingiva and this happened before 1 year what is the diagnosis ?
a. Recurrent herpes infection


2094. sealer will affect on the tooth if it will be still for long time ?
a. Zinc oxide eugenol
2095. same question but he ask what is the minimum number to get good restoration without harming the gingival
during prosthetic crown preparation ?
a. 1-2
b. 3-4 ✅
c. 5-6
d. 7-8


2096. Furcal perforation management ?
a. Immediately by MTA


2097. patient taking clindamycin and having diaherha ?
a. Pseudomembranous colitis


2098. make the tooth extraction in child difficult ?
a. Long and divergent


2099. you treat tooth with pulp extirpating you find pulp stone ?
a. Explains to the patient and referred her to Endodontist


2100. picture of patient with submandibular space infection and fever 39 and the swelling started 2 days ago ?
a. Incision and drainage and treat the source and antibiotics


2101. TB patient and his lab result not finish until now and the case is urgent ?
a. Respiratory surgical mask
b. Room ventilation with normal face mask


2102. patient in drug zometa and old patient complain about intra oral discomfort ?
a. Bone necrosis


2103. which one from options is violation of patient confidentiality ?
a. Transmitted the patient information through Email
2104. case about patient feel sensitive to his teeth and he is a lot of times admitted to hospital and having anexity


disorder and wear of all his teeth with decrease in his lower fascia night what is the management ?
a. Referred to advance restoration treatment and psychology doctor
2105. patient has plaque and calculus and teeth destruction need restoration and crown which will be after finishing the


Ortho treatment ?
a. Crowns


2106. patient with bilateral posterior ridge resorption which occlusion you will instruct for him ?
a. Neutrocentric occlusion
2107. another question for patient with all his lower posterior teeth destroyed and premolars missing and he is going to
do full mouth rehabilitation and all anterior is present. How you will but the occlusion ?


a. Anterior jaw relation
b. Centric relation
c. Maximum intercuspation
d. Bilateral occlusion


2108. case with pedo and he has a molar with open roots and the tooth necrotic or irreversible pulpitis what treatment ?
a. Apexification
2109. asthma patient using inhaler came with feel discomfort with spicy food and acid in tongue what is the case and


treatment ?
a. Median rembiod glossitis > antifungal
b. Fissure tongue > assure him


2110. case for patient with liver cirrhosis will go for simple extraction what to do for him ?
a. Ask for INR before extraction ,


2111. picture of indurated lesion in patient smoke 20 years one packet per day what management ?
a. Biopsy


2112. picture of sealer puff from the middle of the canal asks what is that ?
a. Accessory canal


2113. picture first and second premolar having some thing and ask how many canal for on of them ?
a. It was 2 canal
2114. 2 question about dental trauma in permanent tooth and pup exposure 20 mins and other one 1 hour and was


small exposure what treatment ?
a. Direct pulp capping


2115. barbed broch function ?
a. Remove canal content

2116. dry socket case what management ?
a. Analgesic and irrigation
2117. implant placed in lower premolar area I think 34# and ask what injury will happens because the doctor didn’t do


graft for the bone resorption before placing implant ?
a. Mental foramen
b. mentalis nerve


2118. case about patient with cardiac stent what is considered about him ?
a. Do extraction without prophylactic


2119. question about which case need prophylactic ?
a. History of infective endocarditis


2120. cells responsible for bone resorption and remodelling ?
a. Osteoblast


2121. over extended Gutta percha why ?
a. No apical stop


2122. VIP patient from supervisor call you about him what you should do about him ?
a. Treat him like others and avoid discrimination


2123. remove smear layer for what ?
a. To let the selar pentrate dentinal tobule for sealing


2124. diabetic patient with pus discharge and pocket 7 mm and the tooth where response normally what management ?
a. incision and drainage


2125. abscess what is true in treatment ?
a. Incision in must fluctuating site


2126. iodophors why we didn’t recommend to put the instrument for 60 mins ?
a. not spirocidal
2127. space superior mylohyiod muscle and platesma inferiorly ?


a. submandibular space
b. Submental space
2128. 1.5 mobility which class


a. Class III
b. Class 2


2129. picture of female patient concern about anterior teeth and tell you what is the case she concerned about it ?
a. Short crown


2130. space given 12 and ask about implant length to the maxillary sinus ?
a. 11
2131. First step in preparing lithium disilicate for cementation?


a. Sandblasting
b. hydrofluoric acid
2132. Pic of Tunnelling procedure


2133. Case scenario pedo pt with delayed eruption of central + pic of odontoma on the central, management?
a. Excision


2134. Ameloblastoma case planned for incisional biopsy, what's an important step before?
a. Aspiration


2135. Case scenario edentulous pt feels severe pain in cheek, pain with touch
a. Trigeminal neuralgia


2136. Case scenario CD pt with severely resorbed ridge and feel numbness in lower lip?
a. Mental nerve


2137. Case pt referred from psychiatrist regarding bilateral cheek swelling?
a. Eating disorders


2138. Function of surveyors?
a. Undercuts + path of insertion


2139. While u do post prep sudden bleeding, what is the first step?
a. Take PA
2140. Pt complain of clicking LT side?
a. Anterior disc displacement with reduction ✅

2141. Avulsion case , 45 min extraoral time?
a. Immerse in sodium fluoride


2142. PT with covid?
a. Defer elective tx


2143. TB induration size?
a. more than 15 mm


2144. Severe Hemophilia B what factor given pre-op?
a. Factor IX


2145. Pedo pt with anterior open bite, sticking his tongue out, thumb sucking and rarely tongue thrust, whats the cause?
a. Thumb sucking


2146. Cause of papillary hyperplasia under denture?
a. Continuous wearing, not cleaning


2147. Down syndrome pt which fluoride u DONT use?
a. Mouthwash


2148. u broke file and manage to take it out?
a. Near miss


2149. 22 yo pt class II malocclusion, good facial profile, good lower teeth alignment?
a. Extract upper 4


2150. pic of stillmans cleft, management?
a. CT graft


2151. Pregnant pt, last month + pic of pyogenic granuloma interfering with biting, management?
a. Excision + remove local factors


2152. Diabetic pt with pic of periodontal abscess, management?
a. I&D
2153. Pt with osteoporosis and metastatic breast cancer and missing teeth, treatment modality? (OPG showing missing


posterior teeth all over)
a. RPD


2154. pedo pt apprehensive behaviour has carious molar, what resto?
a. SSC


2155. pic of #25 and 26 with open contact, pt diagnosed with localized moderate periodontitis whats the cause?
a. Open contact


2156. Most common attachment for mandibular overdenture?
a. Locator


2157. Maximum water line bacteria?
a. 500
2158. Pedo pt came after trauma with discolored gray crown, parents reported it was darker in color but its improving
now, what to do?


a. pulpotomy
b. Follow up


2159. Calculate LA MRD for pedo pt 20 kg using lidocaine 1:100000 epi
a. 3.8
2160. Case of pt with pericoronitis, dr explained that the definitive tx is extraction but as a temporary tx we can do


operculectomy, pt said she can’t afford exo and wants to do operculectomy as temp solution, what to do?
a. get informed consent and proceed with operculectomy


2161. HIV pt with white lesion in lateral border of the tongue?
a. Reassure


2162. Pt using pipe with inflammation of minor salivary glands in the palate?
a. Stomatitis necotina


2163. Which appliance is tooth borne ?
a. Hyrax


2164. pr wants to finish ortho quick?
a. Cortical perforation
2165. Pt complaining of pain in while opening her mouth, diagnosed with myofacsial pain syndrome with shifting of
the mouth to RT side, which muscle?


a. Medial pterygoid
b. Lateral


2166. First sign of gingival inflammation appear in?
a. 7-14 days


2167. Pregnant pt with medical issue needs to do abortion, dr discuss and took consent from her husband?
a. Violation of pt’s rights


2168. Gibgivectomy?
a. External bevel


2169. Pedo pt complain of impingement of lower teeth in palate?
a. Refer to ortho


2170. RPD design, from gingival margin to floor of mouth is 6mm?
a. Lingual plate


2171. Sterilisation of carbon post and burs ?
a. Dry heat
b. Autoclave
c. Chemical
2172. patient with stroke before one month want to do dental treatment?


a. give him antibiotic
b. delay 6month
c. do without antibiotic


2173. patient with organ transplant before 2month want to do elective dental treatment ?
a. Delay 6 month
b. do with antibiotic
2174. picture of lesion in lateral side and saying,patient with in uncontrolled epilepsy what name of it ?


a. chancre
b. esopilic ulceration
c. pemhegus
2175. young patient with trauma to upper centrals it make it go palatally crowns with mid root fracture What is the
treatment?


a. followup
b. splint with flexible for 4 weeks


2176. orthodontist refer patient to extraction second premolar and you mistake first premolar what should do ?
a. tell patient and reffer him to orthodontist
b. tell orthodontist to solve it
2177. A patient complains of bad taste and smell in the mouth a few days after replacement of a 3 unit fixed partial
denture. There is no other related pain or discomfort. However, bubbles are seen to appear in the cervical region of
one of the retainers upon application of water and occlusal pressure. Which of the following is the most likely
etiology of the patient complaint?


a. Open margins of the retainer
b. Loosing of the retainer on the abutment
c. Connector fracture adjacent to the retainer
d. Accumulation of food debris beneath the pontic
2178. An 11-year-old presented with a Class Il It skeletal malocclusion due to a retruded mandible.Which of the
following appliance can be useful for enhancing mandibular growth in this pa.ent?


a. head gear
b. functional appliance
c. protac.on face mask
d. fixed orthodon.c appliance
2179. A 56-year-old healthy man is presented to the clinic seeking implant treatment for missing lower right teeth.
Clinical examina.on revealed missing 451 and 46 probing depth range from 3-5 mm, generalized bleeding on probing
(see report)Radiograph: revealed generalized horizontal bone loss 20%. Which of the following is the most likely
treatment of choice?
a. request cone beam CT then proceed with implant treatment
b. request periapical radiographs then proceed with implant treatment


c. request orthopanotomogram(panorama) then proceed with implant
d. scalling root panning oral hygine instruc.ons then wait 4 weeks for the revalua.on visit


2180. Which of the following areas of the jaw exhibits the thinnest cortical plate wall
a. labial aspect of maxillary incisor
2181. after making the impression for single implant interocclusal space was found to be 4 mm. Which of the following


prosthe.c option is the most ideal treatment ?
a. screw retained crown


2182. what form of tissue is formed when there is strong carious odontoblasts are disrupted?
a. Reparative dentine
2183. A 64-year-old man came to the clinic complaing of pain related to tooth #35 Clinically there was a fuctuant
swelling related to In tooth #35. Non restorabl with a periapical abscess.Pa.ent has diabete type 1, pn high dosage of


insulin.He took his regular meal an Insulin dose.Which of the following is the most appropriate management?
a. incision and drainage extract tooth # 35 and prescribe antibiotic


2184. pystomatitis vegtans case with picture and it’s came with mild inflammatory bowel disease what is the treatment
a. Topical corticosteroids


2185. child with avulsion less than 1 hour for how many weeks you splint ?
a. 2 weeks


2186. enamel and dentine without pulp ?
a. Uncomplicated crown fracture


2187. enamel and dentine and exposed pulp ?
a. Complicated crown fracture
2188. case about premolar with metal post and aske about in which canal is placed according to SLOP technique and


provide 2 x ray
a. lingual
2189. during take impression for PFM crown there’s void in the marginal of facial surface in the impression what


management ?
a. Remake impression


2190. end renal disease with some infection in mouth and attach results Uramia was high ?
a. Urimic stomatitis


2191. patient did scaling before some days and came with deep pocket in lower molar and high faver ?
a. scaling and root planning and antibiotic
2192. furcation class I in molar 4 mm with bleeding in brushing and other teeth were normal depth 1-3 mm what


treatment ?
a. Scaling and root planing with odontoplasty
2193. furcation class II in molar with 4 mm depth with bleeding in brushing related to this tooth and good oral hygiene


what is treatment and picture provide is was slight ?
a. Refer to periodontist to do flap surgery
2194. dentist and assistant gut influenza and didn’t come to work and one patient came to clinic and get runny nose


what is the common rout of infection in dental clinic ?
a. Direct and indirect


2195. you are going to do laminate veneer with lingual extended were to put the contact area ?
a. one forth of the lingual surface and 1 mm away from centric contact
b. On third of the lingual surface and 1.5 mm away from centric contact
c. one forth of the lingual surface and 0.5 mm away from centric contact
d. Half of the lingual surface and 1 mm away from centric contact
2196. patient complaint from opaque in his incisal third what is the cause ?


a. insufficient opaque layer
b. One plane preparation
2197. flap is good for biopsy closed by primary intension ?
a. Elliptical ✅
2198. female patient came for check up or may be complain from biting and she has upper premolar previously treated
4 years ago and she did retreatmemt before 1 year and doctor used methylene blue dye and notic line extended from
the distal surface extended to tho tooth distal surface and isolated pocket and aske hat is prognosis of retreatemtn for


this tooth ?
a. Poor
2199. 9 years girl came with her mouther concern about crowding in her lower jaw the question was what is most tooth
is mal aligned in the case ?
a. canine
b. First premolar


c. Second premolar
d. Lateral


2200. patient stable and mentally sound and ask the dentist about which floss is good for her ?
a. Patient preference

2201. patient complain from pain in her upper right teeth when she wake up at morning and after oral examination all


teeth intact and no restoration What cause the pain ?
a. TMJ
b. Sinusitis
2202. child 6 years and very good oral hygiene and free from caries and drink water fluoridation what is supplemental


fluoride you will give in this case ?
a. 0
b. 0.25 mg /day
c. 0.50 mg/ day
d. 1 mg/ day

2203. patient discuss treatment plant and she sign the consent then she delayed the treatment and come after 6 months


which action is right about this case ?
a. Repeat and get new consent

2204. The treatment institution did not ask the employee for infectious disease vaccinations and did not ask about the


danger resulting from this action ?
a. Places employees vulnerable to infectious diseases
2205. 4th year resident want to extract third molar and he is afried the patient might refuse when he tell the patient


about complication that might cause parasthesia or injury to the canal what he should do ?
a. Disclose all the information and complication to the patient and referred him

2206. Wife with her husband have a case will go for RCT and post and crown and the husband tell the doctor to do


extraction cause the treatment plane will cost him what the doctor should do ?
a. Tell the patient (wife) about all the treatment and take the consent from her
2207. the patient came from another doctor. He did bad restoration with an overhang and the patient complained from


bleeding and inflammation what the doctor should do ?
a. Avoid criticism and tell the patient about her problem and explain what will be done about her case
2208. diabetic patient came and told she lost all her teeth that was mobilie what is the cause of her lost ?
a. Periodontitis ✅


2209. patient with diastema that cause by frenal attached and cause balancing what is the proper treatment ?
a. Frenectomy


2210. Hemophilia patient with less than 1% VIII which hemophilia he has ?
a. A
b. B
c. C
d. D


2211. where the submandibular gland duct open ?
a. enter the floor of the mouth under the the front of the tongue

2212. asthma patient and take beta blocker and long scenario and doctor administered local anaesthesia with


epinephrine and the blood pressure of the patient after that 149/89 what and he feel dizzy and headache what he has ?
a. Elevated blood pressure


2213. FPD with gold during metal try in there rocking what should do ? (mentioned previously)
a. Sectioning and soldering


2214. high of saturation what is that mean ?(mentioned previously)
a. Chroma
2215. doctor after finishing the LA injection he got an injury through the treatment. What should be done to prevent


this ?
a. Put in the natural zone


2216. The doctor get injury when he did IANB when he retract the cheek. What should be done to prevent this ?
a. Retract with dental instrument

2217. diabetic patient type 2 and everything normal and good oral hygiene and regular visit her doctor to check blood


sugar what should be done for him ?
a. Came morning after eating breakfast and take his insulin medication

2218. Cause of unilateral cross bite ?(mentioned previously)
a. Unilateral maxillary constriction


2219. 22 years having Revers anterior cross bite 9 mm how to treat the patient ? (mentioned previously)
a. Orthognathic surgery
2220. Patient is gonging to have complete denture and his lower posterior bone in both side occlude buccal to


maxillary bone how you will place the posterior teeth ? (mentioned previously)
a. Bilateral cross bite
2221. Patient misused his adhesive of his complete denture what is the adverse affect ?
a. xerostomia


b. Occlusal discrepancies
c. Loss biting or cutting efficiency
d. Stomatitis


2222. Diabetic patient what is the effect of perio treatment ?
a. Perio treatment will decrease the HbA1c

2223. Case about dental implant (crown ) the patient feel swilling and erythema and feel the crown is moved what is
the cause of this case ? (previously mentioned)


a. screw fracture
b. crown loosening
2224. Hyperthyroidism what you will expect in x ray ? (previously mentioned)
a. Widening in PDL


2225. Gic preferred has composite because it has ? (previously mentioned)
a. Fluoride leaching
b. Coefficient thermal expansion more than enamel
c. Coefficient thermal expansion more than dentine


2226. Which action is sympathetic? (previously mentioned)
a. Constrict pupils


2227. man have a car accident what you should do to prevent airway obstruction ? (previously mentioned)
a. Head tilt and chin left


2228. ortho band placed subgingival what is the affect ?
a. Gingival recession
2229. 17 years old patient came to ER complained form wire irritate buccal mucosa and dentist planned to cut the wire


which one from the equipment should he wear ?(previously mentioned)
a. Eye protection goggle
2230. implant absolute contraindicated in patient have cancer and gut treatment during the last 10 years and treated


with IV bishphosphonates and he is uncontroll diabetic type 2 ?
a. IV bishphosphonates
2231. I think case was about patient having headache and some thing on the trigeminal in one side with tmj pain area


what treatment ?(previously mentioned)
a. Antiviral
2232. dentist did small injury in floor of the mouth to patient once he move and the patient didn’t feel about that cause
of anaesthesia what dentist should do ?(previously mentioned)

a. Tell the patient this is an complications and i will do follow up for you
2233. child with gingival abscess related to primary molar what is the pulp condition that cause this case ?
a. Hypremia
b. Reversible pulpitis


c. Obiletrated
d. Irreversible pulpitits
2234. one case it think patient wake up and his gingival is bleed and having some pigmintation under his skin once the


dentist do oral examination he didn’t notice plaque or calculus what is the diagnosis ?
a. Leukaemia
2235. what is true about root formation ? (previously mentioned)
a. root formation once the tooth erupts in the occlusal


b. root formation once the tooth emerges in from the gingival
c. Root continue his formation when the tooth is moved
2236. pocket depth is 7 mm and the junctional epithelium within cemento enamel junction what is the type of pocket


?(previously mentioned)
a. Pseudo pocket
2237. recession 2 mm from cemento enamel junction and the pocket depth is 5 mm what is the CAL ? (previously


mentioned)
a. 7 mm clinical attachment loss
2238. perio case about patient smoke 1 packet a day or more than that i cant remember and he has generalised pocket


depth from 3-4 mm which stage and grade for this case ?
a. Stage II grade C


2239. Adult patient need upright to mesially tilted molar what make the uprighting molar take more time in this case ?
a. Reestablishing the occlusion
b. Patient age


2240. Hypertension patient have 14,15,16,17 ,26 need extraction
a. Extraction in one visit
b. Extraction upper in one vist and lower in Another visit
c. extract all under sediation
d. Extract upper 14,15 in visit and 16,17 in secend visit 26 in other visit


2241. implant in anterior teeth
a. endosteal


2242. Patient with attrition he have general severe pain especially in the cervical of teeth ?
a. Dentin hypersinsitive


2243. Patient class III maloclussion with hypoplastic max ?
a. Headger

b.


2244. Glossitis and lack filiform papilla
a. Exfoliative biopsy


2245. Solid ameloblastom treatment
a. Radical surgery with margin of 1 cm and resection of adjacent soft tissue


2246. MOD amalgame restoration with pain with biting x ray without any abnormal?
a. VRF
2247. Patient with FPD with roughnes and dental floss break on the clinicall dignosis there open margen with negative


ledge ?
a. Remake


2248. Impression material for flappy ridge in anterior ?
a. Mucostatic
2249. From the paipale to crest of bone in one implant?
a. Less 5 mm ✅

2250. Patient missing 12 she want replace by something inexpensive
a. Interm RPD


2251. Pedo patient with mother came to clinc with absses 2 week ago what the abuse ?
a. Neglecting


2252. Reciprocal arm fracture what are you afraid ?
a. Abutmant stability


2253. Distal edentulous ridge what is the clasp ?
a. Circum


2254. What disease is relieved when using aspirin
a. Odontoameloblastoma


2255. Centar healthcare system?
a. auxiliary team


2256. radiolucent in angle mandibule clear in opg what do before Biopsy it ?
a. MRI


2257. impacted 38 The doctor decided coronectomy instead extraction Why
a. Preseve root inside bone


2258. diabetic patient has calculus teeth severe bleeding deep pockt 5-6 whate mangment ?
a. mechanical clean +Chlorhexidine gely


2259. avulsion tow central 11-21 dr how deffrinte bw befor reimplantio
a. Crown rounded in destial side


2260. ANB =6 whiche class?
a. Class ii
b. Classiii

2261. Patient had lichen planus and use HYDROCORTISONE now complaine burn senstation ?


a. Chemical burn
b. Candidiasis
c. Nicotina stomatitis


2262. antibiotics beta-lactam ?(Mentioned before)
a. amoxicillin with Clavulanic acid

2263. Pedo patinte sever apprehensive And multiable caries mangment by ?


a. Ssc
b. Rmgic
c. Composite
d. Resine
2264. Pedo patinte 3years have caries in latearal insicor mesail and destial and mother concerne about esthtic area ?
a. Ssc
b. Composite
c. Resine face ssc ✅


2265. Amlgam restoration class ii with reccurant caries in proximal mangment ?
a. Gic
b. Compsite
c. Gold
d. Cast metal


2266. Whche Syndrom with Supernumerary teeth and exophthaloms ?
a. Crouzon syndrom
b. cleidocranial syndrom


2267. patinte with lingering pain and mild pain with bite x-ray large radiulcunte?
a. Symptoatic irrversible pulpitis with Symptoatic apical periodontits
b. irrversible pulpitis with Asymptoatic apical periodontits


2268. Metal sound after truma more in ?
a. Intrusion
b. Latrale laxution


2269. Vesiclobullus disease biopsy ?(mentioned before)


a. Intralesional then immunoflurecence
b. Perilesional them immunoflurecence
2270. dentisit didn’t utilize 2 plane reduction of tooth #44 for metal ceramic crown which result in over-prepared
tooth. 2 months after cementation the crown exhibit porcelain chepping on buccal side, What’s the reason ?
a. Unproper porcelain condensation
b. Unproper bond between porcelain and metal


c. Compromised frame work
d. Thick Unsupported porcelain on buccal side
2271. what happened to the interdental papilla when two teeth planed to moved apart by ortho ?(mentioned before )
a. Split and form deep pocket


b. Became fibrous
c. Adapt on the bone
2272. case with full acrylic denture, type of support ?


a. Tooth
b. Tissue
c. Hybrid
d. Primary
2273. extensive distructed #36 need crown , all other teeth are present and canine guidance occlusion, which articulator
to use?(mentioned before)
a. Hand articulator
b. Fully adjustable articulator


c. Non adjustable articulator
d. Semi adjustable articulator


2274. narrowest diameter of root canal ?
a. Apical constriction
b. Radiographic apex
c. Anatomic apex

2275. patient came complain of pain, swelling pus exudate from sinus opening, he has pharyngeal carcinoma and under
radiotherapy?


a. Sialosis
b. Bacterial sialedenitis
c. Viral sialedenitis
d. Fungal sialedenitis

e.
2276. child with interference in eccentric movement in the anterior teeth, manage by grind ?
a. Incisal edge of maxillary anterior

b. Incisal edge of mandibular anterior
c. Lingual incline of upper anterior
d. Lingual incline of lower anterior

2277. anterior implant placment ?


a. At midline
b. Lingual paplila


c. Insicive papilla
d. Lateral to midline


2278. facial eposodic sharp shock pain, trigger point .. ?
a. Neuralgia
b. Myofacial


2279. oral ulcer , genital and skin lesion , positive VDRL test , what medication to give?(mentioned before )
a. Pencillin
b. Prednison
c. Acyclover
d. Flocunazole
2280. mother concerne about her chile chin and asking if she has retruded mandible, what’s determin if the mandible is
retruded ?


a. SNA
b. SNB
c. SN-MP
2281. patient during dental treatment start sweating, dilated pupils, increased heart rate, aggressive and confusion,


headache... ?
a. Hypoglycemia
b. Hypothyroidism
c. Hyperventilation
2282. tb patient under medication but the test result not finish.. ?
a. Room ventilation
b. Face mask


c. Face shield
d. Respiratory mask


2283. patient fell and broken parasymphyseal bilatral , what most important to check?
a. Airway


2284. patient fell on his chin, what mostly broken ?
a. Condyle
b. Body
c. Angle
2285. ulcer recurre 2-3 time in months, how to manage?


a. triamcinolone acetonide
b. Acyclovir

2286. minocycline time in the pocket ?
a. 2


b. 7
c. 14
d. 30
2287. minimum HIV post injection prophylactic in weeks ?


a. 2
b. 4
c. 8
d. 6


2288. patient comply to all first year recall visits after implant, what the recommended interval after that ?
a. 1-2 mon


b. 3-4 mon
c. 5-6 mon
d. 7-8 mon


2289. GCF during inflammation ?
a. Increase gcf
b. Decrease gcf
c. No chamge
d. Totally disappear


2290. minimum between restoration margin and bone crest ?
a. 1-2
b. 3-4
c. 5-6
2291. best impression coping for esthetic zone ?
a. Plastic coping


b. Pick up coping
c. Custom coping


2292. pic of surgical guide for implant? Ask about the role…
a. For soft tissue
b. For hard tissue


2293. what happen if we neglect the anterior guidance in anterior teeth replacement?
a. Tmj problem
b. Misfit of crown
c. Esthetic issue
2294. chiled ingest 50mg flouride and her mother call the dentist, what to instruct her?


a. Reassure
b. Milk and go to emergency
c. Observation and go to emergency
2295. #21 with external root resorption and grade II mobility?


a. Extraction
b. Root canal treatment
2296. pt need to fabricate new complete denture, there is epulis fissuratum, why it’s important to remove it ?


a. To relieve pain
b. Good stability and retention to the new base


2297. instrument used to check canal flaring after preparation?
a. Spreader
b. Master file
c. Patency file
d. Master cone gutta perch


2298. route of HBV transmission?
a. Blood
b. Air
c. Direct intact skin contact


2299. History of trigeminal neuralgia linked to which disease
a. multiple sclerosis


2300. Human papilloma virus associated with ?
a. oropharyngeal


2301. Why do we remove overhang in restoration?
a. To Prevent plaque accumulation and periodontal disease

b.


2302. Patient with Liver cirrhosis needs extraction
a. INR test

b.


2303. location of the second canal in lower canine ?
a. Lingual


2304. bulbous crown, cervical constriction, obliterated pulp chamber
a. dentinogenesis imperfecta

b.


2305. Difference between primary and permanent tooth crown?
a. Bulbous crown


2306. Pt with lymphadenopathy drink unpasteurized milk. Diagnosis?
a. Scrofula
2307. Pt with COPD antibiotic was taken for 6 days came today for treatment BP was 130/80 Oxygen 95%What is the


proper management
a. Pulse oximetry

b.
2308. Pt with history of anterior RPD come with red palate ( picture was provided with erythmatous palate and sign of


pressure at incisive papillae) tx
a. Relief denture( remove from its inner part)
2309. Impression left for more than 15 min then poured. It was chalky and smooth..
a. Dehydration impression shrinkage ✅

2310. Implant to #16.. the time needs for complete osteointegration
a. 6 months
2311. Pt wearing denture for 2 years without any periodic visit. Complaining of tissue grows near his lower denture.


What is the preliminary management for him?
a. Trimming the irritated part of the denture


2312. Patient having edge to edge occlusion which type of restoration ?
a. Full crown


2313. patient insists on having veneers for life. How do you address this request?
a. Explain why this is not a good idea.
2314. Reducing cusp ?
a. Centric
b. Protrusive


c. Eccentric
d. Centric + eccentric


2315. X-ray of upper7 with radiopacity attach to palatal root. Tx?
a. Observation
2316. patient complaining of diastema, on x ray there is no mesodent, what is the most likely cause for diastema (no


other informations)
a. Low frenum attachment


2317. Pt with swelling legs ( pitting oedema) which disease?
a. Heart failure
2318. female patient with lower RPD replace the posterior teeth in quadrant 3 which opposing natural teeth,she
complain from interference when closing to the optimum condyle on the glenoid fossa, what is the type of


interference:
a. Centric relation
2319. Patient came and complain from pain and swelling in the area between #21 and #11, on radiographic


examination you find a unilocular 6mm radiolucency between 21 and 11, what is the most likely diagnosis:
a. Nasopalatine cyst
2320. Pt with missing 12, came complaining of missing tooth due to trauma and malalighnment teeth , protruded 11
with proximal caries, 13 is present with proximal caries , want to replace 12:
a. Implant


b. FPD
c. RPD
d. MARYLAND
2321. Lower 7 with finish RCT, PA was attached, it seemed that conservative access was done. intact mesial and distal


wall, asking about how to restore it?
a. Prefabricated parallel post with composite core
2322. When denuded cementom there will be exposed dentin and there will be communication between pulpal tissue
and PDL Which of the following will be the cause ?


a. VRF
b. developmental defect


2323. What makes pit and fissure sealant more retention?
a. Acid etch
b. Air abrasion
2324. Pt present to clinic with 3 unit ceramic bridge #11#22 with half incisal of #11 chipped, what will you do?


a. Remove and take a new impression
b. Composite repair at the clinic
c. Composite repair at the lab
2325. Pt 14 years came after 3 hours with an avulsed tooth, management?


a. RCT before reimplant
b. Soaked in 2% NaF for 20 min then reimplant
2326. Pt came for treatment of 41,31 didn’t mention which type of treatment ( resto lingual or facial) asking for what is
the appropriate dentist chair position?


a. 9clock


b. 7 clock facial surface
c. 12 clock lingual surface
d. 11clock


2327. Pt asthma using corticosteroids inhaler for 3 months came with pain and redness in the hard palate, Diagnosis?
a. Candida
2328. Child came to the clinic complaining from pain in the primary tooth, and has large, extensive caries. The


diagnosis is irreversible pulpitis, which type of x-ray should you take?
a. Periapical
b. bitewing
c. OPG


2329. Pt came to Er one day after cementing the PFM crown with pain and erythema what is the cause?
a. Metal allergy


2330. Pt came with pain and bone loss related to upper #16 which had an overhang restoration and medial 5 mm pd,
generalized bone loss and history of uncontrolled DM. Asking about the cause of bone resorption mesial to #16 ?
a. Diabetes


b. Periodontitis
c. Overhanging restoration


2331. a female patient done with perio treatment but still has multiple deep pockets , the periodontist decided to do


periodontal surgery. What is the goal ?
a. pocket reduction
2332. Pt came complaining of unilateral crossbite upon examination revealed midline shift with bilateral crossbite.
Treatment?
a. Posterior bite plane
b. Anterior bite plane


c. Unilateral expansion
d. Bilateral expansion


2333. Pt 7 years old with class iii with normal position of mandible what is the treatment?


a. Rapid expansion
b. Slow expansion


2334. Pt 12 years old with class iii with normal position of mandible what is the Treatment?
a. Rapid expansion
b. Slow expansion

2335. Pt with history of trauma an anterior tooth came complaining of discoloured tooth.. PA showed a calcified Canal.


What is the reason for yellowish tooth Colour?
a. Calcification


2336. Restoration needs polishing to prevent dehydration
a. GIC


2337. Pt with repeated LA injection yesterday, come today with pain and trismus What type of LA technique will used?
a. Vazi-rani-Akinos


2338. Implants and there is bleeding with probing the pocket will be?
a. Deeper than natural teeth


2339. INR 3.5 pt using warfarin
a. Do not stop warfare
b. Stop For 3 Days
c. Stop 5 Days
d. Stop 7 Days


2340. Pic of odontoma bt the 2 centrals what is Tx?


a. Marsupialization
b. Excisional biopsy


2341. Which disease uses Bisphosphonates?
a. Paget disease
b. Parkinson
2342. Most common dental emergency faced in the clinic


a. Hypoglycaemia
b. Vasovagal syncope
c. Hyperglycaemia
2343. diabetic pt came to the clinic seeking treatment, while she seated on the chair she felt dizzy and about to faint.


What’s immediate action?
a. Over her a juice


2344. Pt 25 years with Class III normal mandible tx ?
a. Lefort 1


2345. Which trauma causes pulp necrosis?
a. Avulsion
b. Concussion
c. Subluxation


2346. Internal resorption in central incisor treatment?
a. RCT
b. Pulpotomy


2347. Pt came with white pigments all around his teeth after he removed his ortho appliance. What is the most suitable


thing you can do?
a. Reinforce OHI


2348. HIV patient with Linear gingival Erythema that didn’t resolve ,what to give him ?
a. Fluconazole


2349. Case of RCT using ZnOE for planning and planned to place composite final restoration. After finishing the


obturation, the axial wall was finished with a high speed bur, why?
a. The logic answer was to remove remaining ZnOE from walls


2350. Platelets less than 25000
a. Thrombocytopenia


2351. CD with anterior teeth sets far labially
a. affects stability of denture

2352. Pt. has recent kidney failure? With buccal mucosa white patches.
a. Fungal infection


b. ⁠lichenoid drug induced medication
c. Ammonia


2353. Pt. has localized 7m.m. deep bucket in mesiobuccal root in lower molar with endo treated tooth. All teeth within
2-3 PD

a. Localized periodontitis
b. VRF
2354. Best antibiotic prescription indication:


a. acute infection
b. diffused fast growing infection
2355. Pt. came as an emergency with pain and not fluctuating swelling strata 2 days before.
a. abscess


b. Tumer
c. Cellulitis
d. Edema


2356. incisal upper front during smile it has to be in harmony with:
a. lower lip
b. ⁠upper lip
c. ⁠upper gingival something


2357. V instead of F


a. front teeth more incisally
b. ⁠front teeth more cervically


2358. mobile hopeless 31 removed in RPD pt.:(previously mentioned)


a. repeat the whole thing
b. stable a canine and wrought wire clasp.
2359. pt came with 2-3 bone loss under the CEJ in some areas:
a. within normal
b. ⁠bone reshabing


2360. mesial resorption and distal build:(previously mentioned)
a. Physiologic mesial migration


2361. case taking bisphosphonate with stage 4 and pathological mandibular fracture:(previously mentioned)
a. mandible resection


2362. Treatment of oral lesion of inflammatory bowel disease ?(previously mentioned)
a. Systemic steroid


2363. Importance of Occlusal Index wax?(previously mentioned)
a. orientation of facebow records


2364. Which first retentive or reciprocal arm?(previously mentioned)
a. Reciprocal


2365. What is the effect of steroids on DM patients?(previously mentioned)
a. Increase blood sugar


2366. Mediator of gingival inflammation?(previously mentioned)
a. Prostaglandin
2367. Causative bacteria of gingivitis ?(previously mentioned)
a. Fusobacterium
b. P.gingivals T.denticola
2368. patient with good oral hygiene complain of gingival bleeding she brushes her teeth 2 daily with whitening


toothpaste(previously mentioned)
a. Plasma cell gingivitis
b. Plaque induced gingivitis
c. Lichen planus


2369. Ameloblastoma management after 2D xray?(previously mentioned)
a. biopsy
b. CBCT


2370. Missing 23 and 33, will the patient will replace it with FPD 21-x-24 , 31-x-34?(previously mentioned)
a. maxilla more movement because of violation of ante’s law
2371. Class V in old patient which was restored with composite long ago but now it falled down with arrested caries
below what is the best resto(previously mentioned)


a. GIC
b. Composite
2372. Proximal caries in #36 you will put amalgam but after you prepared the proximal box there was less than 1.6


marginal ridge you change plan to onlay, what you will modify in the mesial and distal wall (previously mentioned)
a. diverge occ
b. Converger occ
c. oblique
d. paralla
2373. Female pt had a trauma that caused avulsion in the upper anterior tooth , pt had a problem with the phonetics
and she had severe bone resorption in this area but she dont want to do any bone grafting , what prosthesis you will
put(previously mentioned)
a. Implant


b. FPD
c. sectional rpd
d. convensional rpd
2374. Case scenario about pt with fever and rashes in the extremities and purulent tonsils and hyperplastic fungiform
papilla of the tongue(previously mentioned)


a. Rubella
b. wegener granulomatosis
c. mucosarcosis


2375. Pic of opg with simple bone cyst and ask about the management(previously mentioned)
a. surgical removal
b. follow up
c. Steroid
d. antibiotic


2376. 90 mcv and ask about the type of anemia(previously mentioned)
a. GP6D
2377. HIV pt who is controlled and the presence of the virus is almost not there , and the doctor has needle stick injury
what you will do (previously mentioned)


a. wash and check the virus activity from the pt
b. Encourage bleeding then seek infectious specialist
2378. Pt want to replace #12 with 3-units bridge from #11-13 , in the metal try-in stage there was a bubble what does
that mean
a. increase luting space of #11


2379. Blood in the pillow after sleeping from an old pt in the last 6 months
a. Haemophilia
b. periodontitis
2380. Pedo pt with complicated crown fracture came within 3 hours with small pulp exposure what is the management
a. conventional RCT with apical surgery


b. RCT
c. vital pulp therapy
d. apexification
2381. PA of central with open apex had trauma before and the last dentist did something what is it - there was
something filling the root to the middle only
a. endo regeneration


b. pulpectomy
c. apexogenesis
d. , RCt
2382. 4 y,o pedo felt on the tooth , and a little intrusion caused the primary to touch the permanent puds ,what is the


management
a. extract the primary carefully
b. repo the primary and splint
2383. You want to send a prosthesis to the lap, how you will disinfect
a. steam autoclave


b. dry autoclave ,
c. glutaraldehyde
2384. question about male pt came with generalized recession and the height of the interdental papilla is reduced what


you will do
a. aesthetic surgery
b. reduction of the pocket
2385. after you put cotton to stop bleeding in pulpotomy and the bleeding didn’t stop and become darker red , what is


this indicate for?
a. pulpectomy
2386. patient got hit in his right side with bleeding one eye with a trismus what what kind of fracture is this?
a. Lefort 1
b. Lefort 2


c. Lefort 3
d. Zygomatic fracture
2387. the patient did crown in this upper molar and when the patient close his jaw go anterior-superior what type of


interference is this ?
a. centric
b. eccentric
c. Protrusive
d. working


2388. Pt come to clinic with broken post and core ( photo of metallic post attached with metallic core ) and ask ttt ?
a. abricate new fiberpost
b. cementation with GIC
2389. When he remove the crown there wase (GI core discolored and poor margins . No pain or PA pathosis and tooth
with good condition then Ask about ttt of core ?
a. new composite core
b. new amalgam core


c. correct margins with same material only and fabricate crown
d. Referral for restorability

You might also like